You are on page 1of 116

4 Introduction to the Divergence Theorem and Stokes’ Theorem (Short)

Author: F. P. Dawson & F. Bekmambetova


Table of Contents
Introduction ................................................................................................................................................ 4
1 Part 1: General Overview .................................................................................................................... 5
1.1 Teaching Pedagogy ..................................................................................................................... 6
1.2 A Geometric Approach and the Process of Discovery ................................................................ 6
1.3 Graphing of Vector Fields ........................................................................................................... 8
1.3.1 Vector Fields in 2d ............................................................................................................... 9
1.3.2 Vector Fields in 3d ............................................................................................................. 11
1.3.3 Relationship Between Gradient Fields and Equipotential Curves (2d)/Surfaces (3d) ........ 12
1.3.4 Example: Normal and tangent vectors .............................................................................. 13
1.3.5 Problems from Section 17.1: 23,29, 39, 53, 54, 55 ............................................................ 14
1.4 Definitions ................................................................................................................................ 16
1.4.1 Flux and Circulation and Their Units.................................................................................. 18
1.4.2 Scalar Densities and Their Units ........................................................................................ 22
1.5 Derivation of Contour Integral from a Riemann Sum and Examples ........................................ 23
1.5.1 Example Contour Integral Problems.................................................................................. 29
1.5.2 Problems from Section 17.2: 1, 2, 18, 23, 27, 35, 39, 43, 49, 57, 65, 75, 77, 79 ................ 33
1.6 Derivation of a Surface Integral from a Riemann Sum and Examples ...................................... 35
1.6.1 Example Problems on Surface Parameterization .............................................................. 37
1.6.2 Problems from Section 17.6: 3, 5, 9, 11, 17 ....................................................................... 38
1.6.3 Surface Integrals of Scalar-Valued Functions .................................................................... 39
1.6.4 Computing Flux ................................................................................................................. 43
1.6.5 Example Problems ............................................................................................................. 47
1.6.6 Problems from Section 17.6: 19,23,27,32,37,41,43,45,53,55,56,69,71............................. 51
1.7 Convention on Assigning the Direction of Tangent and Normal Vectors ................................. 53
1.8 Properties of a Vector Field ...................................................................................................... 53
1.8.1 Description of 𝑭𝑭𝑭𝑭𝑭𝑭𝑭𝑭𝑭𝑭 ........................................................................................................ 54
1.8.2 Description of 𝑭𝑭𝑭𝑭𝑭𝑭𝑭𝑭𝑭𝑭 ......................................................................................................... 55
1.8.3 Computing 𝑪𝑪𝑪𝑪𝑪𝑪𝑪𝑪𝑪𝑪𝑪𝑪𝑪𝑪𝑪𝑪𝑪𝑪𝑪𝑪𝑪𝑪 and 𝑭𝑭𝑭𝑭𝑭𝑭𝑭𝑭 ................................................................................. 57
1.8.4 Making Circulation and Flux Independent of radius 𝒓𝒓 ....................................................... 58

1
1.9 Definitions for the Divergence and Curl Operators in 2d .......................................................... 60
1.9.1 Divergence of a Vector Field in 2d ..................................................................................... 61
1.9.2 Curl of a Vector Field in 2d ................................................................................................ 61
1.9.3 Problems from Section 17.5: 3, 6, 11, 15, 19, 21, 23, 29, 33, 39, 49, 59, 60, 63 ................ 62
1.10 Introduction to Helmholtz’s Decomposition Theorem .............................................................. 64
1.11 Course Content Going Forward ................................................................................................ 66
2 Part 2: A Formal Description of First Order Differential Operators .................................................. 67
2.1 Gradient of a Scalar Potential: 𝛁𝛁𝝓𝝓 ............................................................................................ 67
2.2 Divergence and Curl of a Vector Field ...................................................................................... 67
2.3 Definition of Divergence of a Vector Field: 𝛁𝛁 ∙ 𝑭𝑭 ..................................................................... 69
2.3.1 2d vector field ................................................................................................................... 70
2.3.2 3d vector field ................................................................................................................... 71
2.4 Definition of Curl of a Vector Field: 𝛁𝛁 × 𝑭𝑭 .............................................................................. 72
2.4.1 2d vector field ................................................................................................................... 72
2.4.2 3d vector field ................................................................................................................... 73
2.5 Exercise: Compute 𝛁𝛁 × 𝑭𝑭𝑭𝑭 and 𝛁𝛁 × 𝑭𝑭𝑭𝑭 .................................................................................... 75
2.6 Impact of the Infinitesimal Volume’s Shape ............................................................................. 76
3 Part 3: Vector Identities .................................................................................................................... 76
3.1 Scalars or Vectors Generated by Combinations of Addition, Dot Products and Cross Products 77
3.2 Differentiation ........................................................................................................................... 78
3.2.1 Gradient ............................................................................................................................ 78
3.2.2 Divergence ........................................................................................................................ 78
3.2.3 Curl .................................................................................................................................... 79
3.2.4 Second derivatives ............................................................................................................ 79
3.2.5 Third derivatives................................................................................................................ 79
3.3 Exercises to Prove Vector Identities.......................................................................................... 79
4 Part 4: The Divergence Theorem and Stokes’ Theorem, Geometric Characterization of Fields, and a
Sampling of Physical Laws ....................................................................................................................... 80
4.1 Divergence Theorem ................................................................................................................. 83
4.1.1 3d formulation .................................................................................................................. 83
4.1.2 Exercises Illustrating the Use of the Divergence Theorem ................................................ 84
4.1.3 Problems from Section 17.8: 1, 3, 9, 11, 17, 21, 29, 31, 35, 39 (a, c, d, e), 47 ................... 87

2
4.1.4 2d formulation .................................................................................................................. 89
4.1.5 Green’s Flux Theorem ....................................................................................................... 92
4.1.6 Example Problems ............................................................................................................. 92
4.1.7 Problems from Section 17.4: 7, 43 (flux), 47 (flux), 52, 65 ................................................ 92
4.2 Stokes’ Theorem ....................................................................................................................... 93
4.2.1 3d formulation .................................................................................................................. 93
4.2.2 Exercises Illustrating the Use of Stokes Theorem .............................................................. 94
4.2.3 Problems from Section 17.7: 1, 3, 5, 9, 13, 15, 19, 29(a-c), 33, 41, 42, 43, 45, 47, 50 ..... 101
4.2.4 2d formulation ................................................................................................................ 104
4.2.5 Green’s Circulation Theorem .......................................................................................... 105
4.2.6 Example Green’s Circulation Problems ........................................................................... 106
4.2.7 Problems from Section 17.4: 1, 6, 17, 23, 35, 43 (circulation), 47 (circulation), 53 ......... 107
4.3 Conservative Vector Fields (Irrotational Vector Fields).......................................................... 107
4.4 Source Free Vector Field......................................................................................................... 107
4.5 Computing the Field 𝑭𝑭 Given the Curl and Divergence of the Vector Field 𝑭𝑭 ........................ 108
4.6 A Sampling of Applications that Employ Vector Calculus Expressions ................................. 108
4.7 Applying Divergence and Stokes’ Theorem if Region is Not Simply Connected or Simply
Enclosed or the Region Contains Problematic Points ......................................................................... 108
4.7.1 Example Problems ........................................................................................................... 108
4.7.2 Problems from Section 17.8: 27, Section 17.4: 66, 67, 17.8: 39b .................................... 109
4.8 Identities Exploiting the Divergence Theorem and Stokes’ Theorem ..................................... 110
4.9 Time Derivatives of Integral Quantities .................................................................................. 110

© Francis Dawson, 2022

3
Introduction

This set of notes is divided into four parts. The parts highlighted in red are in the expanded set of
notes and are not required reading.

Part 1:
• Overview of new concepts and mathematical tools in preparation for Parts 2-4
o Teaching pedagogy: top down versus bottom up
o Role that geometry plays and how discoveries are made
o Graphing of vector fields
o Definitions used for integrating the dot product of two vector fields: single integrals
and double integrals
o Integrating over a curved line
o Integrating over a curved surface
o Conventions applied to vectors that are tangent and normal to a curve
o Geometrical properties of a vector field: circulation and flux
o Definitions of the divergence and curl operators which are linked to circulation and
flux respectively
o Helmholtz’s decomposition theorem:
 Any vector field can be decomposed into two independent vector fields with
different physical properties: the circulation free and source free
components.
o Description of course content going forward.
Part 2
• Importance of vector operators and relevant practical applications
o The curl of a vector field is like a cross product operation: it operates on a vector
field to produce a second vector field.
o The divergence of a vector field is like a dot product operation: it operates on a
vector field to produce a scalar function.
o The gradient operator operates on a scalar function to produce a vector field.

Part 3
• Important vector identities involving the gradient, divergence, curl, and Laplacian
operators that are important for this course and some future courses.

Part 4

• The details of the 3d Divergence Theorem including the connection between this theorem
and its 2d equivalent, Green’s Flux Theorem, as applied to well behaved regions: section
4.1,

4
• The details of the 3d Stokes’ Theorem including the connection between this theorem and
the 2d equivalent Green’s Circulation Theorem, as applied to well behaved regions:
section 4.2,
• The mathematical definition of an irrotational field: section 4.3,
• The mathematical definition of a source free field: section 4.4,
• A more detailed description of the Helmholtz Decomposition process and how it can be
used to determine the field: section 4.5,
• Modeling of the physical world using vector calculus expressions and an overview of
solution techniques: section 4.6,
• Applying the Divergence and Stokes’ Theorem for a region which is not simply connected
or simply enclosed, or the region contains indefinite points: section 4.7
• A summary of the most common identities that are derived from the Divergence Theorem
or Stokes’ Theorem, including their proofs, in section 4.8,
• The derivation of the time derivative of integral quantities, in section 4.9.

Testable material

There are some worked out exercises embedded within this Reading Assignment that you should
work out on your own and understand. The solutions to these problems can be found at this
location: Solutions to Assignment 4. These problems are typical in terms of testable material and
some of these problems will be reviewed in the applicable tutorial. Refer to Table 4 in the Schedule
document regarding the material that you will be tested on.

1 Part 1: General Overview


This set of notes lays the groundwork for the remaining part of the course. Up to this point we
have computed areas, volumes, and the physical characteristics of objects such as, for example,
the mass of a planar object (a 2d problem) or the mass of a spherical object (a 3d problem). Our
next challenge, from a geometrical perspective, is to investigate the variety of problems that can
be solved when working with curved lines and curved surfaces. Examples include, calculating the
length of a curved line or the total charge on a curved line, the area of a curved surface or the total
charge on a curved surface.

Curved lines can exist in 2d or 3d. In contrast, curved surfaces only exist in a 3d environment.
Every infinitesimal patch on a curved surface has an orientation and thus must have the properties
of a vector: magnitude and orientation. In the 2d case, the differential area lies in the 𝑥𝑥𝑥𝑥 plane,
however the third dimension does not exist and thus the differential area is a scalar with no
orientation.

5
1.1 Teaching Pedagogy

The textbook teaches the material in Chapter 17 the following sequence: 17.1 Vector Fields, 17.2
Line Integrals, 17.3 Conservative Fields, 17.4 Green’s Theorem, 17.5 Divergence and Curl, 17.6
Surface Integrals, 17.7 Stokes’ Theorem and 17.8 Divergence Theorem. Note, Section 17.5
introduces Green’s Theorems (Circulation and Flux) as viewed in a 2d world, without making
reference to the curl of a vector field.
The basic visualization skills and computational tools that you need to master are contained in Part
1 of these notes: integrating a scalar value along a curved line and integrating a scalar value over
a curved surface. The scalar value can be a physical scalar or represent the dot product between a
vector and a differential geometric vector such as an oriented differential length or oriented
differential area. The concept of circulation and flux are developed in the process. The general
properties of the divergence vector operator 𝛁𝛁 ∙ and the curl vector operator 𝛁𝛁 × and their
connection to the concepts of flux and circulation respectively are described in more detail than
the textbook. This is followed by a summary of common vector identities. Some of these identities
are referred to later in these notes.

We show that Green’s Circulation Theorem and Flux Theorem can be obtained directly from
Stokes’ theorem (in 3d) and the Divergence Theorem (in 3d) respectively. The concept of a
Conservative Field and a Source free field are introduced in the context of Helmholtz’s
Decomposition Theorem. This theorem is not covered in the text book. The properties of
Conservative and Source free fields are discussed in more detail in the expanded set of notes. You
are not responsible for these details.

These are the only notes you will require for this part of the course. You are welcome to look
at Chapter 17 of the textbook if you want additional worked out examples. Remember that
the sequencing of the material and exercises, at the end of each section, is different and this
could make things very confusing to the student.

1.2 A Geometric Approach and the Process of Discovery

New abstract concepts are introduced in the context of the observable world and to do this we rely
heavily on geometrical properties.

Many of the great discoveries in physics were initiated by looking for patterns in the observable.
Mathematics created the framework for performing the calculations. This line of thinking became
a branch of science referred to as Mathematical Physics. In this discipline, physics and
mathematics are treated as a single entity. Mathematics can also explore ideas that may have no
immediate practical value.

6
Table 1 Geometric Objects of Interest

Tangent vector in 2d Tangent vector in 3d


(open contour) (open contour)

Normal vector in 2d
(open contour)

Table 1 introduces some general geometries and differential vectors of interest. Definitions are
summarized in Section 1.4. Intuitively we make the following decision: we wish to explore vectors
which are tangent or normal to a geometric entity such as a contour or a surface. The tangent and
normal vectors are the simplest vectors to observationally connect with.

7
Along with the geometrical objects of interest, we need to establish some definitions. You will
notice in the definitions outlined in Section 1.4, that things are organized in opposite pairs such as
(open, closed), (continuous/discontinuous), (right-to-left, left-to-right), (CW, CCW), (tangent,
normal), (in, out), (up, down) and (right, left).

1.3 Graphing of Vector Fields

In this section we give you some guidelines for constructing graphs to depict 2d and 3d vector
fields. Examples are also provided. A review of the relationship between gradient fields and
equipotential level curves (2d) and equipotential surfaces (3d), from the perspective of graphing,
is also provided. The section ends with a selection of problems and solutions. If you feel
comfortable with this material, you can move on to Section 1.4. You will not be tested on this
section, but you will need to know how to sketch simple vector fields, going forward.

Steps to Plot a Vector Field in 2d and 3d

We begin by giving some motivation for what a graphical representation of a vector field implies.
Vector fields represent many types of physical problems such as fluid flow, heat flow, gravitational
field, electric field, to name a few. For example, if 𝑭𝑭 , a vector field, represents the velocity of a
fluid moving in two dimensions, then the graph of the vector field gives a snapshot of how a small
object, such as a cork, moves with the fluid. At every point of the vector field, a particle moves in
the direction of the arrow at that point with a speed equal to the length of the arrow. For this reason,
vector fields are sometimes called flows. When sketching vector fields, it is often useful to draw
continuous curves that are aligned with the vector field. Such curves are called flow curves or
streamlines.

The discussion above also applies to 3d vector fields. The resultant graph may become too
cluttered, with arrows placed in 3d space, and so the visualization of the vector field pattern in 3d
becomes too difficult to interpret.

Here we outline a brief step-by-step method to graph a vector field.

Step 1: Identify a point (𝑥𝑥, 𝑦𝑦) or a point (𝑥𝑥, 𝑦𝑦, 𝑧𝑧) for the 2d and 3d case respectively. Now place a
dot at this point.

Step 2: Determine the direction and magnitude of the vector field value at the point (𝑥𝑥, 𝑦𝑦), for the
2d case, or at the point (𝑥𝑥, 𝑦𝑦, 𝑧𝑧), for the 3d case: the vector field for the 2d case is 𝑭𝑭 =
〈𝑓𝑓(𝑥𝑥, 𝑦𝑦), 𝑔𝑔(𝑥𝑥, 𝑦𝑦)〉 and for the 3d case 𝑭𝑭 = 〈𝑓𝑓 (𝑥𝑥, 𝑦𝑦, 𝑧𝑧), 𝑔𝑔(𝑥𝑥, 𝑦𝑦, 𝑧𝑧), ℎ(𝑥𝑥, 𝑦𝑦, 𝑧𝑧)〉.

Step 3: Place the tail of the vector at the point marked out in Step 1 and mark off the head. The
length of the vector is proportional to its magnitude at that point.

8
Step 4: Repeats Steps 1-3 at different points until sufficient vectors have been plotted to get a
picture of the field pattern.

Note 1: Drawing vectors with their actual length often leads to cluttered pictures of vector fields.
For this reason, most of the vector fields are typically illustrated with proportional scaling: all
vectors are multiplied by a scalar chosen to make the vector field as understandable as possible.
Note 2: A useful observation for two-dimensional vector fields 𝑭𝑭 = 〈𝑓𝑓, 𝑔𝑔〉 is that the slope of
𝑔𝑔(𝑥𝑥,𝑦𝑦)
the vector at (𝑥𝑥, 𝑦𝑦) is 𝑓𝑓(𝑥𝑥,𝑦𝑦).

1.3.1 Vector Fields in 2d

Definition: Vector Field in Two Dimensions

Let 𝑓𝑓 and 𝑔𝑔 be defined on a region 𝑅𝑅 of ℝ2 . A vector field in ℝ2 is a function 𝑭𝑭 that assigns to


each point in 𝑅𝑅 a vector 𝑭𝑭 = 〈𝑓𝑓(𝑥𝑥, 𝑦𝑦), 𝑔𝑔(𝑥𝑥, 𝑦𝑦)〉. The vector field is written as

𝑭𝑭(𝑥𝑥, 𝑦𝑦) = 〈𝑓𝑓(𝑥𝑥, 𝑦𝑦), 𝑔𝑔(𝑥𝑥, 𝑦𝑦)〉 or


𝑭𝑭(𝑥𝑥, 𝑦𝑦) = 𝑓𝑓 (𝑥𝑥, 𝑦𝑦)𝒊𝒊 + 𝑔𝑔(𝑥𝑥, 𝑦𝑦)𝒋𝒋

A vector field 𝑭𝑭 = 〈𝑓𝑓, 𝑔𝑔〉 is continuous or differentiable on a region 𝑅𝑅 of ℝ2 if 𝑓𝑓 and 𝑔𝑔 are


continuous or differentiable on 𝑅𝑅, respectively.

Of particular importance are the radial vector fields,

𝒓𝒓 〈𝑥𝑥, 𝑦𝑦〉 𝒓𝒓 1
𝑭𝑭(𝑥𝑥, 𝑦𝑦) = 𝑘𝑘 = 𝑘𝑘 = 𝑘𝑘
|𝒓𝒓|𝑝𝑝 |𝒓𝒓|𝑝𝑝 |𝒓𝒓| |𝒓𝒓|𝑝𝑝−1
↑ ↑
unit vector magnitude

where 𝑝𝑝 is a real number and 𝑘𝑘 is a constant.

Examples of Vector Fields in 2d

A sampling of 2d vector fields denoted by 𝑭𝑭, and associated mathematical expressions, are shown
in Table 2. You are encouraged to follow the step-by-step procedure to confirm that you obtain
the same vector field patterns.

9
Table 2 Examples of a 2d vector field

Sketch of a 2d channel flow field Sketch of a 2d shear flow field

Sketch of a radial 2d field: 𝒑𝒑 = 𝟎𝟎, 𝒌𝒌 = 𝟐𝟐 Sketch of a rotational 2d field

10
1.3.2 Vector Fields in 3d

Definition: Vector Field in Three Dimensions

Let 𝑓𝑓, 𝑔𝑔 and ℎ be defined on a region 𝐷𝐷 of ℝ3 . A vector field in ℝ3 is a function 𝑭𝑭 that assigns
to each point in 𝐷𝐷 a vector 𝑭𝑭 = 〈𝑓𝑓(𝑥𝑥, 𝑦𝑦, 𝑧𝑧), 𝑔𝑔(𝑥𝑥, 𝑦𝑦, 𝑧𝑧), ℎ(𝑥𝑥, 𝑦𝑦, 𝑧𝑧)〉. The vector field is written as

𝑭𝑭(𝑥𝑥, 𝑦𝑦) = 〈𝑓𝑓(𝑥𝑥, 𝑦𝑦, 𝑧𝑧), 𝑔𝑔(𝑥𝑥, 𝑦𝑦, 𝑧𝑧), ℎ(𝑥𝑥, 𝑦𝑦, 𝑧𝑧)〉 or
𝑭𝑭(𝑥𝑥, 𝑦𝑦) = 𝑓𝑓 (𝑥𝑥, 𝑦𝑦, 𝑧𝑧)𝒊𝒊 + 𝑔𝑔(𝑥𝑥, 𝑦𝑦, 𝑧𝑧)𝒋𝒋 + ℎ(𝑥𝑥, 𝑦𝑦, 𝑧𝑧)𝒌𝒌

A vector field 𝑭𝑭 = 〈𝑓𝑓, 𝑔𝑔, ℎ〉 is continuous or differentiable on a region 𝐷𝐷 of ℝ3 if 𝑓𝑓, 𝑔𝑔 and


ℎ are continuous or differentiable on 𝐷𝐷, respectively

Of particular importance are the radial vector fields,

𝒓𝒓 〈𝑥𝑥, 𝑦𝑦, 𝑧𝑧〉 𝒓𝒓 1


𝑭𝑭(𝑥𝑥, 𝑦𝑦, 𝑧𝑧) = 𝑘𝑘 = 𝑘𝑘 = 𝑘𝑘
|𝒓𝒓| 𝑝𝑝 |𝒓𝒓| 𝑝𝑝 |𝒓𝒓| |𝒓𝒓|𝑝𝑝−1
↑ ↑
unit vector magnitude

where 𝑝𝑝 is a real number and 𝑘𝑘 is a constant.

Examples of Vector Fields in 3d

A sampling of 3d vector fields denoted by 𝑭𝑭, and associated mathematical expressions, are shown
in Table 3. You are encouraged to follow the step-by-step procedure to confirm that you obtain
the same vector field pattern.

Table 3 Examples of a 3d vector field

Sketch of fluid flow field in a cylindrical pipe

11
Sketch of an arbitrary vector field in 3d

1.3.3 Relationship Between Gradient Fields and Equipotential Curves


(2d)/Surfaces (3d)

In Chapter 15.1 we introduced the concept of level curves and level surfaces. In Chapter 15.5 we
showed that the gradient of a vector field is perpendicular to a level curve (2d: 𝑓𝑓 (𝑥𝑥, 𝑦𝑦) = 𝐾𝐾) and
a level surface (3d: 𝑓𝑓 (𝑥𝑥, 𝑦𝑦, 𝑧𝑧) = 𝐾𝐾), where 𝐾𝐾 is a chosen constant value. Level curves and level
surfaces are also referred to as equipotential curves and equipotential surfaces respectively. The
gradient field is obtained from a scalar potential, that is 𝑭𝑭 = ∇𝜙𝜙. A simple illustration showing the
relationship between the gradient of a scalar field and level curves is shown in Table 4 and the
level curves and gradient field for two practical 2d examples are shown in Table 5.

Table 4 General Illustration Showing the Relationship between the Level Curves and the
2d Gradient Field

Relationship between 2d gradient fields and


potential functions

12
Table 5 Examples of Equipotential Curves and the Gradient 2d Field

Sketch of isobars (lines of constant Sketch of isotherms (lines of constant


pressure: 𝝓𝝓(𝒙𝒙, 𝒚𝒚) = 𝐜𝐜𝐜𝐜𝐜𝐜𝐜𝐜𝐜𝐜𝐜𝐜𝐜𝐜𝐜𝐜) and 2d temperature T) and 2d temperature gradient
fluid flow field field: 𝑭𝑭 = 𝛻𝛻𝛻𝛻 = 〈−2𝑥𝑥, −2𝑦𝑦〉
𝑭𝑭 = 𝛻𝛻𝛻𝛻 = 〈𝑥𝑥, −𝑦𝑦〉

1.3.4 Example: Normal and tangent vectors

Let 𝐶𝐶 be the circle 𝑥𝑥 2 + 𝑦𝑦 2 = 𝑎𝑎2 , where 𝑎𝑎 > 0.


𝒓𝒓 〈𝑥𝑥,𝑦𝑦〉
a. Show that at each point of 𝐶𝐶, the radial vector field 𝑭𝑭(𝑥𝑥, 𝑦𝑦) = |𝒓𝒓| = is orthogonal to
�𝑥𝑥 2 +𝑦𝑦 2
the line tangent to 𝐶𝐶 at that point.
〈−𝑦𝑦,𝑥𝑥〉
b. Show that at each point of 𝐶𝐶, the rotation vector field 𝑮𝑮(𝑥𝑥, 𝑦𝑦) = is parallel to the
�𝑥𝑥 2 +𝑦𝑦 2
line tangent to 𝐶𝐶 at that point.

Solution

Let 𝑔𝑔(𝑥𝑥, 𝑦𝑦) = 𝑥𝑥 2 + 𝑦𝑦 2 . The circle 𝐶𝐶 described by the equation 𝑔𝑔(𝑥𝑥, 𝑦𝑦) = 𝑎𝑎2 may be viewed as a
level curve of the surface 𝑧𝑧 = 𝑥𝑥 2 + 𝑦𝑦 2 . As shown in Section 15.5, the gradient ∇𝑔𝑔(𝑥𝑥, 𝑦𝑦) =
〈2𝑥𝑥, 2𝑦𝑦〉, is orthogonal to the line tangent to 𝐶𝐶 at (𝑥𝑥, 𝑦𝑦), with reference to the figure on the next
page.

13
Relationship between a 2d radial and
rotational field on a given contour 𝑪𝑪 given
by 𝒛𝒛 = 𝒈𝒈(𝒙𝒙, 𝒚𝒚) = 𝒙𝒙𝟐𝟐 + 𝒚𝒚𝟐𝟐

𝒓𝒓
a. Notice that ∇𝑔𝑔(𝑥𝑥, 𝑦𝑦) is parallel to 𝑭𝑭(𝑥𝑥, 𝑦𝑦) = |𝒓𝒓| at the point (𝑥𝑥, 𝑦𝑦).It follows that 𝑭𝑭 is also
orthogonal to the line tangent to 𝐶𝐶 at (𝑥𝑥, 𝑦𝑦).
〈−𝑦𝑦,𝑥𝑥〉
b. Notice that ∇𝑔𝑔(𝑥𝑥, 𝑦𝑦) ∙ 𝑮𝑮(𝑥𝑥, 𝑦𝑦) = 〈2𝑥𝑥, 2𝑦𝑦〉 ∙ = 0. Therefore, ∇𝑔𝑔(𝑥𝑥, 𝑦𝑦) is orthogonal
�𝑥𝑥 2 +𝑦𝑦 2
to the vector field 𝑮𝑮(𝑥𝑥, 𝑦𝑦) at (𝑥𝑥, 𝑦𝑦), which implies that 𝑮𝑮(𝑥𝑥, 𝑦𝑦) is parallel to the tangent
line at (𝑥𝑥, 𝑦𝑦).

You can now work on the following problems from Section 17.1 before moving on:

1.3.5 Problems from Section 17.1: 23,29, 39, 53, 54, 55

1. (question 23) Sketch the following vector field: 𝑭𝑭 = 〈𝑦𝑦, −𝑥𝑥 〉


2. (question 29) Normal and tangential components For the vector field 𝑭𝑭 and the curve
𝐶𝐶, complete the following:
a. Determine the points (if any) along the curve 𝐶𝐶 at which the vector field 𝑭𝑭 is
tangent to to 𝐶𝐶
b. Determine the points (if any) along the curve 𝐶𝐶 at which the vector field 𝑭𝑭 is
normal to 𝐶𝐶
c. Sketch 𝐶𝐶 and a few representative vectors of 𝑭𝑭 on 𝐶𝐶.

𝑭𝑭 = 〈𝑥𝑥, 𝑦𝑦〉: 𝐶𝐶 = �(𝑥𝑥, 𝑦𝑦): 𝑥𝑥 = 1�

14
3. (question 39) Find the gradient field 𝑭𝑭 = ∇𝜙𝜙 for the following potential function:
𝑥𝑥 2 +𝑦𝑦 2 +𝑧𝑧2
𝜙𝜙(𝑥𝑥, 𝑦𝑦, 𝑧𝑧) = 2
4. (question 53) Explain why or why not Determine whether the following statements are
true and give an explanation or counterexample.
a. The vector field 𝑭𝑭 = 〈3𝑥𝑥 2 , 1〉 is a gradient field for both 𝜙𝜙1 (𝑥𝑥, 𝑦𝑦) = 𝑥𝑥 3 + 𝑦𝑦 and
𝜙𝜙2 (𝑥𝑥, 𝑦𝑦) = 𝑥𝑥 3 + 𝑦𝑦 + 100. Recall that the gradient is defined by 𝑭𝑭 = ∇𝜙𝜙.
〈𝑦𝑦,𝑥𝑥〉
b. The vector field 𝑭𝑭 = is constant in direction and magnitude on a unit
�𝑥𝑥 2 +𝑦𝑦2
circle.
〈𝑦𝑦,𝑥𝑥〉
c. The vector field 𝑭𝑭 = is neither a radial field nor a rotation field.
�𝑥𝑥 2 +𝑦𝑦2
5. (question 54) Electric field due to a point charge The electric field in the 𝑥𝑥𝑥𝑥 − plane
due to a point charge at (0,0) is a gradient field with a potential function 𝑉𝑉 (𝑥𝑥, 𝑦𝑦) =
𝑘𝑘
, where 𝑘𝑘 > 1 is a physical constant. The 2d unit radial vector pointing outwards
�𝑥𝑥 2 +𝑦𝑦 2
〈𝑥𝑥,𝑦𝑦〉
from the origin is defined as .
�𝑥𝑥 2 +𝑦𝑦 2
a. Find the components of the electric field in the 𝑥𝑥 − and 𝑦𝑦 − directions, where
𝑬𝑬 = −∇𝑉𝑉(𝑥𝑥, 𝑦𝑦).
b. Show that the vectors of the electric field point in a radial direction. (outward
𝑘𝑘
from the origin) and the radial component of 𝑬𝑬 can be expressed as 𝐸𝐸𝑟𝑟 = 𝑟𝑟 2,
where 𝑟𝑟 = �𝑥𝑥 2 + 𝑦𝑦 2 .
c. Show that the vector field is orthogonal to the equipotential (level) curves at all
point in the domain of 𝑉𝑉.
6. (question 55) Electric field due to a line of charge The electric field in the 𝑥𝑥𝑥𝑥 − plane
due to an infinite line of charge along the z- axis is a gradient field with a potential
𝑟𝑟𝑜𝑜
function 𝑉𝑉(𝑥𝑥, 𝑦𝑦) = 𝑐𝑐𝑐𝑐𝑐𝑐 � �, where 𝑐𝑐 > 0 is a constant and 𝑟𝑟𝑜𝑜 is a reference distance
�𝑥𝑥 +𝑦𝑦2
2

at which the potential is assumed to be 0 (see figure below).


a. Find the components of the electric field in the 𝑥𝑥 − and 𝑦𝑦 − directions, where
𝑬𝑬 = −∇𝑉𝑉(𝑥𝑥, 𝑦𝑦).
b. Show that the electric field at a point in in the 𝑥𝑥𝑥𝑥 − plane is directed outwards
𝑐𝑐
from the origin and has magnitude |𝐸𝐸| = , where 𝑟𝑟 = �𝑥𝑥 2 + 𝑦𝑦 2.
𝑟𝑟
c. Show that the vector field is orthogonal to the equipotential (level) curves at all
point in the domain of 𝑉𝑉.

15
You can find the solutions to the problems at the following link: Solutions to Reading Assignment
4 Problems.

1.4 Definitions

Define the following vectors:

• Unit vector tangent to an open or closed contour (𝐶𝐶) with the symbol 𝑻𝑻 that is either
directed right-to-left or left-to-right.
• Unit vector normal to a 2d open or closed contour (𝐶𝐶) with the symbol 𝒏𝒏𝒔𝒔 that is either
pointing to the right or to the left of the contour, as the contour is traversed. Note, the
subscript to 𝒏𝒏 is lower case 𝒔𝒔 to represent a contour.
• Unit vector normal to a 3d open or closed surface (𝑆𝑆) with the symbol 𝒏𝒏𝑺𝑺 that is either
pointing upward from the top side of the surface or downward from the bottom side of the
surface. A direction vector with a positive 𝑧𝑧̂ component is considered upward and a
direction vector with a negative 𝑧𝑧̂ component is considered downward. Notice that the
subscript to 𝒏𝒏 is upper case 𝑺𝑺 to represent a surface.

Note: The fact that there are options (CW/CCW or upward/downward) creates an ambiguity. We
will be introducing conventions to remove this ambiguity.

Consider the following question motivated by intuition: what type of operation involving a pair of
vectors, 𝑭𝑭 and 𝒃𝒃, leads to a scalar function which can then be integrated?

Answer: take the dot product of the two vectors: 𝑭𝑭 ∙ 𝒃𝒃.

The next question is what do the vectors 𝒃𝒃 and 𝑭𝑭 represent?

Vector 𝒃𝒃

The vector 𝒃𝒃 can have one of the following three forms depending on what is being considered:

16
• The differential arc length vector 𝑻𝑻𝑑𝑑𝑑𝑑 in the direction of the unit vector 𝑻𝑻 in 2d and 3d; 𝑑𝑑𝑑𝑑
represents the magnitude of the differential arc length.
• The differential arc length vector 𝒏𝒏𝒔𝒔 𝑑𝑑𝑑𝑑 in the direction of the unit vector 𝒏𝒏𝒔𝒔 in 2d. 𝑑𝑑𝑑𝑑
represents the differential arc length. The notion of a vector normal to a contour, having
attributes of a differential arc length, is somewhat counterintuitive. However, our definition
allows for it and it serves a useful purpose, when we consider it in relation to the vector 𝑭𝑭.
• The differential surface area vector 𝒏𝒏𝑺𝑺𝑑𝑑𝑑𝑑 in the direction of the unit vector 𝒏𝒏𝑺𝑺 in 3d; 𝑑𝑑𝑑𝑑
represents the magnitude of the differential surface area.

Note 1: Henceforth, the symbols 𝒏𝒏𝒔𝒔 and 𝒏𝒏𝑺𝑺 will frequently be replaced by 𝒏𝒏 to minimize the
number of symbols used. The context in which a problem, question or narrative is posed will
determine how the vector 𝒏𝒏 is to be interpreted.
Note 2: Throughout the notes and lectures, we sometimes use the symbol 𝒅𝒅𝒅𝒅 to represent a
differential arc length vector and use the symbol 𝒅𝒅𝒅𝒅 to represent a differential surface area vector.
The two respective cases can be expressed in various ways, as illustrated below:

����⃑
𝒅𝒅𝒅𝒅 = 𝑻𝑻𝑑𝑑𝑑𝑑 = 𝑑𝑑𝑑𝑑
����⃑
𝒅𝒅𝒅𝒅 = 𝒏𝒏𝑺𝑺𝑑𝑑𝑑𝑑 = 𝑑𝑑𝑑𝑑

Vector 𝑭𝑭

𝑭𝑭 can take on the following interpretations:

• A unit vector oriented in the direction of 𝒃𝒃. These are the cases to consider:

 𝑻𝑻 (a 2d unit vector parallel to a contour)


 𝑻𝑻 (a 3d unit vector parallel to an arbitrary contour or contour that lies on a surface)
 𝒏𝒏𝒔𝒔 (a 2d unit vector normal to a contour)
 𝒏𝒏𝑺𝑺 (a 3d unit vector normal to a surface in 3d)

• A scalar function with physical properties multiplied by a unit vector and oriented in the
direction of 𝒃𝒃. Examples could include a mass density or a charge density. These are the
cases to consider:

 𝑓𝑓(𝑥𝑥(𝑠𝑠), 𝑦𝑦(𝑠𝑠))𝑻𝑻 (a 2d vector parallel to a contour)


 𝑓𝑓(𝑥𝑥(𝑠𝑠), 𝑦𝑦(𝑠𝑠), 𝑧𝑧(𝑠𝑠))𝑻𝑻 (a 3d vector parallel to a contour)
 𝑓𝑓(𝑥𝑥(𝑠𝑠), 𝑦𝑦(𝑠𝑠))𝒏𝒏𝒔𝒔 (a 2d vector normal to a contour)
 𝑓𝑓(𝑥𝑥(𝑢𝑢, 𝑣𝑣), 𝑦𝑦(𝑢𝑢, 𝑣𝑣), 𝑧𝑧(𝑢𝑢, 𝑣𝑣))𝒏𝒏𝑺𝑺 (a 3d vector normal to a surface in 3d)

17
Note that 𝑓𝑓 is a function of a single parameterization variable 𝑠𝑠 when describing the
properties along a contour (tangent or normal for the 2d case) and a function of two
parameterization variables, 𝑢𝑢 and 𝑣𝑣, when describing the properties on a surface.

• A vector with physical properties.

 e.g., a gravitational field or electric field

1.4.1 Flux and Circulation and Their Units

The word flux means something that leaves or enters a 2d contour in a plane or leaves or enters
a 2d surface in a 3d domain. The word circulation indicates a measure of how much rotation
exists in a vector field. The word circulation, without a physical attribute associated with it, is a
form of metric representing the strength of circulation, but it has no absolute meaning. It is only a
starting point for discussions.

The definition for Circulation and Flux taken in the context of closed contours or closed surfaces
rather than open contours or open surfaces is intuitively easier to understand and has a geometrical
connection; a closed contour encloses an area, and a closed surface encloses a volume. On an
open contour or open surface, it is still possible to speak of a flux and circulation. Circulation and
flux are scalars.

Circulation and flux are associated with a physical attributes that have units. We will give the
physical attribute the symbol #. So, for example, a current flux, has units of Amperes (A), a
displacement flux has units of Coulombs (C), and a voltage circulation has units of volts (V). The
flux and circulation can be related back to the vector entities # flux density and # circulation
density respectively. However, some vectors, for example, current circulation density and
displacement circulation density have no physical importance. The laws of physics will dictate
which vector entities exist.

Table 6 shows all possible combinations of the integration of 𝑭𝑭 ∙ 𝒃𝒃 and the description or name we
give to each integral.

18
Table 6 Definitions of Integral Combinations with Three Representations of 𝑭𝑭

Description 𝑭𝑭: physical Description 𝑭𝑭: geometric

Circulation Length of open contour ∫𝐶𝐶 𝑭𝑭 ∙ 𝑻𝑻𝑑𝑑𝑑𝑑 𝑭𝑭 = 𝑻𝑻 → ∫𝐶𝐶 𝑑𝑑𝑑𝑑


�𝑭𝑭 ∙ 𝑻𝑻𝑑𝑑𝑑𝑑
Units of 𝑭𝑭: Circulation/m 𝐶𝐶
in 2d or 3d
Circulation Length of closed contour ∮𝐶𝐶 𝑭𝑭 ∙ 𝑻𝑻𝑑𝑑𝑑𝑑 𝑭𝑭 = 𝑻𝑻 → ∮𝐶𝐶 𝑑𝑑𝑑𝑑
�𝑭𝑭 ∙ 𝑻𝑻𝑑𝑑𝑑𝑑
Units of 𝑭𝑭: Circulation/m 𝐶𝐶 in 2d or 3d
Flux in a 2d plane Length of open contour ∫𝐶𝐶 𝑭𝑭 ∙ 𝒏𝒏𝒔𝒔 𝑑𝑑𝑑𝑑 𝑭𝑭 = 𝒏𝒏𝒔𝒔 → ∫𝐶𝐶 𝑑𝑑𝑑𝑑
�𝑭𝑭 ∙ 𝒏𝒏𝒔𝒔 𝑑𝑑𝑑𝑑
Units of 𝑭𝑭: Flux/m 𝐶𝐶 in 2d or 3d
Flux in a 2d plane Length of closed contour ∮𝐶𝐶 𝑭𝑭 ∙ 𝒏𝒏𝒔𝒔 𝑑𝑑𝑑𝑑 𝑭𝑭 = 𝒏𝒏𝒔𝒔 → ∮𝐶𝐶 𝑑𝑑𝑑𝑑
�𝑭𝑭 ∙ 𝒏𝒏𝒔𝒔 𝑑𝑑𝑑𝑑
Units of 𝑭𝑭: Flux/m 𝐶𝐶
in 2d or 3d
Flux Surface area of open ∬𝑆𝑆 𝑭𝑭 ∙ 𝒏𝒏𝑺𝑺 𝑑𝑑𝑑𝑑 𝑭𝑭 = 𝒏𝒏𝑺𝑺 → ∬𝑆𝑆 𝑑𝑑𝑑𝑑
�𝑭𝑭 ∙ 𝒏𝒏𝑺𝑺 𝑑𝑑𝑑𝑑
Units of 𝑭𝑭: Flux/m2 𝑆𝑆
surface
Flux Surface area of closed ∯𝑆𝑆 𝑭𝑭 ∙ 𝒏𝒏𝑺𝑺𝑑𝑑𝑑𝑑 𝑭𝑭 = 𝒏𝒏𝑺𝑺 → ∯𝑆𝑆 𝑑𝑑𝑑𝑑
�𝑭𝑭 ∙ 𝒏𝒏𝑺𝑺 𝑑𝑑𝑑𝑑
Units of 𝑭𝑭: Flux/m2 𝑆𝑆
surface

19
Description 𝑭𝑭: physical Description 𝑭𝑭: geometric

Property # of open ∫𝐶𝐶 𝑭𝑭 ∙ 𝑻𝑻𝑑𝑑𝑑𝑑 𝑭𝑭 = 𝜆𝜆(𝑥𝑥 (𝑠𝑠), 𝑦𝑦(𝑠𝑠))𝑻𝑻 → ∫𝐶𝐶 𝜆𝜆(𝑥𝑥(𝑠𝑠), 𝑦𝑦(𝑠𝑠))𝑑𝑑𝑑𝑑 Area of curved
�𝑭𝑭 ∙ 𝑻𝑻𝑑𝑑𝑑𝑑
contour in 2d or 3d ∫𝐶𝐶 𝑭𝑭 ∙ 𝑻𝑻𝑑𝑑𝑑𝑑 𝑭𝑭 = 𝜆𝜆(𝑥𝑥 (𝑠𝑠), 𝑦𝑦(𝑠𝑠), 𝑧𝑧(𝑠𝑠))𝑻𝑻 → surface with an 𝐶𝐶
open contour 𝑭𝑭 = 𝑓𝑓(𝑥𝑥(𝑠𝑠), 𝑦𝑦(𝑠𝑠))𝑻𝑻
∫𝐶𝐶 𝜆𝜆(𝑥𝑥(𝑠𝑠), 𝑦𝑦(𝑠𝑠), 𝑧𝑧(𝑠𝑠))𝑑𝑑𝑑𝑑
# represents entity base lying in the
such as charge (𝐶𝐶) 𝑥𝑥𝑥𝑥 plane → �𝑓𝑓(𝑥𝑥(𝑠𝑠), 𝑦𝑦(𝑠𝑠))𝑑𝑑𝑑𝑑
units of 𝜆𝜆: #/𝑚𝑚 𝐶𝐶
units of 𝑓𝑓: 𝑚𝑚
Property # of closed ∮𝐶𝐶 𝑭𝑭 ∙ 𝑻𝑻𝑑𝑑𝑑𝑑 𝑭𝑭 = 𝜆𝜆(𝑥𝑥 (𝑠𝑠), 𝑦𝑦(𝑠𝑠))𝑻𝑻 → ∮𝐶𝐶 𝜆𝜆(𝑥𝑥(𝑠𝑠), 𝑦𝑦(𝑠𝑠))𝑑𝑑𝑑𝑑 Area of curved
�𝑭𝑭 ∙ 𝑻𝑻𝑑𝑑𝑑𝑑
contour in 2d or 3d ∮𝐶𝐶 𝑭𝑭 ∙ 𝑻𝑻𝑑𝑑𝑑𝑑 𝑭𝑭 = 𝜆𝜆(𝑥𝑥 (𝑠𝑠), 𝑦𝑦(𝑠𝑠), 𝑧𝑧(𝑠𝑠))𝑻𝑻 → surface with a 𝐶𝐶
closed contour 𝑭𝑭 = 𝑓𝑓(𝑥𝑥(𝑠𝑠), 𝑦𝑦(𝑠𝑠))𝑻𝑻
∮𝐶𝐶 𝜆𝜆(𝑥𝑥(𝑠𝑠), 𝑦𝑦(𝑠𝑠), 𝑧𝑧(𝑠𝑠))𝑑𝑑𝑑𝑑
# represents entity base lying in the
such as charge (𝐶𝐶) 𝑥𝑥𝑥𝑥 plane → �𝑓𝑓(𝑥𝑥(𝑠𝑠), 𝑦𝑦(𝑠𝑠))𝑑𝑑𝑑𝑑
units of 𝜆𝜆: #/𝑚𝑚 𝐶𝐶
units of 𝑓𝑓: 𝑚𝑚
Property # of open ∫𝐶𝐶 𝑭𝑭 ∙ 𝒏𝒏𝒔𝒔 𝑑𝑑𝑑𝑑 𝑭𝑭 = 𝜆𝜆(𝑥𝑥 (𝑠𝑠), 𝑦𝑦(𝑠𝑠))𝒏𝒏𝒔𝒔 → ∫𝐶𝐶 𝜆𝜆(𝑥𝑥(𝑠𝑠), 𝑦𝑦(𝑠𝑠))𝑑𝑑𝑑𝑑 Area of curved
�𝑭𝑭 ∙ 𝒏𝒏𝒔𝒔 𝑑𝑑𝑑𝑑
contour in 2d or 3d ∫𝐶𝐶 𝑭𝑭 ∙ 𝒏𝒏𝒔𝒔 𝑑𝑑𝑑𝑑 𝑭𝑭 = 𝜆𝜆(𝑥𝑥 (𝑠𝑠), 𝑦𝑦(𝑠𝑠), 𝑧𝑧(𝑠𝑠))𝒏𝒏𝒔𝒔 → surface with an 𝐶𝐶
open contour 𝑭𝑭 = 𝑓𝑓(𝑥𝑥(𝑠𝑠), 𝑦𝑦(𝑠𝑠))𝒏𝒏𝒔𝒔
∫𝐶𝐶 𝜆𝜆(𝑥𝑥(𝑠𝑠), 𝑦𝑦(𝑠𝑠), 𝑧𝑧(𝑠𝑠))𝑑𝑑𝑑𝑑
# represents entity base lying in the
such as charge (𝐶𝐶) 𝑥𝑥𝑥𝑥 plane → �𝑓𝑓(𝑥𝑥(𝑠𝑠), 𝑦𝑦(𝑠𝑠))𝑑𝑑𝑑𝑑
units of 𝜆𝜆: #/𝑚𝑚 𝐶𝐶
units of 𝑓𝑓: 𝑚𝑚
Property # of closed ∮𝐶𝐶 𝑭𝑭 ∙ 𝒏𝒏𝒔𝒔 𝑑𝑑𝑑𝑑 𝑭𝑭 = 𝜆𝜆(𝑥𝑥 (𝑠𝑠), 𝑦𝑦(𝑠𝑠))𝒏𝒏𝒔𝒔 → ∮𝐶𝐶 𝜆𝜆(𝑥𝑥(𝑠𝑠), 𝑦𝑦(𝑠𝑠))𝑑𝑑𝑑𝑑 Area of curved
�𝑭𝑭 ∙ 𝒏𝒏𝒔𝒔 𝑑𝑑𝑑𝑑
contour in 2d or 3d ∮𝐶𝐶 𝑭𝑭 ∙ 𝒏𝒏𝒔𝒔 𝑑𝑑𝑑𝑑 𝑭𝑭 = 𝜆𝜆(𝑥𝑥 (𝑠𝑠), 𝑦𝑦(𝑠𝑠), 𝑧𝑧(𝑠𝑠))𝒏𝒏𝒔𝒔 → surface with a 𝐶𝐶
closed contour 𝑭𝑭 = 𝑓𝑓(𝑥𝑥(𝑠𝑠), 𝑦𝑦(𝑠𝑠))𝒏𝒏𝒔𝒔
∮𝐶𝐶 𝜆𝜆(𝑥𝑥(𝑠𝑠), 𝑦𝑦(𝑠𝑠), 𝑧𝑧(𝑠𝑠))𝑑𝑑𝑑𝑑
# represents entity base lying in the
such as charge (𝐶𝐶) 𝑥𝑥𝑥𝑥 plane → �𝑓𝑓(𝑥𝑥(𝑠𝑠), 𝑦𝑦(𝑠𝑠))𝑑𝑑𝑑𝑑
units of 𝜆𝜆: #/𝑚𝑚 𝐶𝐶
units of 𝑓𝑓: 𝑚𝑚

20
Description 𝑭𝑭: physical

Property # over an open surface area


�𝑭𝑭 ∙ 𝒏𝒏𝑺𝑺𝑑𝑑𝑑𝑑
𝑆𝑆
# represents entity such as mass (𝑘𝑘𝑘𝑘) 𝑭𝑭 = 𝜎𝜎(𝑥𝑥(𝑢𝑢, 𝑣𝑣 ), 𝑦𝑦(𝑢𝑢, 𝑣𝑣 ), 𝑧𝑧(𝑢𝑢, 𝑣𝑣))𝒏𝒏𝑺𝑺
→ �𝜎𝜎(𝑥𝑥 (𝑢𝑢, 𝑣𝑣 ), 𝑦𝑦(𝑢𝑢, 𝑣𝑣 ), 𝑧𝑧(𝑢𝑢, 𝑣𝑣))𝑑𝑑𝑑𝑑
𝑆𝑆
units of 𝜎𝜎: #/𝑚𝑚2
Property # over a closed surface area
�𝑭𝑭 ∙ 𝒏𝒏𝑺𝑺𝑑𝑑𝑑𝑑
𝑆𝑆
# represents entity such as mass (𝑘𝑘𝑘𝑘) 𝑭𝑭 = 𝜎𝜎(𝑥𝑥(𝑢𝑢, 𝑣𝑣 ), 𝑦𝑦(𝑢𝑢, 𝑣𝑣 ), 𝑧𝑧(𝑢𝑢, 𝑣𝑣))𝒏𝒏𝑺𝑺
→ �𝜎𝜎(𝑥𝑥 (𝑢𝑢, 𝑣𝑣 ), 𝑦𝑦(𝑢𝑢, 𝑣𝑣 ), 𝑧𝑧(𝑢𝑢, 𝑣𝑣))𝑑𝑑𝑑𝑑
𝑆𝑆
units of 𝜎𝜎: #/𝑚𝑚2
Units of integral (# ∙ 𝑚𝑚) involve the ∫𝐶𝐶 𝑭𝑭 ∙ 𝑻𝑻𝑑𝑑𝑑𝑑 𝑭𝑭 = 𝑓𝑓(𝑥𝑥(𝑠𝑠), 𝑦𝑦(𝑠𝑠))𝑻𝑻 → ∫𝐶𝐶 𝑓𝑓(𝑥𝑥(𝑠𝑠), 𝑦𝑦(𝑠𝑠))𝑑𝑑𝑑𝑑 in 2d
entity of interest referred to as 𝑓𝑓 with ∮𝐶𝐶 𝑭𝑭 ∙ 𝑻𝑻𝑑𝑑𝑑𝑑 𝑭𝑭 = 𝑓𝑓(𝑥𝑥(𝑠𝑠), 𝑦𝑦(𝑠𝑠))𝑻𝑻 → ∮𝐶𝐶 𝑓𝑓(𝑥𝑥(𝑠𝑠), 𝑦𝑦(𝑠𝑠))𝑑𝑑𝑑𝑑 in 2d
specific units #, integrated over a
∫𝐶𝐶 𝑭𝑭 ∙ 𝑻𝑻𝑑𝑑𝑑𝑑 𝑭𝑭 = 𝑓𝑓(𝑥𝑥(𝑠𝑠), 𝑦𝑦(𝑠𝑠), 𝑧𝑧(𝑠𝑠))𝑻𝑻 → ∫𝐶𝐶 𝑓𝑓(𝑥𝑥 (𝑠𝑠), 𝑦𝑦(𝑠𝑠), 𝑧𝑧(𝑠𝑠))𝑑𝑑𝑑𝑑 in 3d
length. This integral is then divided by
the length to compute 𝑓𝑓𝑎𝑎𝑎𝑎𝑎𝑎 ∮𝐶𝐶 𝑭𝑭 ∙ 𝑻𝑻𝑑𝑑𝑑𝑑 𝑭𝑭 = 𝑓𝑓(𝑥𝑥(𝑠𝑠), 𝑦𝑦(𝑠𝑠), 𝑧𝑧(𝑠𝑠))𝑻𝑻 → ∮𝐶𝐶 𝑓𝑓(𝑥𝑥(𝑠𝑠), 𝑦𝑦(𝑠𝑠), 𝑧𝑧(𝑠𝑠))𝑑𝑑𝑑𝑑 in 3d

units of 𝑓𝑓: #
Units of integral (# ∙ 𝑚𝑚2 ) involve the ∬𝑆𝑆 𝑭𝑭 ∙ 𝒏𝒏𝑺𝑺 𝑑𝑑𝑑𝑑 𝑭𝑭 = 𝑓𝑓(𝑥𝑥(𝑢𝑢, 𝑣𝑣 ), 𝑦𝑦(𝑢𝑢, 𝑣𝑣 ), 𝑧𝑧(𝑢𝑢, 𝑣𝑣))𝒏𝒏𝑺𝑺 → ∬𝑆𝑆 𝑓𝑓(𝑥𝑥(𝑢𝑢, 𝑣𝑣 ), 𝑦𝑦(𝑢𝑢, 𝑣𝑣 ), 𝑧𝑧(𝑢𝑢, 𝑣𝑣))𝑑𝑑𝑑𝑑
entity of interest referred to as 𝑓𝑓 with ∯𝑆𝑆 𝑭𝑭 ∙ 𝒏𝒏𝑺𝑺𝑑𝑑𝑑𝑑 𝑭𝑭 = 𝑓𝑓(𝑥𝑥 (𝑢𝑢, 𝑣𝑣 ), 𝑦𝑦(𝑢𝑢, 𝑣𝑣 ), 𝑧𝑧(𝑢𝑢, 𝑣𝑣))𝒏𝒏𝑺𝑺 → ∯𝑆𝑆 𝑓𝑓(𝑥𝑥(𝑢𝑢, 𝑣𝑣 ), 𝑦𝑦(𝑢𝑢, 𝑣𝑣 ), 𝑧𝑧(𝑢𝑢, 𝑣𝑣))𝑑𝑑𝑑𝑑
specific units #, integrated over an
area. This integral is then divided by units of 𝑓𝑓: #
the area to compute 𝑓𝑓𝑎𝑎𝑎𝑎𝑎𝑎

21
1.4.2 Scalar Densities and Their Units

If we remove the word flux and circulation from # flux density and # circulation density
respectively we obtain an ordinary density (# density), which is a scalar. In some of the literature
or in books the use of nomenclature is a bit sloppy. For example, the word current density is used
frequently, which is incorrect. To be accurate, it is a current flux density which is a vector.
Generally, the units will implicitly tell you if the entity is a vector or a scalar.

Scalar Density

If the word charge density or mass density are used, then charge (units in 𝐶𝐶) or mass (units in 𝑘𝑘𝑘𝑘)
have physical meaning. Density without the prefix flux or circulation can have units of # ∙ 𝑚𝑚−1 ,
# ∙ 𝑚𝑚−2 or # ∙ 𝑚𝑚−3 depending on whether the # density is # per unit length, # per unit area or #
per unit volume.

Example: With reference to Table 6, consider a physical property such as a charge. The integral
∭𝑉𝑉 𝜌𝜌(𝑥𝑥, 𝑦𝑦, 𝑧𝑧)𝑑𝑑𝑑𝑑 , ∬𝐴𝐴 𝜎𝜎𝜎𝜎𝜎𝜎𝜎𝜎𝜎𝜎 and ∫𝐿𝐿 𝜆𝜆𝜆𝜆𝜆𝜆 represent the total charge, in the volume, on the surface
and along the line respectively. We will use the symbol 𝜆𝜆 (units 𝐶𝐶/𝑚𝑚), for line charges, and 𝜎𝜎
(units 𝐶𝐶/𝑚𝑚2 ), for surface charges, to represent the charge per unit length and charge per unit area
respectively.

Computing the area of a curved surface

With reference to Table 6, the integrals ∫𝐶𝐶 𝑓𝑓(𝑥𝑥(𝑠𝑠), 𝑦𝑦(𝑠𝑠))𝑑𝑑𝑑𝑑 and ∮𝐶𝐶 𝑓𝑓(𝑥𝑥(𝑠𝑠), 𝑦𝑦(𝑠𝑠))𝑑𝑑𝑑𝑑 represent the
area of a pleated surface where the height of the pleated surface, a scalar, is defined by
𝑓𝑓(𝑥𝑥(𝑠𝑠), 𝑦𝑦(𝑠𝑠)) and has units of metres. The shape of the pleat in the transverse direction is defined
by the contour traversed in the 𝑥𝑥𝑥𝑥 plane.

Computing an average of a scalar function

With reference to Table 6, it is also possible to integrate a scalar entity, such as a temperature, over
a region (line, area, volume) and then divide by the size of the region. For example, the average of
𝑓𝑓, designated as 𝑓𝑓𝑎𝑎𝑎𝑎𝑎𝑎 , is defined by the following expressions for a closed contour, a closed surface,
and a volume respectively. Similar expressions can be written for an open contour 𝐶𝐶 and an open
surface 𝑆𝑆.

∮𝐶𝐶 𝑓𝑓𝑓𝑓𝑓𝑓 ∯𝑆𝑆 𝑓𝑓𝑓𝑓𝑓𝑓 ∭𝑉𝑉 𝑓𝑓𝑓𝑓𝑓𝑓


𝑓𝑓𝑎𝑎𝑎𝑎𝑎𝑎 = 𝑓𝑓𝑎𝑎𝑎𝑎𝑎𝑎 = 𝑓𝑓𝑎𝑎𝑎𝑎𝑎𝑎 =
∮𝐶𝐶 𝑑𝑑𝑑𝑑 ∯𝑆𝑆 𝑑𝑑𝑑𝑑 ∭𝑉𝑉 𝑑𝑑𝑑𝑑

22
Lastly, we can also have expressions which are a hybrid of the above for example:

∮𝐶𝐶 𝑥𝑥𝑥𝑥𝑥𝑥𝑥𝑥 ∯𝑆𝑆 𝑥𝑥𝑥𝑥𝑥𝑥𝑥𝑥 ∭𝑉𝑉 𝑥𝑥𝑥𝑥𝑥𝑥𝑥𝑥


𝑥𝑥𝑎𝑎𝑎𝑎𝑎𝑎 = 𝑥𝑥𝑎𝑎𝑎𝑎𝑎𝑎 = 𝑥𝑥𝑎𝑎𝑎𝑎𝑎𝑎 =
∮𝐶𝐶 𝜆𝜆𝜆𝜆𝜆𝜆 ∯𝑆𝑆 𝜎𝜎𝜎𝜎𝜎𝜎 ∭𝑉𝑉 𝜌𝜌𝜌𝜌𝜌𝜌

These expressions, for example, would give the centre of mass in the 𝑥𝑥 direction, for a closed line
𝑘𝑘𝑘𝑘
segment, a closed surface area and a volume. 𝜆𝜆, 𝜎𝜎 and 𝜌𝜌 represent the mass density in units of ,
𝑚𝑚
𝑘𝑘𝑘𝑘 𝑘𝑘𝑘𝑘
and respectively. Similar expressions can be reconstructed to represent the centre of mass
𝑚𝑚2 𝑚𝑚3
for the 𝑦𝑦 and 𝑧𝑧 directions: the symbol 𝑥𝑥 would be replaced by 𝑦𝑦 and 𝑧𝑧 respectively. Note, the first
two expressions could also be interpreted in the context of 2d problems in which case the centre
of mass with respect to the 𝑧𝑧 direction would not exist. Example problems where you need to
compute an average quantity are given in section 1.5 and 1.6.

1.5 Derivation of Contour Integral from a Riemann Sum and Examples

We will now define what is meant by a contour integral derived from a Riemann sum and consider
some examples in parallel.

Resultant of integrating a scalar density with geometrical meaning along a curved line:

Area swept out: A scalar density 𝑓𝑓(𝑥𝑥(𝑠𝑠), 𝑦𝑦(𝑠𝑠)) in 2d can be the height in the 𝑧𝑧̂ direction and is as
a function of distance 𝑠𝑠 along the arc length or contour 𝐶𝐶. In this case, the integral
∫𝐶𝐶 𝑓𝑓(𝑥𝑥(𝑠𝑠), 𝑦𝑦(𝑠𝑠))𝑑𝑑𝑑𝑑 or ∮𝐶𝐶 𝑓𝑓(𝑥𝑥(𝑠𝑠), 𝑦𝑦(𝑠𝑠))𝑑𝑑𝑑𝑑, represents the vertical area swept out as you follow an
arbitrary contour 𝐶𝐶 traversed in the 𝑥𝑥𝑥𝑥 plane. You can visualize this as a pleated curtain where the
height of the curtain can vary and a path along the base of the curtain is taken to be the contour
𝐶𝐶 that is followed. We will now show how this type of integral is formulated as a Riemann Sum
with reference to Fig. 1.

Assume that 𝐶𝐶 is a smooth curve of finite length, parameterized in terms of arc length as 𝑟𝑟⃑(𝑠𝑠) =
〈𝑥𝑥(𝑠𝑠), 𝑦𝑦(𝑠𝑠)〉 for 𝑎𝑎 ≤ 𝑠𝑠 ≤ 𝑏𝑏, and let 𝑓𝑓 be defined on 𝐶𝐶. The parameter 𝑠𝑠 resides on the 𝑠𝑠 −axis.
As 𝑠𝑠 varies from 𝑎𝑎 to 𝑏𝑏 on the 𝑠𝑠 −axis, the curve 𝐶𝐶 in the 𝑥𝑥𝑥𝑥 −plane is generated from the point
(𝑥𝑥(𝑎𝑎), 𝑦𝑦(𝑎𝑎)) to the point (𝑥𝑥(𝑏𝑏), 𝑦𝑦(𝑏𝑏)).

We subdivide 𝐶𝐶 into 𝑛𝑛 small arcs by forming a partition of [𝑎𝑎, 𝑏𝑏]:

𝑎𝑎 = 𝑠𝑠0 < 𝑠𝑠1 < ⋯ < 𝑠𝑠𝑛𝑛−1 < 𝑠𝑠𝑛𝑛 = 𝑏𝑏

Let 𝑠𝑠𝑘𝑘∗ be a point in the 𝑘𝑘𝑘𝑘ℎ subinterval [𝑠𝑠𝑘𝑘−1 , 𝑠𝑠𝑘𝑘 ], which corresponds to a point (𝑥𝑥 (𝑠𝑠𝑘𝑘∗ ), 𝑦𝑦(𝑠𝑠𝑘𝑘∗ )) on
the 𝑘𝑘𝑘𝑘ℎ arc of 𝐶𝐶, for 𝑘𝑘 = 1, 2, . . , 𝑛𝑛. The length of the 𝑘𝑘𝑘𝑘ℎ arc is denoted by ∆𝑠𝑠𝑘𝑘 . This partition

23
also divides the curtain into 𝑛𝑛 panels. The 𝑘𝑘𝑘𝑘ℎ panel has an approximate height of 𝑓𝑓(𝑥𝑥(𝑠𝑠𝑘𝑘∗ ), 𝑦𝑦(𝑠𝑠𝑘𝑘∗ ))
and a base of length ∆𝑠𝑠𝑘𝑘 ; therefore, the approximate area of the 𝑘𝑘𝑘𝑘ℎ panel is 𝑓𝑓(𝑥𝑥(𝑠𝑠𝑘𝑘∗ ), 𝑦𝑦(𝑠𝑠𝑘𝑘∗ ))∆𝑠𝑠𝑘𝑘
(Figure 1). Summing the areas of the panels, the approximate area of the curtain is given by the
Riemann sum.

𝑛𝑛

area ≅ � 𝑓𝑓(𝑥𝑥 (𝑠𝑠𝑘𝑘∗ ), 𝑦𝑦(𝑠𝑠𝑘𝑘∗ ))∆𝑠𝑠𝑘𝑘


𝑘𝑘=1

Fig. 1 Illustration of the Riemann sum to compute the area of a curtain

We now let ∆ be the maximum value of ∆𝑠𝑠1 , … , ∆𝑠𝑠𝑛𝑛 . If the limit of the Riemann sum as 𝑛𝑛 → ∞
and ∆→ 0 exists, over all partitions along the contour 𝐶𝐶, the limit is called a line integral, and it
gives the area of the curtain.

24
Definition: Scalar Line Integral in the Plane, Arc Length Parameter

Suppose the scalar-valued function 𝑓𝑓 is defined on the smooth curve 𝐶𝐶: 𝑟𝑟(𝑠𝑠) = 〈𝑥𝑥 (𝑠𝑠), 𝑦𝑦(𝑠𝑠)〉,
parameterized by the arc length 𝑠𝑠. The line integral of 𝒇𝒇 over 𝑪𝑪 is
𝑛𝑛

�𝑓𝑓(𝑥𝑥(𝑠𝑠), 𝑦𝑦(𝑠𝑠))𝑑𝑑𝑑𝑑 = lim � 𝑓𝑓(𝑥𝑥(𝑠𝑠𝑘𝑘∗ ), 𝑦𝑦(𝑠𝑠𝑘𝑘∗ ))∆𝑠𝑠𝑘𝑘


∆→0
𝐶𝐶 𝑛𝑛→∞ 𝑘𝑘=1

provided this limit exists over all possible partitions of 𝐶𝐶. When the limit exists, 𝑓𝑓 is said to be
integrable on 𝐶𝐶. Generally, this type of calculation is difficult except for the simplest of cases
therefore two other formulations are applied.

Integrating a scalar density with a physical meaning along a curved open or closed contour

Physical attribute: 𝑓𝑓 (𝑥𝑥(𝑠𝑠), 𝑦𝑦(𝑠𝑠)) = 𝜆𝜆(𝑥𝑥(𝑠𝑠), 𝑦𝑦(𝑠𝑠)) in 2d or 𝑓𝑓 (𝑥𝑥, 𝑦𝑦, 𝑧𝑧) = 𝜆𝜆(𝑥𝑥(𝑠𝑠), 𝑦𝑦(𝑠𝑠), 𝑧𝑧(𝑠𝑠)) in 3d
have a physical meaning such as charge per unit length 𝐶𝐶/𝑚𝑚. Then the integral for
∫𝐶𝐶 𝜆𝜆(𝑥𝑥(𝑠𝑠), 𝑦𝑦(𝑠𝑠))𝑑𝑑𝑑𝑑 or ∮𝐶𝐶 𝜆𝜆(𝑥𝑥(𝑠𝑠), 𝑦𝑦(𝑠𝑠))𝑑𝑑𝑑𝑑, in 2d, or ∫𝐶𝐶 𝜆𝜆(𝑥𝑥(𝑠𝑠), 𝑦𝑦(𝑠𝑠), 𝑧𝑧(𝑠𝑠))𝑑𝑑𝑑𝑑 or
∮𝐶𝐶 𝜆𝜆(𝑥𝑥(𝑠𝑠), 𝑦𝑦(𝑠𝑠), 𝑧𝑧(𝑠𝑠))𝑑𝑑𝑑𝑑, in 3d, represents the total charge on the line defined by the contour.

Explicit form: express points along 𝑠𝑠 in terms of the variable 𝑥𝑥 in which case the expressions
become ∫𝐶𝐶 𝜆𝜆(𝑥𝑥, 𝑦𝑦(𝑥𝑥))𝑔𝑔(𝑥𝑥)𝑑𝑑𝑑𝑑 , in 2d, where 𝑑𝑑𝑑𝑑 is expressed in terms of a function of 𝑥𝑥, that is
𝑔𝑔(𝑥𝑥)𝑑𝑑𝑑𝑑. For a three-dimensional line, ∫𝐶𝐶 𝜆𝜆(𝑥𝑥, 𝑦𝑦(𝑥𝑥), 𝑧𝑧(𝑥𝑥))𝑔𝑔(𝑥𝑥)𝑑𝑑𝑑𝑑, where as before, 𝑑𝑑𝑑𝑑 is expressed
in terms of a function of 𝑥𝑥, that is 𝑔𝑔(𝑥𝑥)𝑑𝑑𝑑𝑑.

Parameterized form: express points along 𝑠𝑠 in terms of the variable 𝑡𝑡 in which case the
expressions become ∫𝐶𝐶 𝜆𝜆(𝑥𝑥 (𝑡𝑡), 𝑦𝑦(𝑡𝑡))𝑔𝑔(𝑡𝑡)𝑑𝑑𝑑𝑑 , in 2d, where 𝑑𝑑𝑑𝑑 is expressed in terms of a function of
𝑡𝑡, that is 𝑔𝑔(𝑡𝑡)𝑑𝑑𝑑𝑑. For a three-dimensional line, ∫𝐶𝐶 𝜆𝜆(𝑥𝑥(𝑡𝑡), 𝑦𝑦(𝑡𝑡), 𝑧𝑧(𝑡𝑡))𝑔𝑔(𝑡𝑡)𝑑𝑑𝑑𝑑, where as before, 𝑑𝑑𝑑𝑑
is expressed in terms of a function of 𝑡𝑡, that is 𝑔𝑔(𝑡𝑡)𝑑𝑑𝑑𝑑.

The mathematical machinery required to calculate line or contour (𝐶𝐶) integrals such as
∫𝐶𝐶 𝜆𝜆(𝑥𝑥(𝑠𝑠), 𝑦𝑦(𝑠𝑠))𝑑𝑑𝑑𝑑, ∫𝐶𝐶 𝜆𝜆(𝑥𝑥(𝑠𝑠), 𝑦𝑦(𝑠𝑠), 𝑧𝑧(𝑠𝑠))𝑑𝑑𝑑𝑑, ∫𝐶𝐶 𝑭𝑭 ∙ 𝑻𝑻𝑑𝑑𝑑𝑑 and ∫𝐶𝐶 𝑭𝑭 ∙ 𝒏𝒏𝒔𝒔 𝑑𝑑𝑑𝑑 is described next. The
integrals can be either over open or closed contours. These types of calculations are important later
in the course when we look at applications that relate to the physical world.

����⃑ and 𝑑𝑑𝑑𝑑 and provide some


We summarize, for your convenience, the main equations for 𝑻𝑻, 𝒏𝒏𝒔𝒔 , 𝑑𝑑𝑑𝑑
worked-out examples.

𝐂𝐂𝐂𝐂𝐂𝐂𝐂𝐂𝐂𝐂𝐂𝐂𝐂𝐂𝐂𝐂𝐂𝐂 𝑻𝑻, 𝒏𝒏𝒔𝒔 , ����⃑


𝒅𝒅𝒅𝒅 𝐚𝐚𝐚𝐚𝐚𝐚 𝒅𝒅𝒅𝒅

25
Earlier, we described the formulation of the Riemann sum, but we chose to work with the length
of an arc 𝑑𝑑𝑑𝑑. Unfortunately, for many shapes we can not work directly with the notion of an arc
length and so we start with a parametrized representation of the contour or an explicit
representation of the contour. The procedure to obtain the quantities of interest is summarized
next:

Step 1: Determine the position vector 𝑟𝑟⃑ that traces out the contour (open or closed) and the bounds
on 𝑡𝑡 (for parameterized representation) and 𝑥𝑥 (for explicit representation)

2d 3d
Parameterized 𝑟𝑟⃑ = 〈𝑥𝑥(𝑡𝑡), 𝑦𝑦(𝑡𝑡)〉 𝑟𝑟⃑ = 〈𝑥𝑥 (𝑡𝑡), 𝑦𝑦(𝑡𝑡), 𝑧𝑧(𝑡𝑡)〉
𝑟𝑟⃑ = 〈𝑥𝑥, 𝑦𝑦(𝑥𝑥)〉 𝑟𝑟⃑ = 〈𝑥𝑥, 𝑦𝑦(𝑥𝑥 ), 𝑧𝑧(𝑥𝑥)〉
Explicit

𝑟𝑟⃑̇
Step 2: Compute the unit tangent vector 𝑻𝑻 = �𝑟𝑟⃑̇�. A pictorial representation of how this vector is
derived, assuming a parameterized representation, is shown in Fig. 2.

𝑟𝑟̇⃑ = vector tangent to the contour

2d 3d
Parameterized 𝑟𝑟⃑̇ = 〈𝑥𝑥̇ (𝑡𝑡), 𝑦𝑦̇ (𝑡𝑡)〉 𝑟𝑟⃑̇ = 〈𝑥𝑥̇ (𝑡𝑡), 𝑦𝑦̇ (𝑡𝑡), 𝑧𝑧̇ (𝑡𝑡)〉
𝑑𝑑𝑑𝑑 𝑑𝑑𝑑𝑑 𝑑𝑑𝑑𝑑
Explicit 𝑟𝑟⃑̇ = 〈1, 〉 𝑟𝑟⃑̇ = 〈1, , 〉
𝑑𝑑𝑑𝑑 𝑑𝑑𝑑𝑑 𝑑𝑑𝑑𝑑

𝑟𝑟⃑̇
The unit tangent vector 𝑻𝑻 = �𝑟𝑟⃑̇� is thus:

2d 3d
𝑟𝑟⃑̇ 〈𝑥𝑥̇ (𝑡𝑡),𝑦𝑦̇ (𝑡𝑡)〉 〈𝑥𝑥̇ (𝑡𝑡),𝑦𝑦̇ (𝑡𝑡),𝑧𝑧̇ (𝑡𝑡)〉
𝑻𝑻 = �𝑟𝑟⃑̇� = where 𝑻𝑻 = ̇
�𝑟𝑟⃑̇� �𝑟𝑟⃑�
Parameterized 2 2 where �𝑟𝑟⃑̇� =
�𝑟𝑟⃑̇� = ��𝑥𝑥̇ (𝑡𝑡)� + �𝑦𝑦̇ (𝑡𝑡)�
2 2 2
��𝑥𝑥̇ (𝑡𝑡)� + �𝑦𝑦̇ (𝑡𝑡)� + �𝑧𝑧̇ (𝑡𝑡)�

𝑑𝑑𝑑𝑑 𝑑𝑑𝑑𝑑 𝑑𝑑𝑑𝑑


〈1, 〉 〈1, , 〉
𝑑𝑑𝑑𝑑 𝑑𝑑𝑑𝑑 𝑑𝑑𝑑𝑑
Explicit 𝑻𝑻 = 𝑻𝑻 =
�𝑟𝑟⃑̇� �𝑟𝑟⃑̇�

𝑑𝑑𝑑𝑑 2 𝑑𝑑𝑑𝑑 𝑑𝑑𝑑𝑑 2 2


where �𝑟𝑟⃑̇� = �1 + �𝑑𝑑𝑑𝑑� where �𝑟𝑟⃑̇� = �1 + �𝑑𝑑𝑑𝑑� + �𝑑𝑑𝑑𝑑�

26
Unit 2d normal vector 𝒏𝒏𝒔𝒔

A direction of a unit normal vector 𝒏𝒏𝒔𝒔 , in 2d, can point inwards or outwards from a contour. A
degree of freedom is removed by applying the following convention. The vector 𝒏𝒏𝒔𝒔 points to your
right if you traverse a closed contour in a CCW direction. This implies that 𝒏𝒏𝒔𝒔 points outward from
the contour.

2d

〈𝑦𝑦̇ (𝑡𝑡), −𝑥𝑥̇ (𝑡𝑡)〉 2 2


Parameterized 𝒏𝒏𝒔𝒔 = �𝑟𝑟⃑̇� = ��𝑥𝑥̇ (𝑡𝑡)� + �𝑦𝑦̇ (𝑡𝑡)�
�𝑟𝑟⃑̇�

𝑑𝑑𝑑𝑑 2
〈 , −1〉 𝑑𝑑𝑑𝑑
Explicit 𝒏𝒏𝒔𝒔 = 𝑑𝑑𝑑𝑑
�𝑟𝑟⃑̇� = �1 + � �
�𝑟𝑟⃑̇� 𝑑𝑑𝑑𝑑

Note: you can show that the vector 𝒏𝒏𝒔𝒔 is normal to 𝑻𝑻 by taking the dot product 𝒏𝒏𝒔𝒔 ∙ 𝑻𝑻 and proving
that the result is zero. Alternatively, you can show that 𝒏𝒏𝒔𝒔 = 𝑻𝑻 × 〈0,0,1〉 where 〈0,0,1〉 is the unit
vector normal to the 𝑥𝑥𝑥𝑥 − plane.

����⃑ and its magnitude 𝒅𝒅𝒅𝒅


Differential arc length vector 𝒅𝒅𝒅𝒅

Differential Arc Length Magnitude of Differential Arc Length


Vector Vector
Parameterized ����⃑ = 𝒅𝒅𝒅𝒅 = 𝑟𝑟⃑̇𝑑𝑑𝑑𝑑
𝑑𝑑𝑑𝑑 ����⃑ � = �𝑟𝑟⃑̇�𝑑𝑑𝑑𝑑
𝑑𝑑𝑑𝑑 = �𝑑𝑑𝑑𝑑

Explicit 𝑑𝑑𝑑𝑑 = 𝒅𝒅𝒅𝒅 = 𝑟𝑟⃑̇𝑑𝑑𝑑𝑑


����⃑ ����⃑ � = �𝑟𝑟⃑̇�𝑑𝑑𝑑𝑑
𝑑𝑑𝑑𝑑 = �𝑑𝑑𝑑𝑑

Magnitude of differential arc Magnitude of differential arc


����⃑� for a 2d contour
length �𝒅𝒅𝒅𝒅 ����⃑� for a 3d contour
length �𝒅𝒅𝒅𝒅

Parameterized 𝑑𝑑𝑑𝑑 = �(𝑥𝑥̇ )2 + (𝑦𝑦̇ )2 𝑑𝑑𝑑𝑑 𝑑𝑑𝑑𝑑 = �(𝑥𝑥̇ )2 + (𝑦𝑦̇ )2 + (𝑧𝑧̇ )2 𝑑𝑑𝑑𝑑

𝑑𝑑𝑑𝑑 = �(𝑑𝑑𝑑𝑑 )2 + (𝑑𝑑𝑑𝑑)2 = 𝑑𝑑𝑑𝑑 = �(𝑑𝑑𝑑𝑑 )2 + (𝑑𝑑𝑑𝑑)2 + (𝑑𝑑𝑑𝑑)2


Explicit 2
�1 + �𝑑𝑑𝑑𝑑� 𝑑𝑑𝑑𝑑 𝑑𝑑𝑑𝑑 2 𝑑𝑑𝑑𝑑 2
𝑑𝑑𝑑𝑑 = �1 + � � + � � 𝑑𝑑𝑑𝑑
𝑑𝑑𝑑𝑑 𝑑𝑑𝑑𝑑

27
Tangent vector:
𝒓𝒓(𝑡𝑡 + ∆𝑡𝑡) − 𝒓𝒓(𝑡𝑡) ∆𝒓𝒓
∆𝒓𝒓 = 𝒓𝒓(𝑡𝑡 + ∆𝑡𝑡) − 𝒓𝒓(𝑡𝑡) = ∆𝑡𝑡 = ∆𝑡𝑡
∆𝑡𝑡 ∆𝑡𝑡
∆𝒓𝒓 𝑑𝑑𝒓𝒓
⟹ 𝒅𝒅𝒅𝒅 = 𝒅𝒅𝒅𝒅 = lim ∆𝑡𝑡 = 𝑑𝑑𝑑𝑑 = 𝒓𝒓′ 𝑑𝑑𝑑𝑑 = 𝒓𝒓̇ 𝑑𝑑𝑑𝑑
∆𝑡𝑡→0 ∆𝑡𝑡 𝑑𝑑𝑑𝑑

Fig. 2 Parameterization of a contour

The next step is to determine the method for computing a circulation and flux integral.

Consider a 2d vector field 𝑭𝑭 = 〈𝑓𝑓(𝑥𝑥, 𝑦𝑦), 𝑔𝑔(𝑥𝑥, 𝑦𝑦)〉.

Circulation integral in 2d: We can express the circulation integral in the following equivalent
forms:

�𝑭𝑭 ∙ 𝑻𝑻𝑑𝑑𝑑𝑑 = �𝑭𝑭 ∙ 𝑟𝑟⃑̇𝑑𝑑𝑑𝑑 = �𝑭𝑭 ∙ 〈𝑥𝑥̇ , 𝑦𝑦̇ 〉 𝑑𝑑𝑑𝑑


𝐶𝐶 𝐶𝐶 𝐶𝐶

= �𝑓𝑓(𝑥𝑥, 𝑦𝑦(𝑥𝑥 ))𝑑𝑑𝑑𝑑 + 𝑔𝑔(𝑥𝑥(𝑦𝑦), 𝑦𝑦)𝑑𝑑𝑑𝑑


𝐶𝐶

Recall that 𝑻𝑻𝑑𝑑𝑑𝑑 = 𝑟𝑟⃑̇𝑑𝑑𝑑𝑑.

28
Flux integral in 2d: The flux integral can be written in the following equivalent forms:

�𝑭𝑭 ∙ 𝒏𝒏𝒔𝒔 𝑑𝑑𝑑𝑑 = �𝑭𝑭 ∙ 〈𝑦𝑦̇ , −𝑥𝑥̇ 〉 𝑑𝑑𝑑𝑑


𝐶𝐶 𝐶𝐶

= �𝑓𝑓(𝑥𝑥(𝑦𝑦), 𝑦𝑦))𝑑𝑑𝑑𝑑 − 𝑔𝑔(𝑥𝑥, 𝑦𝑦(𝑥𝑥))𝑑𝑑𝑑𝑑


𝐶𝐶

Consider a 3d vector field 𝑭𝑭 = 〈𝑓𝑓(𝑥𝑥, 𝑦𝑦, 𝑧𝑧), 𝑔𝑔(𝑥𝑥, 𝑦𝑦, 𝑧𝑧), ℎ(𝑥𝑥, 𝑦𝑦, 𝑧𝑧)〉.

Circulation integral in 3d: We can express the circulation integral in the following equivalent
forms:

𝑑𝑑𝑑𝑑 = �𝑭𝑭 ∙ 𝑟𝑟⃑̇𝑑𝑑𝑑𝑑


�𝑭𝑭 ∙ 𝑻𝑻𝑑𝑑𝑑𝑑 = �𝑭𝑭 ∙ ����⃑
𝐶𝐶 𝐶𝐶 𝐶𝐶

= �𝑓𝑓(𝑥𝑥, 𝑦𝑦(𝑥𝑥 ), 𝑧𝑧(𝑥𝑥 ))𝑑𝑑𝑑𝑑 + 𝑔𝑔(𝑥𝑥 (𝑦𝑦), 𝑦𝑦, 𝑧𝑧(𝑦𝑦))𝑑𝑑𝑑𝑑 + ℎ(𝑥𝑥 (𝑧𝑧), 𝑦𝑦(𝑧𝑧), 𝑧𝑧))𝑑𝑑𝑑𝑑
𝐶𝐶

Important point:

When computing circulation or flux using a parameterized formulation, it is important to


remember that the parameterization must be set up in the following way, assuming 𝑡𝑡 is the
parameterization variable. If you move from starting point 𝐴𝐴 to finishing point 𝐵𝐵, then the position
vector 𝒓𝒓
�⃑ moves from point 𝑨𝑨 to point 𝑩𝑩 as 𝒕𝒕 increases. On the other hand, if you move from
starting point 𝐵𝐵 to finishing point 𝐴𝐴, then the position vector 𝑟𝑟⃑ moves from point 𝐵𝐵 to point 𝐴𝐴 as 𝑡𝑡
increases. Note: the method of parameterization is not unique but try to use the simplest form of
parameterization to make your life easy.

Illustration: Position vector for a straight line from point 𝐴𝐴 at (0,0) to point 𝐵𝐵 at (1,1)

a. Traversing the line from point 𝐵𝐵 to point 𝐴𝐴: 𝑟𝑟⃑ = 〈1 − 𝑡𝑡, 1 − 𝑡𝑡〉 0 ≤ 𝑡𝑡 ≤ 1
b. Traversing the line from point 𝐴𝐴 to point 𝐵𝐵: 𝑟𝑟⃑ = 〈𝑡𝑡, 𝑡𝑡〉 0 ≤ 𝑡𝑡 ≤ 1

1.5.1 Example Contour Integral Problems

1. You are given the following three points on the 𝑥𝑥𝑥𝑥-plane: (0,0), (1,0), (1,1). The points
are connected by the following segments:
a. 𝑦𝑦 = 0 from (0,0) to (1,0), segment 𝐴𝐴,
b. 𝑥𝑥 = 1 from (1,0), (1,1), segment 𝐵𝐵.
c. 𝑦𝑦 = 𝑥𝑥 2 from (1,1), (0,0), segment 𝐶𝐶.

29
A 2d vector field is defined as 𝑭𝑭 = 〈𝑓𝑓(𝑥𝑥, 𝑦𝑦), 𝑔𝑔(𝑥𝑥, 𝑦𝑦)〉 = 〈𝑦𝑦, −𝑥𝑥 〉. The following two
integrals, which are equivalent, assume a counter-clockwise traversal

a. ∮𝐶𝐶 𝑭𝑭 ∙ 𝑻𝑻𝑑𝑑𝑑𝑑
b. ∮𝐶𝐶 𝑓𝑓(𝑥𝑥, 𝑦𝑦(𝑥𝑥 ))𝑑𝑑𝑑𝑑 + 𝑔𝑔(𝑥𝑥 (𝑦𝑦), 𝑦𝑦)𝑑𝑑𝑑𝑑

For the integral in (a), the differential arc length vector 𝑻𝑻𝑑𝑑𝑑𝑑 is parameterized in terms of a
variable 𝑡𝑡. For the integral in (b), the differential arc length vector 𝑻𝑻𝑑𝑑𝑑𝑑 is represented by
〈𝑑𝑑𝑑𝑑, 𝑑𝑑𝑑𝑑〉. Compute the answers for case (a) and (b) and confirm that the results are
equivalent.

Solution (a)

Segment 𝐴𝐴: 𝑟𝑟⃑ = 〈𝑡𝑡, 0〉 𝑟𝑟⃑̇ = 〈1,0〉 0 ≤ 𝑡𝑡 ≤ 1 𝑭𝑭 = 〈0, −𝑡𝑡〉


Segment 𝐵𝐵: 𝑟𝑟⃑ = 〈1, 𝑡𝑡〉 𝑟𝑟⃑̇ = 〈0,1〉 0 ≤ 𝑡𝑡 ≤ 1 𝑭𝑭 = 〈𝑡𝑡, −1〉
Segment 𝐶𝐶: 𝑟𝑟⃑ = 〈1 − 𝑡𝑡, (1 − 𝑡𝑡 )2 〉 𝑟𝑟⃑̇ = 〈−1, −2(1 − 𝑡𝑡)〉 0 ≤ 𝑡𝑡 ≤ 1
𝑭𝑭 = 〈(1 − 𝑡𝑡)2 , −(1 − 𝑡𝑡)〉

𝑆𝑆𝑆𝑆𝑆𝑆𝑆𝑆𝑆𝑆𝑆𝑆𝑆𝑆 𝐴𝐴 𝑆𝑆𝑆𝑆𝑆𝑆𝑆𝑆𝑆𝑆𝑆𝑆𝑆𝑆 𝐵𝐵 𝑆𝑆𝑆𝑆𝑆𝑆𝑆𝑆𝑆𝑆𝑆𝑆𝑆𝑆 𝐶𝐶


1 1 1
�𝑭𝑭 ∙ 𝑟𝑟̇⃑𝑑𝑑𝑑𝑑 = � 〈0, −𝑡𝑡〉 ∙ 〈1,0〉𝑑𝑑𝑑𝑑 + � 〈𝑡𝑡, −1〉 ∙ 〈0,1〉𝑑𝑑𝑑𝑑 + � 〈(1 − 𝑡𝑡)2 , −(1 − 𝑡𝑡)〉
𝐶𝐶 0 0 0
1 1 1
2
∙ 〈−1, −2(1 − 𝑡𝑡)〉𝑑𝑑𝑑𝑑 = � 0𝑑𝑑𝑑𝑑 − � 𝑑𝑑𝑑𝑑 + � (1 − 𝑡𝑡)2 𝑑𝑑𝑑𝑑 = −
0 0 0 3
Solution (b)

�𝑓𝑓(𝑥𝑥, 𝑦𝑦(𝑥𝑥 ))𝑑𝑑𝑑𝑑 + 𝑔𝑔(𝑥𝑥 (𝑦𝑦), 𝑦𝑦)𝑑𝑑𝑑𝑑 = �𝑦𝑦(𝑥𝑥)𝑑𝑑𝑑𝑑 − 𝑥𝑥(𝑦𝑦)𝑑𝑑𝑑𝑑


𝐶𝐶 𝐶𝐶

In evaluating this expression, we need to express 𝑦𝑦 as a function of 𝑥𝑥 for the closed contour
integral involving 𝑑𝑑𝑑𝑑 and express 𝑥𝑥 as a function of 𝑦𝑦 for the closed contour integral
involving 𝑑𝑑𝑑𝑑. Since the closed contour involves three piecewise continuous line segments,
there will be three separate integrals comprising the closed contour integral. The results for
both integrals are summarized next.

For ∮𝐶𝐶 𝑦𝑦(𝑥𝑥)𝑑𝑑𝑑𝑑, the segments, as described in part (a), are the same here, therefore,

𝑆𝑆𝑆𝑆𝑆𝑆𝑆𝑆𝑆𝑆𝑆𝑆𝑆𝑆 𝐴𝐴: 𝑦𝑦 = 0,
𝑆𝑆𝑆𝑆𝑆𝑆𝑆𝑆𝑆𝑆𝑆𝑆𝑆𝑆 𝐵𝐵: function for 𝑦𝑦 does not exist and 𝑑𝑑𝑑𝑑 = 0
𝑆𝑆𝑆𝑆𝑆𝑆𝑆𝑆𝑆𝑆𝑆𝑆𝑆𝑆 𝐶𝐶: 𝑦𝑦 = 𝑥𝑥 2 .

30
Hence the integral becomes

𝑆𝑆𝑆𝑆𝑆𝑆𝑆𝑆𝑆𝑆𝑆𝑆𝑆𝑆 𝐴𝐴 𝑆𝑆𝑆𝑆𝑆𝑆𝑆𝑆𝑆𝑆𝑆𝑆𝑆𝑆 𝐶𝐶
1 0
1
�𝑦𝑦(𝑥𝑥)𝑑𝑑𝑑𝑑 = � 0𝑑𝑑𝑑𝑑 + � 𝑥𝑥 2 𝑑𝑑𝑑𝑑 = −
𝐶𝐶 0 1 3

For ∮𝐶𝐶 𝑥𝑥(𝑦𝑦)𝑑𝑑𝑑𝑑, 𝑆𝑆𝑆𝑆𝑆𝑆𝑆𝑆𝑆𝑆𝑆𝑆𝑆𝑆 𝐴𝐴: function for 𝑥𝑥 does not exist and 𝑑𝑑𝑑𝑑 = 0, 𝑆𝑆𝑆𝑆𝑆𝑆𝑆𝑆𝑆𝑆𝑆𝑆𝑆𝑆 𝐵𝐵: 𝑥𝑥 =
1, 𝑆𝑆𝑆𝑆𝑆𝑆𝑆𝑆𝑆𝑆𝑆𝑆𝑆𝑆 𝐶𝐶: 𝑥𝑥 = �𝑦𝑦, therefore,

𝑆𝑆𝑆𝑆𝑆𝑆𝑆𝑆𝑆𝑆𝑆𝑆𝑆𝑆 𝐵𝐵 𝑆𝑆𝑆𝑆𝑆𝑆𝑆𝑆𝑆𝑆𝑆𝑆𝑆𝑆 𝐶𝐶
1 0
1
�−𝑥𝑥(𝑦𝑦)𝑑𝑑𝑑𝑑 = − � 𝑑𝑑𝑑𝑑 − � �𝑦𝑦 𝑑𝑑𝑑𝑑 = −
𝐶𝐶 0 1 3

Hence

2
�𝑦𝑦(𝑥𝑥)𝑑𝑑𝑑𝑑 − 𝑥𝑥(𝑦𝑦)𝑑𝑑𝑑𝑑 = −
𝐶𝐶 3

which is the same as Solution (a), as it should be.

2. Consider a contour in 3d parameterized as follows:

√5 2
𝑟𝑟⃑ = 〈𝑡𝑡 2 , 3𝑡𝑡, 𝑡𝑡 〉 0 ≤ 𝑡𝑡 ≤ 1
2

and assume 𝜆𝜆(𝑥𝑥(𝑡𝑡), 𝑦𝑦(𝑡𝑡), 𝑧𝑧(𝑡𝑡)) = 𝑡𝑡 represents the charge density in units of 𝐶𝐶/𝑚𝑚.

Develop the integral expression for the total charge on the line ∫𝐶𝐶 𝜆𝜆(𝑠𝑠) 𝑑𝑑𝑑𝑑 using the
following two approaches and show that they are equivalent:

1
a. ∫0 𝜆𝜆(𝑥𝑥 (𝑡𝑡), 𝑦𝑦(𝑡𝑡), 𝑧𝑧(𝑡𝑡))�(𝑥𝑥̇ )2 + (𝑦𝑦̇ )2 + (𝑧𝑧̇ )2 𝑑𝑑𝑑𝑑
1 𝑑𝑑𝑑𝑑(𝑥𝑥) 2 𝑑𝑑𝑑𝑑(𝑥𝑥) 2
b. ∫0 𝜆𝜆(𝑥𝑥, 𝑦𝑦(𝑥𝑥 ), 𝑧𝑧(𝑥𝑥))�1 + � � +� � 𝑑𝑑𝑑𝑑
𝑑𝑑𝑑𝑑 𝑑𝑑𝑑𝑑

31
Solution (a)

√5
We need to determine 𝑟𝑟⃑̇ first from the expression 𝑟𝑟⃑ = 〈𝑡𝑡 2 , 3𝑡𝑡, 2 𝑡𝑡 2 〉.

𝑟𝑟⃑̇ = 〈2𝑡𝑡, 3, √5𝑡𝑡〉 0 ≤ 𝑡𝑡 ≤ 1


�𝑟𝑟⃑̇� = �(𝑥𝑥̇ )2 + (𝑦𝑦̇ )2 + (𝑧𝑧̇ )2 = 3�𝑡𝑡 2 + 1
1 1
� 𝜆𝜆(𝑥𝑥 (𝑡𝑡), 𝑦𝑦(𝑡𝑡), 𝑧𝑧(𝑡𝑡))�𝑟𝑟⃑̇�𝑑𝑑𝑑𝑑 = � 𝜆𝜆(𝑥𝑥(𝑡𝑡), 𝑦𝑦(𝑡𝑡), 𝑧𝑧(𝑡𝑡))�(𝑥𝑥̇ )2 + (𝑦𝑦̇ )2 + (𝑧𝑧̇ )2 𝑑𝑑𝑑𝑑
0 0
1
= � 3𝑡𝑡 �𝑡𝑡 2 + 1𝑑𝑑𝑑𝑑
0

Next consider a substitution of variables: 𝑥𝑥 = 𝑡𝑡 2 𝑑𝑑𝑑𝑑 = 2𝑡𝑡𝑡𝑡𝑡𝑡. The upper and lower integral
13
limits become 1 and 0 respectively. The resultant integral is ∫0 √𝑥𝑥 + 1𝑑𝑑𝑑𝑑.
2

Solution (b)

We need an expression of the form

𝑟𝑟⃑ = 〈𝑥𝑥, 𝑖𝑖(𝑥𝑥 ), 𝑗𝑗(𝑥𝑥)〉

working from the original definition for 𝑟𝑟⃑. The result is

𝑖𝑖 (𝑥𝑥 ) = 3√𝑥𝑥
√5
𝑗𝑗(𝑥𝑥 ) = 𝑥𝑥
2

therefore 𝜆𝜆(𝑥𝑥, 𝑦𝑦(𝑥𝑥 ), 𝑧𝑧(𝑥𝑥)) = √𝑥𝑥 and the limits of integration are 0 ≤ 𝑥𝑥 ≤ 1.

1
𝑑𝑑𝑑𝑑(𝑥𝑥) 2 𝑑𝑑𝑑𝑑(𝑥𝑥) 2 1
9 5 1
3
� √𝑥𝑥 �1 + � � +� � 𝑑𝑑𝑑𝑑 = � √𝑥𝑥 �1 + + 𝑑𝑑𝑑𝑑 = � √𝑥𝑥 + 1𝑑𝑑𝑑𝑑
0 𝑑𝑑𝑑𝑑 𝑑𝑑𝑑𝑑 0 4𝑥𝑥 4 0 2

This integral has the same form as the integral in Solution (a).

3. The temperature of the circular plate 𝑅𝑅 = {(𝑥𝑥, 𝑦𝑦): 𝑥𝑥 2 + 𝑦𝑦 2 ≤ 1} is 𝑇𝑇(𝑥𝑥, 𝑦𝑦) = 100(𝑥𝑥 2 +
2𝑦𝑦 2 ) 𝐶𝐶 0 . Compute the average temperature on the boundary of the plate.

32
Solution

The average temperature over a closed contour is defined by:

∮𝐶𝐶 𝑇𝑇(𝑠𝑠)𝑑𝑑𝑑𝑑
𝑇𝑇𝑎𝑎𝑎𝑎𝑎𝑎 =
∮𝐶𝐶 𝑑𝑑𝑑𝑑

Hence, we need to compute two integrals.

We use the parametric description of the boundary and proceed from there:

𝐶𝐶 = {(𝑥𝑥, 𝑦𝑦): 𝑥𝑥 = 𝑐𝑐𝑐𝑐𝑐𝑐𝑐𝑐, 𝑦𝑦 = 𝑠𝑠𝑠𝑠𝑠𝑠𝑠𝑠, 0 ≤ 𝑡𝑡 ≤ 2𝜋𝜋}

𝑟𝑟⃑ = 〈𝑐𝑐𝑐𝑐𝑐𝑐𝑐𝑐, 𝑠𝑠𝑠𝑠𝑠𝑠𝑠𝑠〉 𝑟𝑟̇⃑ = 〈−𝑠𝑠𝑠𝑠𝑛𝑛𝑛𝑛, 𝑐𝑐𝑐𝑐𝑐𝑐𝑐𝑐〉 𝑑𝑑𝑑𝑑 = �𝑟𝑟̇⃑�𝑑𝑑𝑑𝑑 = �(𝑥𝑥̇ )2 + (𝑦𝑦̇ )2 𝑑𝑑𝑑𝑑 = 𝑑𝑑𝑑𝑑

2𝜋𝜋 2𝜋𝜋
�𝑇𝑇(𝑠𝑠)𝑑𝑑𝑑𝑑 = � 𝑇𝑇(𝑥𝑥(𝑡𝑡), 𝑦𝑦(𝑡𝑡)) 𝑑𝑑𝑑𝑑 = � 100(𝑥𝑥 2 (𝑡𝑡) + 2𝑦𝑦 2 (𝑡𝑡)) 𝑑𝑑𝑑𝑑
𝐶𝐶 0 0
2𝜋𝜋
= � 100(𝑐𝑐𝑐𝑐𝑐𝑐 2 (𝑡𝑡) + 2𝑠𝑠𝑠𝑠𝑠𝑠2 (𝑡𝑡)) 𝑑𝑑𝑑𝑑
0
2𝜋𝜋 2𝜋𝜋 2𝜋𝜋
1 − 𝑐𝑐𝑐𝑐𝑐𝑐2𝑡𝑡
� 100�2 − 𝑐𝑐𝑐𝑐𝑐𝑐 2 (𝑡𝑡)� 𝑑𝑑𝑑𝑑 = � 100 �2 − � �� 𝑑𝑑𝑑𝑑 = � 50(3 + 𝑐𝑐𝑐𝑐𝑐𝑐2𝑡𝑡)𝑑𝑑𝑑𝑑
0 0 2 0
= 300𝜋𝜋
2𝜋𝜋
�𝑑𝑑𝑑𝑑 = � 𝑑𝑑𝑑𝑑 = 2𝜋𝜋
𝐶𝐶 0
∮𝐶𝐶 𝑇𝑇(𝑠𝑠)𝑑𝑑𝑑𝑑 300𝜋𝜋
𝑇𝑇𝑎𝑎𝑎𝑎𝑎𝑎 = = = 150 𝐶𝐶 0
∮𝐶𝐶 𝑑𝑑𝑑𝑑 2𝜋𝜋

The method of parameterizing a contour is not unique. Spherical and cylindrical coordinates are
but two examples of parametrization used frequently. Parametrization in general is very helpful in
expressing complex surfaces in a more computationally viable coordinate system. At this point
you can work on the following problems which have been extracted from section 17.2 of the
textbook:

1.5.2 Problems from Section 17.2: 1, 2, 18, 23, 27, 35, 39, 43, 49, 57, 65, 75, 77, 79

𝑏𝑏
1. (question 1) How does a line integral differ from the single-variable integral ∫𝑎𝑎 𝑓𝑓 (𝑥𝑥 )𝑑𝑑𝑑𝑑 ?
2. (question 2) If a curve C is given by 𝒓𝒓(𝑡𝑡) = 〈𝑡𝑡, 𝑡𝑡 2 〉, what is |𝒓𝒓′ (𝑡𝑡)|?

33
3. (question 18) Scalar line integral Evaluate the following line integral along the curve C:
𝑡𝑡 𝑡𝑡
∫𝐶𝐶 (𝑥𝑥 2 − 2𝑦𝑦 2 )𝑑𝑑𝑑𝑑: 𝐶𝐶 is the line segment 𝒓𝒓(𝑡𝑡) = 〈 2 , 2〉, for 0 ≤ 𝑡𝑡 ≤ 4.
√ √
4. (question 23) Scalar line integral Evaluate the following line integral along the curve C:
∫𝐶𝐶 (𝑦𝑦 − 𝑧𝑧)𝑑𝑑𝑑𝑑: 𝐶𝐶 is the line segment 𝒓𝒓(𝑡𝑡) = 〈3𝑐𝑐𝑐𝑐𝑐𝑐𝑐𝑐, 3𝑠𝑠𝑠𝑠𝑠𝑠𝑠𝑠, 4𝑡𝑡〉, for 0 ≤ 𝑡𝑡 ≤ 2𝜋𝜋
5. (question 27) Scalar line integral Evaluate the following line integral along the curve C:
𝑥𝑥
∫𝐶𝐶 2 +𝑦𝑦2 𝑑𝑑𝑑𝑑: 𝐶𝐶 is the line segment from (1,1) to (10,10).
𝑥𝑥
6. (question 35) Mass and density A thin wire represented by the smooth curve C with a
mass density 𝜆𝜆 (mass per unit length) has a mass ∫𝐶𝐶 𝜆𝜆𝜆𝜆𝜆𝜆. Find the mass of the following
wire: 𝐶𝐶: {(𝑥𝑥, 𝑦𝑦): 𝑦𝑦 = 2𝑥𝑥 2 , 0 ≤ 𝑥𝑥 ≤ 3}: 𝜆𝜆(𝑥𝑥, 𝑦𝑦) = 1 + 𝑥𝑥𝑥𝑥.
7. (question 39) Length of curves Use a scalar line integral to find the length of the following
𝑡𝑡 𝑡𝑡
line: 𝒓𝒓(𝑡𝑡) = 〈20𝑠𝑠𝑠𝑠𝑠𝑠 � � , 20𝑐𝑐𝑐𝑐𝑐𝑐 � � , 𝑡𝑡/2〉, for 0 ≤ 𝑡𝑡 ≤ 2
4 4
8. (question 43) Line integrals of vector field in a plane Given the following vector field
and oriented curve 𝐶𝐶, evaluate ∫𝐶𝐶 𝑭𝑭 ∙ 𝑻𝑻𝑑𝑑𝑑𝑑 : 𝑭𝑭 = 〈𝑦𝑦, 𝑥𝑥 〉 on the line segment from (1,1) to
(5,10).
9. (question 49) Work integrals Given the force field 𝑭𝑭, find the work required to move an
object on the given oriented curve: 𝑭𝑭 = 〈𝑦𝑦, −𝑥𝑥 〉 on the line segment from (1,2) to (0,0)
followed by the line segment from (0,0) to (0,4).
10. (question 57) Circulation Consider the following vector field 𝑭𝑭 and closed oriented curve
𝐶𝐶 in the plane (see figure below): 𝑭𝑭 = 〈𝑦𝑦 − 𝑥𝑥, 𝑥𝑥 〉: 𝐶𝐶: 𝒓𝒓(𝑡𝑡) = 〈2𝑐𝑐𝑐𝑐𝑐𝑐𝑐𝑐, 2𝑠𝑠𝑠𝑠𝑠𝑠𝑠𝑠〉, for 0 ≤ 𝑡𝑡 ≤
2𝜋𝜋.
a. Based on the picture, make a conjecture about whether the circulation of 𝑭𝑭 on 𝐶𝐶 is
positive negative or zero.
b. Compute the circulation and interpret the results.

34
11. (question 65) Changing orientation Let 𝑓𝑓(𝑥𝑥, 𝑦𝑦) = 𝑥𝑥 and let 𝐶𝐶 be the segment of the
parabola 𝑦𝑦 = 𝑥𝑥 2 joining 𝑂𝑂(0,0) and 𝑃𝑃(1,1).
a. Find a parametrization of 𝐶𝐶 in the direction from 𝑂𝑂 to 𝑃𝑃. Evaluate ∫𝐶𝐶 𝑓𝑓𝑓𝑓𝑓𝑓
b. Find a parametrization of 𝐶𝐶 in the direction from 𝑃𝑃 to 𝑂𝑂. Evaluate ∫𝐶𝐶 𝑓𝑓𝑓𝑓𝑓𝑓
c. Compare the results of parts (a) and (b).
12. (question 75) Zero Circulation field Consider the vector field 𝑭𝑭 = 〈𝑎𝑎𝑎𝑎 + 𝑏𝑏𝑏𝑏, 𝑐𝑐𝑐𝑐 + 𝑑𝑑𝑑𝑑〉.
Show that 𝑭𝑭 has zero circulation on any oriented circle centered at the origin, for any 𝑎𝑎, 𝑏𝑏,
𝑐𝑐 and 𝑑𝑑 provided 𝑏𝑏 = 𝑐𝑐.
13. (question 77) Zero flux field Consider the vector field 𝑭𝑭 = 〈𝑎𝑎𝑎𝑎 + 𝑏𝑏𝑏𝑏, 𝑐𝑐𝑐𝑐 + 𝑑𝑑𝑑𝑑〉. Show that
𝑭𝑭 has zero flux across any oriented circle centered at the origin, for any 𝑎𝑎, 𝑏𝑏, 𝑐𝑐 and 𝑑𝑑
provided 𝑎𝑎 = −𝑑𝑑.
𝒓𝒓 〈𝑥𝑥,𝑦𝑦,𝑧𝑧〉
14. (question 79) Inverse force fields Consider the radial field 𝐹𝐹 = |𝒓𝒓|𝑝𝑝 = |𝒓𝒓|𝑝𝑝
, where 𝑝𝑝 > 1
(the inverse square law corresponds to 𝑝𝑝 = 3). Le 𝐶𝐶 be the line segment from (1,1,1) to
(𝑎𝑎, 𝑎𝑎, 𝑎𝑎), where 𝑎𝑎 > 1, given by 𝒓𝒓(𝑡𝑡) = 〈𝑡𝑡, 𝑡𝑡, 𝑡𝑡〉 for 1 ≤ 𝑡𝑡 ≤ 𝑎𝑎.
a. Find the work done in moving an object along 𝐶𝐶 with 𝑝𝑝 = 2.
b. If 𝑎𝑎 → ∞ in part(a), is the work finite?
c. Find the work done in moving an object along 𝐶𝐶 with 𝑝𝑝 = 4.
d. If 𝑎𝑎 → ∞ in part(c), is the work finite?
e. Find the work done in moving an object along 𝐶𝐶 with any 𝑝𝑝 > 1.
f. If 𝑎𝑎 → ∞ in part(e), for what values of 𝑝𝑝 is the work finite?

You can find the solutions to the problems at the following link: Solutions to Reading Assignment
4 Problems.

1.6 Derivation of a Surface Integral from a Riemann Sum and Examples

We will now define what is meant by a surface integral derived from a Riemann sum and consider
some examples in parallel.

Resultant of integrating a scalar density with physical meaning over a surface:

Physical attribute: The scalar density 𝜎𝜎(𝑥𝑥, 𝑦𝑦) in 2d has a physical meaning with units such as
𝐶𝐶
charge per unit area ( ). Then the integral for ∬𝑆𝑆 𝜎𝜎(𝑥𝑥, 𝑦𝑦) 𝑑𝑑𝑑𝑑 in 2d (𝑆𝑆 is a 2d planar surface)
𝑚𝑚2
represents the total charge on the surface defined by 𝑆𝑆. A surface in 2d is planar and has no
orientation and so sometimes 𝑑𝑑𝑑𝑑 is replaced by 𝑑𝑑𝑑𝑑 to indicate that it is a plane coinciding with the
𝑥𝑥𝑥𝑥 plane. To be more general, 𝑑𝑑𝑑𝑑 for any plane that is parallel to one of the three principal planes
in 3d, that is the 𝑥𝑥𝑥𝑥, 𝑥𝑥𝑥𝑥 or 𝑦𝑦𝑦𝑦 planes, can be written as 𝑑𝑑𝑑𝑑 = 𝑑𝑑𝑑𝑑 = 𝑑𝑑𝑑𝑑𝑑𝑑𝑑𝑑, 𝑑𝑑𝑑𝑑𝑑𝑑𝑑𝑑 or 𝑑𝑑𝑑𝑑𝑑𝑑𝑑𝑑
respectively. You can show this to be the case once the equations for computing 𝑑𝑑𝑆𝑆 are derived
later in this section.

35
The integral ∬𝑆𝑆 𝜎𝜎(𝑥𝑥(𝑢𝑢, 𝑣𝑣), 𝑦𝑦(𝑢𝑢, 𝑣𝑣), 𝑧𝑧(𝑢𝑢, 𝑣𝑣))𝑑𝑑𝑑𝑑 (𝑆𝑆 is an open curved surface in 3d) or
∯𝑆𝑆 𝜎𝜎(𝑥𝑥(𝑢𝑢, 𝑣𝑣), 𝑦𝑦(𝑢𝑢, 𝑣𝑣), 𝑧𝑧(𝑢𝑢, 𝑣𝑣))𝑑𝑑𝑑𝑑 (𝑆𝑆 is a closed curved surface in 3d) represents the total charge
on the curved surface defined by 𝑆𝑆. The use of two new variables 𝑢𝑢 and 𝑣𝑣 comes about since points
on the curved surface 𝑆𝑆 are mapped to points on a planar surface 𝑢𝑢𝑢𝑢. 𝑑𝑑𝑑𝑑 will be expressed in terms
of 𝑢𝑢 and 𝑣𝑣 and the differential area 𝑑𝑑𝑑𝑑𝑑𝑑𝑑𝑑. This mapping and how the integral are formulated in
terms of a Riemann sum will be discussed next.

The next step is to show how∬𝑆𝑆 𝜎𝜎(𝑥𝑥(𝑢𝑢, 𝑣𝑣), 𝑦𝑦(𝑢𝑢, 𝑣𝑣), 𝑧𝑧(𝑢𝑢, 𝑣𝑣))𝑑𝑑𝑑𝑑 is formulated as a Riemann Sum
with reference to Figures 3, 4 and 5. In performing integrations, we are familiar with integrating
over a plane. A curved surface is a form of plane that has been twisted and contorted. However, it
is possible to create a mapping that takes every point on the curved surface and maps it to a point
on a planar surface. The points on the curved surface in 3d are represented by three variables but
the 𝑧𝑧 component of the points on the surface are functions of 𝑥𝑥 and 𝑦𝑦. If we define a plane with
coordinates 𝑢𝑢 and 𝑣𝑣, then in principle it is possible to map points on the 𝑢𝑢𝑢𝑢 plane to points
(𝑥𝑥 (𝑢𝑢, 𝑣𝑣 ), 𝑦𝑦(𝑢𝑢, 𝑣𝑣 ), 𝑧𝑧(𝑢𝑢, 𝑣𝑣)) on the curved surface. These points on the surface are mapped out by a
position vector 𝑟𝑟⃑, which is a function of the variables 𝑢𝑢 and 𝑣𝑣, as follows:

𝑟𝑟⃑ = 〈𝑥𝑥 (𝑢𝑢, 𝑣𝑣 ), 𝑦𝑦(𝑢𝑢, 𝑣𝑣 ), 𝑧𝑧(𝑢𝑢, 𝑣𝑣)〉

The curved surface will have a boundary and so the boundary points will map to boundary points
in the 𝑢𝑢𝑢𝑢 plane. This formal process just described is referred to as parameterization of the
surface.

The method to parameterize a surface is not unique. However, there will be certain mappings
which are computationally simple, and it is those that we will be in search of. In order to keep
things simple, for the derivation, we will assume for now that the boundary of the curved surface
when mapped to the 𝑢𝑢𝑢𝑢 plane is a rectangle, as shown in Fig. 3.

Fig. 3 Mapping of Region 𝑹𝑹 to region 𝑺𝑺

36
At this point it is helpful to illustrate the parameterization process using several examples, in which
we assume that the region in the 𝑢𝑢𝑢𝑢 plane is rectangular.

1.6.1 Example Problems on Surface Parameterization

Example 1: Parameterization of a cylinder

In Cartesian coordinates, the set

{(𝑥𝑥, 𝑦𝑦, 𝑧𝑧): 𝑥𝑥 = 𝑎𝑎 𝑐𝑐𝑐𝑐𝑐𝑐𝑐𝑐, 𝑦𝑦 = 𝑎𝑎 𝑠𝑠𝑠𝑠𝑠𝑠𝑠𝑠, 0 ≤ 𝜃𝜃 ≤ 2𝜋𝜋, 0 ≤ 𝑧𝑧 ≤ ℎ}

is a cylindrical surface of radius 𝑎𝑎 and height ℎ with its axis along the 𝑧𝑧 -axis. Using the parameters
𝑢𝑢 = 𝜃𝜃 and 𝑣𝑣 = 𝑧𝑧, a parametric description of the cylinder is

𝑟𝑟⃑(𝑢𝑢, 𝑣𝑣 ) = 〈𝑥𝑥(𝑢𝑢, 𝑣𝑣 ), 𝑦𝑦(𝑢𝑢, 𝑣𝑣 ), 𝑧𝑧(𝑢𝑢, 𝑣𝑣)〉 = 〈𝑎𝑎 𝑐𝑐𝑐𝑐𝑐𝑐𝑐𝑐, 𝑎𝑎 𝑠𝑠𝑠𝑠𝑠𝑠𝑠𝑠, 𝑣𝑣 〉

where 0 ≤ 𝑢𝑢 ≤ 2𝜋𝜋 and 0 ≤ 𝑣𝑣 ≤ ℎ, as per Figure Ex 1.

Fig. Ex 1 Surface parameterization of a cylinder

Example 2: Parameterization of a sphere

The parametric description of a sphere of radius 𝑎𝑎, centred at the origin, comes directly from
spherical coordinates:

{(𝜌𝜌, 𝜑𝜑, 𝜃𝜃 ): 𝜌𝜌 = 𝑎𝑎, 0 ≤ 𝜑𝜑 ≤ 𝜋𝜋, 0 ≤ 𝜃𝜃 ≤ 2𝜋𝜋}

Recall the following relationship among spherical coordinates (Section 14.5 of the textbook).

37
𝑥𝑥 = 𝑎𝑎 𝑠𝑠𝑠𝑠𝑠𝑠𝑠𝑠 𝑐𝑐𝑐𝑐𝑐𝑐𝑐𝑐, 𝑦𝑦 = 𝑎𝑎 𝑠𝑠𝑠𝑠𝑠𝑠𝜑𝜑 𝑠𝑠𝑠𝑠𝑠𝑠𝑠𝑠, 𝑧𝑧 = 𝑎𝑎 𝑐𝑐𝑐𝑐𝑐𝑐𝑐𝑐
When we define the parameters 𝑢𝑢 = 𝜑𝜑 and 𝑣𝑣 = 𝜃𝜃, a parametric description of the sphere is

𝑟𝑟⃑(𝑢𝑢, 𝑣𝑣 ) = 〈𝑎𝑎 𝑠𝑠𝑠𝑠𝑠𝑠𝑠𝑠 𝑐𝑐𝑐𝑐𝑐𝑐𝑐𝑐, 𝑎𝑎 𝑠𝑠𝑠𝑠𝑠𝑠𝑠𝑠 𝑠𝑠𝑠𝑠𝑠𝑠𝑠𝑠, 𝑎𝑎 𝑐𝑐𝑐𝑐𝑐𝑐𝑐𝑐〉

where 0 ≤ 𝑢𝑢 ≤ 𝜋𝜋 and 0 ≤ 𝑣𝑣 ≤ 2𝜋𝜋, as per Figure Ex 2.

Fig. Ex 2 Surface parameterization of a sphere

At this point you can work on the following problems from section 17.6 of the textbook:

1.6.2 Problems from Section 17.6: 3, 5, 9, 11, 17

1. (question 3) Give a parametric description for a sphere with radius 𝑎𝑎, including the
intervals for the parameters.
2. (question 5) Explain how to compute the surface integral of a scalar-valued function 𝑓𝑓 over
a sphere using a parametric description of the sphere.
3. (question 9) Parametric description Give a parametric description of the form 𝒓𝒓(𝑢𝑢, 𝑣𝑣 ) =
〈𝑥𝑥(𝑢𝑢, 𝑣𝑣 ), 𝑦𝑦(𝑢𝑢, 𝑣𝑣 ), 𝑧𝑧(𝑢𝑢, 𝑣𝑣)〉 for the following surface. The description is not unique. Specify
the required rectangle in the 𝑢𝑢𝑢𝑢 − plane: the plane 2𝑥𝑥 − 4𝑦𝑦 + 3𝑧𝑧 = 16.
4. (question 11) Parametric description Give a parametric description of the form 𝒓𝒓(𝑢𝑢, 𝑣𝑣 ) =
〈𝑥𝑥(𝑢𝑢, 𝑣𝑣 ), 𝑦𝑦(𝑢𝑢, 𝑣𝑣 ), 𝑧𝑧(𝑢𝑢, 𝑣𝑣)〉 for the following surface. The description is not unique. Specify
the required rectangle in the 𝑢𝑢𝑢𝑢 − plane: The frustrum of the cone 𝑧𝑧 2 = 𝑥𝑥 2 + 𝑦𝑦 2 , for 2 ≤
𝑧𝑧 ≤ 8.

38
5. (question 17) Identify the surface Describe the surface with the given parametric
representation: 𝒓𝒓(𝑢𝑢, 𝑣𝑣 ) = 〈𝑣𝑣 𝑐𝑐𝑐𝑐𝑐𝑐𝑐𝑐, 𝑣𝑣 𝑠𝑠𝑠𝑠𝑠𝑠𝑠𝑠, 4𝑣𝑣 〉, for 0 ≤ 𝑢𝑢 ≤ 𝜋𝜋, 0 ≤ 𝑣𝑣 ≤ 3.

You can find the solutions to the problems at the following link: Solutions to Reading Assignment
4 Problems.

1.6.3 Surface Integrals of Scalar-Valued Functions

We now develop the surface integral of a scalar-valued function 𝑓𝑓 defined on a smooth


parameterized surface 𝑆𝑆 described by the equation

𝑟𝑟⃑(𝑢𝑢, 𝑣𝑣 ) = 〈𝑥𝑥(𝑢𝑢, 𝑣𝑣), 𝑦𝑦(𝑢𝑢, 𝑣𝑣), 𝑧𝑧(𝑢𝑢, 𝑣𝑣)〉

where the parameters vary over a rectangle 𝑅𝑅 = {(𝑢𝑢, 𝑣𝑣 ): 𝑎𝑎 ≤ 𝑢𝑢 ≤ 𝑏𝑏, 𝑐𝑐 ≤ 𝑣𝑣 ≤ 𝑑𝑑 }. The functions 𝑥𝑥,
𝑦𝑦 and 𝑧𝑧 are assumed to have continuous partial derivatives with respect to 𝑢𝑢 and 𝑣𝑣. The rectangular
region 𝑅𝑅 in the 𝑢𝑢𝑢𝑢-plane is partitioned into rectangles, with sides of length ∆𝑢𝑢 and ∆𝑣𝑣, that are
ordered in some convenient way, for 𝑘𝑘 = 1, … , 𝑛𝑛. The 𝑘𝑘𝑘𝑘ℎ rectangle 𝑅𝑅𝑘𝑘 which has area ∆𝐴𝐴 =
∆𝑢𝑢∆𝑣𝑣, corresponds to a curved patch 𝑆𝑆𝑘𝑘 , on the surface, which has area ∆𝑆𝑆𝑘𝑘 , as shown in Fig. 4.

Fig. 4 Mapping of a differential area in the 𝒖𝒖𝒖𝒖-plane to a differential area on the curved
surface

We let (𝑢𝑢𝑘𝑘 , 𝑣𝑣𝑘𝑘 ) be the lower-left corner point of 𝑅𝑅𝑘𝑘 . The parameterization then assigns (𝑢𝑢𝑘𝑘 , 𝑣𝑣𝑘𝑘 ) to
a point 𝑃𝑃(𝑥𝑥(𝑢𝑢𝑘𝑘 , 𝑣𝑣𝑘𝑘 ), 𝑦𝑦(𝑢𝑢𝑘𝑘 , 𝑣𝑣𝑘𝑘 ), 𝑧𝑧(𝑢𝑢𝑘𝑘 , 𝑣𝑣𝑘𝑘 )), or more simply, 𝑃𝑃(𝑥𝑥𝑘𝑘 , 𝑦𝑦𝑘𝑘 , 𝑧𝑧𝑘𝑘 ), on 𝑆𝑆𝑘𝑘 . To construct the
surface integral, we define a Riemann sum, which adds up function values multiplied by areas of
the respective patches: So, for example, if a surface patch has a particular property 𝜎𝜎 (eg.
𝐶𝐶
charge/unit area) with units then the total charge 𝑄𝑄 over the curved surface is given by:
𝑚𝑚2

39
𝑛𝑛

𝑄𝑄 = � 𝜎𝜎(𝑥𝑥 (𝑢𝑢𝑘𝑘 , 𝑣𝑣𝑘𝑘 ), 𝑦𝑦(𝑢𝑢𝑘𝑘 , 𝑣𝑣𝑘𝑘 ), 𝑧𝑧(𝑢𝑢𝑘𝑘 , 𝑣𝑣𝑘𝑘 ))∆𝑆𝑆𝑘𝑘


𝑘𝑘=1
𝑛𝑛→∞

The crucial step is computing ∆𝑆𝑆𝑘𝑘 , the area of the 𝑘𝑘𝑘𝑘ℎ patch 𝑆𝑆𝑘𝑘 . Fig. 5 shows the patch 𝑆𝑆𝑘𝑘 and the
point 𝑃𝑃(𝑥𝑥𝑘𝑘 , 𝑦𝑦𝑘𝑘 , 𝑧𝑧𝑘𝑘 ). Two special vectors are tangent to the surface at 𝑃𝑃.

• 𝑡𝑡⃑𝑢𝑢 is a vector tangent to the surface corresponding to a change in 𝑢𝑢 with 𝑣𝑣 constant in the
𝑢𝑢𝑢𝑢-plane.
• 𝑡𝑡⃑𝑣𝑣 is a vector tangent to the surface corresponding to a change in 𝑣𝑣 with 𝑢𝑢 constant in the
𝑢𝑢𝑢𝑢-plane.

Because the surface 𝑆𝑆 may be written as 𝑟𝑟⃑(𝑢𝑢, 𝑣𝑣 ) = 〈𝑥𝑥(𝑢𝑢, 𝑣𝑣), 𝑦𝑦(𝑢𝑢, 𝑣𝑣), 𝑧𝑧(𝑢𝑢, 𝑣𝑣)〉, a tangent vector
corresponding to a change 𝑢𝑢 with 𝑣𝑣 fixed is

𝜕𝜕𝑟𝑟⃑ 𝜕𝜕𝜕𝜕 𝜕𝜕𝜕𝜕 𝜕𝜕𝜕𝜕


𝑡𝑡⃑𝑢𝑢 =
=〈 , , 〉
𝜕𝜕𝜕𝜕 𝜕𝜕𝜕𝜕 𝜕𝜕𝜕𝜕 𝜕𝜕𝜕𝜕
Similarly, a tangent vector corresponding to a change 𝑣𝑣 with 𝑢𝑢 fixed is

𝜕𝜕𝑟𝑟⃑ 𝜕𝜕𝜕𝜕 𝜕𝜕𝜕𝜕 𝜕𝜕𝜕𝜕


𝑡𝑡⃑𝑣𝑣 = =〈 , , 〉
𝜕𝜕𝜕𝜕 𝜕𝜕𝜕𝜕 𝜕𝜕𝜕𝜕 𝜕𝜕𝜕𝜕

Now consider an increment ∆𝑢𝑢 in 𝑢𝑢 with 𝑣𝑣 fixed. The tangent vector 𝑡𝑡⃑𝑢𝑢 ∆𝑢𝑢 forms one side of a
parallelogram (Fig. 5). Similarly, with an increment ∆𝑣𝑣 in 𝑣𝑣 with 𝑢𝑢 fixed, the tangent vector 𝑡𝑡⃑𝑣𝑣 ∆𝑣𝑣
forms the other side of that parallelogram. The area of this parallelogram is equal to the area of the
patch 𝑆𝑆𝑘𝑘 , which is ∆𝑆𝑆𝑘𝑘 , as ∆𝑆𝑆𝑘𝑘 approaches zero.

Applying the cross product, the area of the parallelogram is

�𝑡𝑡⃑𝑢𝑢 ∆𝑢𝑢 × 𝑡𝑡⃑𝑣𝑣 ∆𝑣𝑣� = �𝑡𝑡⃑𝑢𝑢 × 𝑡𝑡⃑𝑣𝑣 �∆𝑢𝑢∆𝑣𝑣 = ∆𝑆𝑆𝑘𝑘

Note that 𝑡𝑡⃑𝑢𝑢 × 𝑡𝑡⃑𝑣𝑣 is evaluated at (𝑢𝑢𝑘𝑘 , 𝑣𝑣𝑘𝑘 ) and is a vector normal to the surface at 𝑃𝑃, which we
assume to be nonzero at all points of 𝑆𝑆.

We write the Riemann sum with the observation that that the areas of the parallelograms
approximate the areas of the patches 𝑆𝑆𝑘𝑘 :

40
𝑛𝑛

� 𝜎𝜎(𝑥𝑥 (𝑢𝑢𝑘𝑘 , 𝑣𝑣𝑘𝑘 ), 𝑦𝑦(𝑢𝑢𝑘𝑘 , 𝑣𝑣𝑘𝑘 ), 𝑧𝑧(𝑢𝑢𝑘𝑘 , 𝑣𝑣𝑘𝑘 ))∆𝑆𝑆𝑘𝑘


𝑘𝑘=1
𝑛𝑛

≅ � 𝜎𝜎(𝑥𝑥 (𝑢𝑢𝑘𝑘 , 𝑣𝑣𝑘𝑘 ), 𝑦𝑦(𝑢𝑢𝑘𝑘 , 𝑣𝑣𝑘𝑘 ), 𝑧𝑧(𝑢𝑢𝑘𝑘 , 𝑣𝑣𝑘𝑘 ))�𝑡𝑡⃑𝑢𝑢 × 𝑡𝑡⃑𝑣𝑣 �∆𝑢𝑢∆𝑣𝑣
𝑘𝑘=1

Definition: Scalar Surface Integral

We now assume that 𝜎𝜎 is continuous on 𝑆𝑆. As ∆𝑢𝑢 and ∆𝑣𝑣 approach zero, the areas of the
parallelograms approach the areas of the corresponding patches on 𝑆𝑆. In this limit, the Riemann
sum approaches the surface integral of 𝜎𝜎 over the surface 𝑆𝑆, which we write as ∬𝑆𝑆 𝜎𝜎(𝑥𝑥, 𝑦𝑦, 𝑧𝑧)𝑑𝑑𝑑𝑑:

𝑛𝑛

lim � 𝜎𝜎(𝑥𝑥 (𝑢𝑢𝑘𝑘 , 𝑣𝑣𝑘𝑘 ), 𝑦𝑦(𝑢𝑢𝑘𝑘 , 𝑣𝑣𝑘𝑘 ), 𝑧𝑧(𝑢𝑢𝑘𝑘 , 𝑣𝑣𝑘𝑘 ))�𝑡𝑡⃑𝑢𝑢 × 𝑡𝑡⃑𝑣𝑣 �∆𝑢𝑢∆𝑣𝑣
∆𝑢𝑢,∆𝑣𝑣→0
𝑘𝑘=1

= � 𝜎𝜎 (𝑥𝑥(𝑢𝑢, 𝑣𝑣), 𝑦𝑦(𝑢𝑢, 𝑣𝑣), 𝑧𝑧(𝑢𝑢, 𝑣𝑣)) �𝑡𝑡⃑𝑢𝑢 × 𝑡𝑡⃑𝑣𝑣 �𝑑𝑑𝑑𝑑 = �𝜎𝜎 (𝑥𝑥, 𝑦𝑦, 𝑧𝑧)𝑑𝑑𝑑𝑑
𝑅𝑅 𝑆𝑆

The integral over 𝑆𝑆 is evaluated as an ordinary double integral over the region 𝑅𝑅 in the 𝑢𝑢𝑢𝑢-plane.

Notice, we have used the symbol 𝑑𝑑𝑑𝑑 to infer that the integration is over a plane with no orientation,
namely the 𝑢𝑢𝑢𝑢-plane. If 𝑅𝑅 is a rectangular region, as we have assumed, the double integral becomes
an iterated integral with respect to 𝑢𝑢 and 𝑣𝑣, with constant limits.

Fig. 5 Geometry of the area patch 𝑺𝑺𝒌𝒌

41
Alternatively, we can express the integral of 𝜎𝜎 over a surface, in explicit form, as follows:
∬𝑆𝑆 𝜎𝜎(𝑥𝑥, 𝑦𝑦, 𝑧𝑧(𝑥𝑥, 𝑦𝑦))𝑑𝑑𝑑𝑑 (𝑆𝑆 represents an open curved surface in 3d) or ∯𝑆𝑆 𝜎𝜎(𝑥𝑥, 𝑦𝑦, 𝑧𝑧(𝑥𝑥, 𝑦𝑦))𝑑𝑑𝑑𝑑 (𝑆𝑆
represents a closed surface). In this case, points on the curved surface 𝑆𝑆 are mapped to points on a
planar surface 𝑥𝑥𝑥𝑥. 𝑑𝑑𝑑𝑑 will be expressed in terms of 𝑥𝑥 and 𝑦𝑦 and the differential area 𝑑𝑑𝑑𝑑𝑑𝑑𝑑𝑑. This
alternative approach will be discussed shortly.

Resultant of integrating a functional over a surface:

Geometrical attribute: The integral ∬𝑆𝑆 𝑑𝑑𝑑𝑑 in 2d (𝑆𝑆 is a 2d planar surface) represents the total
surface area defined by 𝑆𝑆. The integral ∬𝑆𝑆 𝑑𝑑𝑑𝑑 (𝑆𝑆 representing an open curved surface in 3d) or
∯𝑆𝑆 𝑑𝑑𝑑𝑑 (𝑆𝑆 representing a closed surface in 3d) is the area of the curved surface defined by 𝑆𝑆.

The mathematical machinery required to calculate surface (𝑆𝑆) integrals such as


∬𝑆𝑆 𝜎𝜎(𝑥𝑥(𝑢𝑢, 𝑣𝑣), 𝑦𝑦(𝑢𝑢, 𝑣𝑣), 𝑧𝑧(𝑢𝑢, 𝑣𝑣))𝑑𝑑𝑑𝑑, and ∬𝑆𝑆 𝑭𝑭 ∙ 𝒏𝒏𝑺𝑺𝑑𝑑𝑑𝑑 is described next and is also described in Ch
17.6 of the textbook. These types of calculations are important later in the course when we look at
applications that relate to the physical world. We summarize, for your convenience, the main
equations for 𝒏𝒏𝑺𝑺 and 𝑑𝑑𝑑𝑑.

Computing 𝒏𝒏𝑺𝑺 and 𝑑𝑑𝑑𝑑

We will describe the method for computing the surface integral in two ways: parameterizing the
surface and using an explicit representation of the surface.

The first step is to determine an expression for the position vector that traces out the surface (open
or closed). All calculations required thereafter stem from this calculation as a starting point. For
the general case, the surface is parameterized using two variables, say 𝑢𝑢 and 𝑣𝑣. Then

• 𝒓𝒓 = 𝑟𝑟⃑ = 〈𝑥𝑥 (𝑢𝑢, 𝑣𝑣 ), 𝑦𝑦(𝑢𝑢, 𝑣𝑣 ), 𝑧𝑧(𝑢𝑢, 𝑣𝑣)〉 where 𝑢𝑢 and 𝑣𝑣 are parameterization variables that
sweep out the points on the surface and 𝑟𝑟⃑ is a position vector that points from the origin to
a particular point on the surface.

Next, we determine the vector 𝒏𝒏𝑺𝑺 and the differential surface area 𝑑𝑑𝑑𝑑

𝒓𝒓 ×𝒓𝒓 𝜕𝜕𝜕𝜕 𝜕𝜕𝜕𝜕 𝜕𝜕𝜕𝜕 𝜕𝜕𝜕𝜕 𝜕𝜕𝜕𝜕 𝜕𝜕𝜕𝜕


• 𝒏𝒏𝑺𝑺 = |𝒓𝒓𝑢𝑢 ×𝒓𝒓𝑣𝑣 | where 𝒓𝒓𝑢𝑢 = 𝒕𝒕𝑢𝑢 = 〈 , , 〉 𝒓𝒓𝑣𝑣 = 𝒕𝒕𝑣𝑣 = 〈 , , 〉
𝑢𝑢 𝑣𝑣 𝜕𝜕𝜕𝜕 𝜕𝜕𝜕𝜕 𝜕𝜕𝜕𝜕 𝜕𝜕𝜕𝜕 𝜕𝜕𝜕𝜕 𝜕𝜕𝜕𝜕
𝚤𝚤̂ 𝚥𝚥̂ 𝑘𝑘�
�𝜕𝜕𝜕𝜕 𝜕𝜕𝜕𝜕 𝜕𝜕𝜕𝜕 �
𝒕𝒕𝑢𝑢 × 𝒕𝒕𝑣𝑣 = 𝜕𝜕𝜕𝜕 𝜕𝜕𝜕𝜕 𝜕𝜕𝜕𝜕�

𝜕𝜕𝜕𝜕 𝜕𝜕𝜕𝜕 𝜕𝜕𝜕𝜕
𝜕𝜕𝜕𝜕 𝜕𝜕𝜕𝜕 𝜕𝜕𝜕𝜕 𝑑𝑑𝑑𝑑𝑑𝑑

42
• 𝑑𝑑𝑑𝑑 = |𝒕𝒕𝑢𝑢 × 𝒕𝒕𝑣𝑣 |𝑑𝑑𝑑𝑑𝑑𝑑𝑑𝑑

Note, we are henceforth using the symbols 𝒕𝒕𝑢𝑢 and 𝒕𝒕𝑣𝑣 to remain consistent with the book
terminology. Also, the symbol 𝒕𝒕 is akin to the first letter of the word tangent. 𝒕𝒕𝑢𝑢 and 𝒕𝒕𝑣𝑣 are two
lines that define the tangential plane at a given point on the surface.

Alternatively, if the surface is given in explicit form, that is 𝑧𝑧 = 𝑓𝑓(𝑥𝑥, 𝑦𝑦), then:

• 𝒓𝒓 = 𝑟𝑟⃑ = 〈𝑥𝑥, 𝑦𝑦, 𝑓𝑓(𝑥𝑥, 𝑦𝑦)〉 where 𝑥𝑥 and 𝑦𝑦 are the variables that sweep out the points on the
surface and 𝑟𝑟⃑ is a position vector that points from the origin to a particular point on the
surface.

Next, we determine the vector 𝒏𝒏𝑺𝑺 and the differential surface area 𝑑𝑑𝑑𝑑

𝒕𝒕𝑥𝑥 ×𝒕𝒕𝑦𝑦 𝜕𝜕𝜕𝜕 𝜕𝜕𝜕𝜕


• 𝒏𝒏𝑺𝑺 = �𝒕𝒕 where 𝒕𝒕𝑥𝑥 = 〈1,0, 〉 and 𝒕𝒕𝑦𝑦 = 〈0,1, 〉
𝑥𝑥 ×𝒕𝒕𝑦𝑦 � 𝜕𝜕𝜕𝜕 𝜕𝜕𝜕𝜕

𝚤𝚤̂ 𝚥𝚥̂ 𝑘𝑘�


� 𝜕𝜕𝜕𝜕 � 𝜕𝜕𝜕𝜕 𝜕𝜕𝜕𝜕
𝒕𝒕𝑥𝑥 × 𝒕𝒕𝑦𝑦 = 1 0 𝜕𝜕𝜕𝜕 = 〈− , − , 1〉
� 𝜕𝜕𝜕𝜕 𝜕𝜕𝜕𝜕
𝜕𝜕𝜕𝜕 �
0 1
𝜕𝜕𝜕𝜕 𝑑𝑑𝑑𝑑𝑑𝑑
𝜕𝜕𝜕𝜕 2 𝜕𝜕𝜕𝜕 2
• 𝑑𝑑𝑑𝑑 = �𝒕𝒕𝑥𝑥 × 𝒕𝒕𝑦𝑦 �𝑑𝑑𝑑𝑑𝑑𝑑𝑑𝑑 = �� � + � � + 1 𝑑𝑑𝑑𝑑𝑑𝑑𝑑𝑑
𝜕𝜕𝜕𝜕 𝜕𝜕𝜕𝜕

1.6.4 Computing Flux

Consider a 3d vector field 𝑭𝑭 = 〈𝑔𝑔(𝑥𝑥, 𝑦𝑦, 𝑧𝑧), ℎ(𝑥𝑥, 𝑦𝑦, 𝑧𝑧), 𝑖𝑖(𝑥𝑥, 𝑦𝑦, 𝑧𝑧)〉. We can express the flux in the
following equivalent forms: ∬ 𝑭𝑭 ∙ 𝒏𝒏𝑺𝑺 𝑑𝑑𝑑𝑑 = ∬ 𝑭𝑭 ∙ 𝑑𝑑𝑑𝑑 ����⃑ = ∬ 𝑭𝑭 ∙ 𝒅𝒅𝒅𝒅.
𝑆𝑆 𝑆𝑆 𝑆𝑆

Surface parameterization:

∬𝑆𝑆 𝑭𝑭 ∙ 𝒏𝒏𝑺𝑺 𝑑𝑑𝑑𝑑 =


∬𝑅𝑅〈𝑔𝑔(𝑥𝑥(𝑢𝑢, 𝑣𝑣 ), 𝑦𝑦(𝑢𝑢, 𝑣𝑣 ), 𝑧𝑧(𝑢𝑢, 𝑣𝑣 )), ℎ(𝑥𝑥(𝑢𝑢, 𝑣𝑣), 𝑦𝑦(𝑢𝑢, 𝑣𝑣), 𝑧𝑧(𝑢𝑢, 𝑣𝑣)), 𝑖𝑖(𝑥𝑥(𝑢𝑢, 𝑣𝑣), 𝑦𝑦(𝑢𝑢, 𝑣𝑣), 𝑧𝑧(𝑢𝑢, 𝑣𝑣))〉 ∙ 𝒏𝒏𝑺𝑺 𝑑𝑑𝑑𝑑

𝒏𝒏𝑺𝑺𝑑𝑑𝑑𝑑 = 𝒕𝒕𝑢𝑢 × 𝒕𝒕𝑣𝑣 𝑑𝑑𝑑𝑑𝑑𝑑𝑑𝑑

𝒕𝒕 ×𝒕𝒕
where 𝒏𝒏𝑺𝑺 is the unit vector perpendicular to the differential surface area and is equivalent to |𝒕𝒕𝑢𝑢 ×𝒕𝒕𝑣𝑣 |,
𝑢𝑢 𝑣𝑣
and 𝑑𝑑𝑑𝑑 is the magnitude of the differential surface area, which is equivalent to |𝒕𝒕𝑢𝑢 × 𝒕𝒕𝑣𝑣 |𝑑𝑑𝑑𝑑𝑑𝑑𝑑𝑑.

43
Explicit representation of the surface:

If we use an explicit representation of the surface, then the differential surface area is in terms of
the parameters 𝑥𝑥 and 𝑦𝑦 and 𝑑𝑑𝑑𝑑 and 𝑑𝑑𝑑𝑑. The 𝑧𝑧 component of the surface is expressed as a function
of 𝑥𝑥 and 𝑦𝑦. In summary,

�𝑭𝑭 ∙ 𝒏𝒏𝑺𝑺𝑑𝑑𝑑𝑑 = � 〈𝑔𝑔(𝑥𝑥, 𝑦𝑦, 𝑓𝑓(𝑥𝑥, 𝑦𝑦)), ℎ�𝑥𝑥, 𝑦𝑦, 𝑓𝑓 (𝑥𝑥, 𝑦𝑦)�, 𝑖𝑖(𝑥𝑥, 𝑦𝑦, 𝑓𝑓(𝑥𝑥, 𝑦𝑦))〉 ∙ 𝒕𝒕𝑥𝑥 × 𝒕𝒕𝑦𝑦 𝑑𝑑𝑑𝑑𝑑𝑑𝑑𝑑
𝑆𝑆 𝑅𝑅

We will be working generally with simple objects for calculation purposes. Table 7 shows the
essential relationships for the explicit and parametric descriptions of cylinders, cones, spheres, and
paraboloids. The listed normal vectors are chosen to point away from the 𝑧𝑧-axis. You are not
required to memorize these formulae, but you can use the table to assist you in your understanding
of how to formulate the expressions for different types of surfaces.

44
Table 7 Summary of Essential Relationships for the Explicit and Parametric Descriptions of Common Objects

Explicit Description 𝒛𝒛 = 𝒇𝒇(𝒙𝒙, 𝒚𝒚), 𝒙𝒙 = 𝒇𝒇(𝒚𝒚, 𝒛𝒛) or 𝒚𝒚 = 𝒇𝒇(𝒛𝒛, 𝒙𝒙)

Surface Equations Describing the Surface Normal to the Surface ±(𝒕𝒕𝑥𝑥 × 𝒕𝒕𝑦𝑦 )
assumimg 𝒛𝒛 = 𝒇𝒇(𝒙𝒙, 𝒚𝒚)
𝒕𝒕𝑥𝑥 = 〈1,0, 𝑧𝑧𝑥𝑥 〉 𝒕𝒕𝑦𝑦 = 〈0,1, 𝑧𝑧𝑦𝑦 〉
±�𝒕𝒕𝑥𝑥 × 𝒕𝒕𝑦𝑦 � = ±〈−𝑧𝑧𝑥𝑥 , −𝑧𝑧𝑦𝑦 , 1〉
Cylinder of height ℎ and radius 𝑎𝑎 aligned 𝒓𝒓 = ⟨𝑥𝑥, 𝑦𝑦(𝑧𝑧, 𝑥𝑥 ), 𝑧𝑧⟩ right half
with the 𝑧𝑧 axis. Normal surface vector is 𝑥𝑥
pointing radially outwards from the right half 𝒓𝒓 = 〈𝑥𝑥, √𝑎𝑎2 − 𝑥𝑥 2 , 𝑧𝑧〉 𝒕𝒕𝑥𝑥 × 𝒕𝒕𝑧𝑧 = 〈 , 1, 0 〉
√𝑎𝑎2 − 𝑥𝑥2
cylindrical surface. left half 𝒓𝒓 = 〈𝑥𝑥, −√𝑎𝑎2 − 𝑥𝑥 2 , 𝑧𝑧〉 left half
𝑥𝑥
−𝑎𝑎 ≤ 𝑥𝑥 ≤ 𝑎𝑎 𝒕𝒕𝑥𝑥 × 𝒕𝒕𝑧𝑧 = 〈 , −1, 0 〉
0 ≤ 𝑧𝑧 ≤ ℎ
√𝑎𝑎2 − 𝑥𝑥2
|𝒕𝒕𝑥𝑥 × 𝒕𝒕𝑧𝑧 | = 𝑎𝑎
Note: in this case we use 𝒕𝒕𝒙𝒙 × 𝒕𝒕𝒛𝒛 instead of 𝒕𝒕𝒙𝒙 × 𝒕𝒕𝒚𝒚
Lateral surface of an upside-down cone 𝒓𝒓 = ⟨𝑥𝑥, y, 𝑧𝑧(𝑥𝑥, 𝑦𝑦)⟩ 𝑥𝑥 𝑦𝑦
with 45 degree angle with respect to the 𝑧𝑧
−�𝒕𝒕𝑥𝑥 × 𝒕𝒕𝑦𝑦 � = 〈 , ,−1〉
�𝑥𝑥2 + 𝑦𝑦2 �𝑥𝑥2 + 𝑦𝑦2
axis and height ℎ along 𝑧𝑧 axis, apex at the 𝒓𝒓 = 〈𝑥𝑥, 𝑦𝑦, �𝑥𝑥 2 + 𝑦𝑦 2 〉
origin and 𝑧𝑧 component of the normal to �𝒕𝒕𝑥𝑥 × 𝒕𝒕𝑦𝑦 � = √2
−�ℎ2 − 𝑥𝑥 2 ≤ 𝑦𝑦 ≤ �ℎ2 − 𝑥𝑥 2
the lateral surface is negative. −ℎ ≤ 𝑥𝑥 ≤ ℎ
Sphere of radius 𝑎𝑎 and outward normal. 𝒓𝒓 = ⟨𝑥𝑥, y, 𝑧𝑧(𝑥𝑥, 𝑦𝑦)⟩ upper hemisphere
𝑥𝑥 𝑦𝑦
upper hemisphere 𝒓𝒓 = 〈𝑥𝑥, 𝑦𝑦, �𝑎𝑎2 − 𝑥𝑥 2 − 𝑦𝑦 2 〉
𝒕𝒕𝑥𝑥 × 𝒕𝒕𝑦𝑦 = 〈 , ,1〉
�𝑎𝑎2 − 𝑥𝑥2 − 𝑦𝑦2 �𝑎𝑎2 − 𝑥𝑥2 − 𝑦𝑦2
lower hemisphere 𝒓𝒓 = 〈𝑥𝑥, 𝑦𝑦, −�𝑎𝑎2 − 𝑥𝑥 2 − 𝑦𝑦 2 〉 lower hemisphere
−√𝑎𝑎2 − 𝑥𝑥 2 ≤ 𝑦𝑦 ≤ √𝑎𝑎2 − 𝑥𝑥 2 𝑥𝑥 𝑦𝑦
−𝑎𝑎 ≤ 𝑥𝑥 ≤ 𝑎𝑎 −�𝒕𝒕𝑥𝑥 × 𝒕𝒕𝑦𝑦 � = 〈 , , −1〉
�𝑎𝑎2 − 𝑥𝑥2 − 𝑦𝑦2 �𝑎𝑎2 − 𝑥𝑥2 − 𝑦𝑦2
𝑎𝑎
�𝒕𝒕𝑥𝑥 × 𝒕𝒕𝑦𝑦 � =
�𝑎𝑎 − 𝑥𝑥 2 − 𝑦𝑦 2
2

Paraboloid with height ℎ along the 𝑧𝑧 axis, 𝒓𝒓 = ⟨𝑥𝑥, y, 𝑧𝑧(𝑥𝑥, 𝑦𝑦)⟩ −�𝒕𝒕𝑥𝑥 × 𝒕𝒕𝑦𝑦 � = 〈2𝑥𝑥, 2𝑦𝑦, − 1 〉
projected radius √ℎ on the 𝑥𝑥𝑥𝑥 plane and
𝒓𝒓 = 〈𝑥𝑥, 𝑦𝑦, 𝑥𝑥 2 + 𝑦𝑦 2 〉 �𝒕𝒕𝑥𝑥 × 𝒕𝒕𝑦𝑦 � = �1 + 4(𝑥𝑥 2 + 𝑦𝑦 2 )
𝑧𝑧 component of the normal surface vector
is negative. −√ℎ − 𝑥𝑥 2 ≤ 𝑦𝑦 ≤ √ℎ − 𝑥𝑥 2
−√ℎ ≤ 𝑥𝑥 ≤ √ℎ

45
Parametric Description 𝒙𝒙(𝒖𝒖, 𝒗𝒗), 𝒚𝒚(𝒖𝒖, 𝒗𝒗), 𝒛𝒛(𝒖𝒖, 𝒗𝒗)

Surface Equations Describing the Surface Normal to the Surface ±(𝑡𝑡⃑𝑢𝑢 × 𝑡𝑡⃑𝑣𝑣 )
𝚤𝚤̂ 𝚥𝚥̂ 𝑘𝑘�
� 𝜕𝜕𝜕𝜕 𝜕𝜕𝜕𝜕 𝜕𝜕𝜕𝜕 �
±�𝑡𝑡⃑𝑢𝑢 × 𝑡𝑡⃑𝑣𝑣 � = ± 𝜕𝜕𝜕𝜕 𝜕𝜕𝜕𝜕 𝜕𝜕𝜕𝜕
� �
𝜕𝜕𝜕𝜕 𝜕𝜕𝜕𝜕 𝜕𝜕𝜕𝜕
𝜕𝜕𝜕𝜕 𝜕𝜕𝜕𝜕 𝜕𝜕𝜕𝜕 𝑑𝑑𝑑𝑑𝑑𝑑

Cylinder of height ℎ aligned with 𝑧𝑧 axis 𝒓𝒓 = 〈𝑎𝑎𝑎𝑎𝑎𝑎𝑎𝑎𝑎𝑎, 𝑎𝑎𝑎𝑎𝑎𝑎𝑎𝑎𝑎𝑎, 𝑣𝑣〉 𝑡𝑡⃑𝑢𝑢 × 𝑡𝑡⃑𝑣𝑣 = 〈𝑎𝑎𝑎𝑎𝑎𝑎𝑎𝑎𝑎𝑎, 𝑎𝑎𝑎𝑎𝑎𝑎𝑎𝑎𝑎𝑎, 0〉,
(radius 𝑎𝑎) and normal surface vector is 0 ≤ 𝑢𝑢 ≤ 2𝜋𝜋 0 ≤ 𝑣𝑣 ≤ ℎ �𝑡𝑡⃑𝑢𝑢 × 𝑡𝑡⃑𝑣𝑣 � = 𝑎𝑎
pointing radially outwards from the
cylindrical surface.
Lateral surface of an upside-down cone 𝒓𝒓 = 〈𝑣𝑣𝑣𝑣𝑣𝑣𝑣𝑣𝑣𝑣, 𝑣𝑣𝑣𝑣𝑣𝑣𝑣𝑣𝑣𝑣, 𝑣𝑣〉 𝑡𝑡⃑𝑢𝑢 × 𝑡𝑡⃑𝑣𝑣 = 〈𝑣𝑣𝑣𝑣𝑣𝑣𝑣𝑣𝑣𝑣, 𝑣𝑣𝑣𝑣𝑣𝑣𝑣𝑣𝑣𝑣, −𝑣𝑣〉,
with 45 degree angle with respect to the 𝑧𝑧 0 ≤ 𝑢𝑢 ≤ 2𝜋𝜋 0 ≤ 𝑣𝑣 ≤ ℎ �𝑡𝑡⃑𝑢𝑢 × 𝑡𝑡⃑𝑣𝑣 � = √2𝑣𝑣
axis and height ℎ along 𝑧𝑧 axis, apex at the
origin and 𝑧𝑧 component of the normal to
the lateral surface is negative.
Sphere of radius 𝑎𝑎 and outward normal. 𝒓𝒓 = 〈𝑎𝑎𝑎𝑎𝑎𝑎𝑎𝑎𝑎𝑎𝑎𝑎𝑎𝑎𝑎𝑎𝑎𝑎, 𝑎𝑎𝑎𝑎𝑎𝑎𝑎𝑎𝑎𝑎𝑎𝑎𝑎𝑎𝑎𝑎𝑎𝑎, 𝑎𝑎𝑎𝑎𝑎𝑎𝑎𝑎𝑎𝑎〉 𝑡𝑡⃑𝑢𝑢 × 𝑡𝑡⃑𝑣𝑣 = 〈𝑎𝑎2 𝑠𝑠𝑠𝑠𝑠𝑠2 𝑢𝑢𝑢𝑢𝑢𝑢𝑢𝑢𝑢𝑢, 𝑎𝑎2 𝑠𝑠𝑠𝑠𝑠𝑠2 𝑢𝑢𝑢𝑢𝑢𝑢𝑢𝑢𝑢𝑢, 𝑎𝑎2 𝑠𝑠𝑠𝑠𝑠𝑠𝑠𝑠𝑠𝑠𝑠𝑠𝑠𝑠𝑠𝑠〉,
0 ≤ 𝑢𝑢 ≤ 𝜋𝜋 0 ≤ 𝑣𝑣 ≤ 2𝜋𝜋 �𝑡𝑡⃑𝑢𝑢 × 𝑡𝑡⃑𝑣𝑣 � = 𝑎𝑎2 𝑠𝑠𝑠𝑠𝑠𝑠𝑠𝑠

Paraboloid with height ℎ along the 𝑧𝑧 axis, 𝒓𝒓 = 〈𝑣𝑣𝑣𝑣𝑣𝑣𝑣𝑣𝑣𝑣, 𝑣𝑣𝑣𝑣𝑣𝑣𝑣𝑣𝑣𝑣, 𝑣𝑣 2 〉 𝑡𝑡⃑𝑢𝑢 × 𝑡𝑡⃑𝑣𝑣 = 〈2𝑣𝑣 2 𝑐𝑐𝑐𝑐𝑐𝑐𝑐𝑐, 2𝑣𝑣 2 𝑠𝑠𝑠𝑠𝑠𝑠𝑠𝑠, −𝑣𝑣〉,
projected radius √ℎ on the 𝑥𝑥𝑥𝑥 plane and 0 ≤ 𝑢𝑢 ≤ 2𝜋𝜋 0 ≤ 𝑣𝑣 ≤ √ℎ �𝑡𝑡⃑𝑢𝑢 × 𝑡𝑡⃑𝑣𝑣 � = 𝑣𝑣 �1 + 4𝑣𝑣 2
𝑧𝑧 component of the normal surface vector
is negative.

46
1.6.5 Example Problems

1. Derive the expressions describing the position vector for the surface and the differential
surface area vector (𝒏𝒏𝒔𝒔 𝑑𝑑𝑑𝑑 = 𝒕𝒕𝑥𝑥 × 𝒕𝒕𝑦𝑦 𝑑𝑑𝑑𝑑𝑑𝑑𝑑𝑑 = 𝒕𝒕𝑢𝑢 × 𝒕𝒕𝑣𝑣 𝑑𝑑𝑑𝑑𝑑𝑑𝑑𝑑), for the following geometries
in explicit and parametric form, and then setup the double integrals for computing the
surface area for the explicit and parametric cases:

a. Cylinder aligned with z axis (radius a) and the surface normal vector is pointing
radially outwards from the cylindrical surface.
b. Lateral surface of an upside-down cone with 45-degree angle with respect to the
𝑧𝑧 axis and height ℎ along 𝑧𝑧 axis, apex at the origin and 𝑧𝑧 component of the normal
to the lateral surface is negative.
c. Sphere of radius 𝑎𝑎 and outward normal.
d. Paraboloid with height ℎ along the 𝑧𝑧 axis, projected radius √ℎ on the 𝑥𝑥𝑥𝑥 plane and
𝑧𝑧 component of the normal surface vector is negative.

Solution:

The solutions for the equations describing the position vector for the surface and the
differential surface area vector normal to the surface are presented in Table 3. The double
integrals for each case are given below.

Surface Explicit Parametric

Cylinder aligned with ℎ 𝑎𝑎


𝑎𝑎 ℎ 2𝜋𝜋

𝑧𝑧 axis (radius 𝑎𝑎) and 2 � � 𝑑𝑑𝑑𝑑𝑑𝑑𝑑𝑑 � � 𝑎𝑎𝑎𝑎𝑎𝑎𝑎𝑎𝑎𝑎


0 −𝑎𝑎 √𝑎𝑎 − 𝑥𝑥
2 2
0 0
the surface normal Note: factor 2 is due to
vector is pointing contribution of the left and right 𝑢𝑢 = 𝜃𝜃, 𝑣𝑣 = 𝑧𝑧
radially outwards side of the cylinder’s surface
from the cylindrical
surface.
Lateral surface of an ℎ √ℎ2 −𝑥𝑥 2 ℎ 2𝜋𝜋

upside-down cone � � √2𝑑𝑑𝑑𝑑𝑑𝑑𝑑𝑑 � � √2𝑣𝑣𝑑𝑑𝑑𝑑𝑑𝑑𝑑𝑑


2 −𝑥𝑥 2 0 0
with 45 degree angle −ℎ −√ℎ

with respect to the 𝑧𝑧 𝑢𝑢 = 𝜃𝜃, 𝑣𝑣 = 𝑟𝑟


axis and height ℎ
along 𝑧𝑧 axis, apex at
the origin and 𝑧𝑧
component of the
normal to the lateral
surface is negative

47
2𝜋𝜋 𝜋𝜋
Sphere of radius 𝑎𝑎 and 𝑎𝑎 √𝑎𝑎2 −𝑥𝑥2
𝑎𝑎
outward normal 2� � 𝑑𝑑𝑑𝑑𝑑𝑑𝑑𝑑 � � 𝑎𝑎2 𝑠𝑠𝑠𝑠𝑠𝑠𝑠𝑠𝑠𝑠𝑠𝑠𝑠𝑠𝑠𝑠
−𝑎𝑎 −√𝑎𝑎2 −𝑥𝑥2 �𝑎𝑎 − 𝑥𝑥 − 𝑦𝑦
2 2 2 0 0

Note: factor 2 is due to


contribution of top and bottom 𝑢𝑢 = 𝜑𝜑, 𝑣𝑣 = 𝜃𝜃
surface
Paraboloid with √ℎ √ℎ−𝑥𝑥2 √ℎ 2𝜋𝜋
height ℎ along the 𝑧𝑧 � � �1 + 4(𝑥𝑥2 + 𝑦𝑦2 )𝑑𝑑𝑑𝑑𝑑𝑑𝑑𝑑 � � 𝑣𝑣�1 + 4𝑣𝑣 2 𝑑𝑑𝑑𝑑𝑑𝑑𝑑𝑑
0 0
axis, projected radius − ℎ −√ 2
√ ℎ−𝑥𝑥

√ℎ on the 𝑥𝑥𝑥𝑥 plane 𝑢𝑢 = 𝜃𝜃, 𝑣𝑣 = 𝑟𝑟


and 𝑧𝑧 component of
the normal surface
vector is negative

2. Find the surface area of the cylinder {(𝑟𝑟, 𝜃𝜃: 𝑟𝑟 = 4, 0 ≤ 𝜃𝜃 ≤ 2𝜋𝜋)} between the planes 𝑧𝑧 = 0
and 𝑧𝑧 = 16 − 2𝑥𝑥.

Solution

The figure on the left below shows the cylinder bounded by the two planes: 𝑧𝑧 = 0 and 𝑧𝑧 =
16 − 2𝑥𝑥.

Fig. Area of the exterior surface of a sliced cylinder

48
With 𝑢𝑢 = 𝜃𝜃 and 𝑣𝑣 = 𝑧𝑧, a parametric description of the cylinder is:

𝑟𝑟(𝑢𝑢, 𝑣𝑣 ) = 〈𝑥𝑥(𝑢𝑢, 𝑣𝑣 ), 𝑦𝑦(𝑢𝑢, 𝑣𝑣 ), 𝑧𝑧(𝑢𝑢, 𝑣𝑣)〉 = 〈4𝑐𝑐𝑐𝑐𝑐𝑐𝑐𝑐, 4𝑠𝑠𝑠𝑠𝑠𝑠𝑠𝑠, 𝑣𝑣 〉

The challenge is finding the limits on 𝑣𝑣 which is the 𝑧𝑧-coordinate. The plane 𝑧𝑧 = 16 − 2𝑥𝑥
intersects the cylinder in an ellipse: along this ellipse, as 𝑢𝑢 varies, between 0 and 2𝜋𝜋, the
parameter 𝑣𝑣 also changes. To find the relationship between 𝑢𝑢 and 𝑣𝑣 along this intersection
curve, notice that at any point on the cylinder, we have 𝑥𝑥 = 4𝑐𝑐𝑐𝑐𝑐𝑐𝑐𝑐 (remember that 𝑢𝑢 = 𝜃𝜃).
Making this substitution in the equation of the plane, we have

𝑧𝑧 = 16 − 2𝑥𝑥 = 16 − 2(4𝑐𝑐𝑐𝑐𝑐𝑐𝑐𝑐) = 16 − 8𝑐𝑐𝑐𝑐𝑐𝑐𝑢𝑢

Substituting 𝑣𝑣 = 𝑧𝑧, the relationship between 𝑢𝑢 and 𝑣𝑣 is 𝑣𝑣 = 16 − 8𝑐𝑐𝑐𝑐𝑐𝑐𝑐𝑐, as shown in the


right-hand figure in Fig. Ex 3. Therefore, the region of integration in the 𝑢𝑢𝑢𝑢-plane is

𝑅𝑅 = {(𝑢𝑢, 𝑣𝑣 ): 0 ≤ 𝑢𝑢 ≤ 2𝜋𝜋, 0 ≤ 𝑣𝑣 ≤ 16 − 8𝑐𝑐𝑐𝑐𝑐𝑐𝑐𝑐 }

From Table 3, �𝑡𝑡⃑𝑢𝑢 × 𝑡𝑡⃑𝑣𝑣 � = 𝑎𝑎 = 4. The surface integral for the area is thus

2𝜋𝜋 16−8𝑐𝑐𝑐𝑐𝑐𝑐𝑐𝑐 2𝜋𝜋


�𝑑𝑑𝑑𝑑 = � �𝑡𝑡⃑𝑢𝑢 × 𝑡𝑡⃑𝑣𝑣 �𝑑𝑑𝑑𝑑 = � 4𝑑𝑑𝑑𝑑 = � � 4𝑑𝑑𝑑𝑑𝑑𝑑𝑑𝑑 = 4 � (16 − 8𝑐𝑐𝑐𝑐𝑐𝑐𝑐𝑐) 𝑑𝑑𝑑𝑑
𝑆𝑆 𝑅𝑅 𝑅𝑅 0 0 0
= 4(16𝑢𝑢 − 8𝑠𝑠𝑠𝑠𝑠𝑠𝑠𝑠)|2𝜋𝜋
0 = 128𝜋𝜋

3. The temperature on the surface of a sphere of radius 𝑎𝑎 varies with latitude according to the
function 𝑇𝑇(𝜑𝜑, 𝜃𝜃 ) = 10 + 50𝑠𝑠𝑠𝑠𝑠𝑠𝑠𝑠 𝐶𝐶 𝑜𝑜 , for 0 ≤ 𝜑𝜑 ≤ 𝜋𝜋 and 0 ≤ 𝜃𝜃 ≤ 2𝜋𝜋 (𝜑𝜑 and 𝜃𝜃 are
spherical coordinates, so the temperature is 10 𝐶𝐶 0 at the poles, increasing to 60 𝐶𝐶 𝑜𝑜 at the
equator). Find the average temperature over the sphere.

Solution

The average temperature is defined by:

∬𝑆𝑆 𝑇𝑇(𝑥𝑥, 𝑦𝑦, 𝑧𝑧)𝑑𝑑𝑑𝑑


𝑇𝑇𝑎𝑎𝑎𝑎𝑎𝑎 =
∬𝑆𝑆 𝑑𝑑𝑑𝑑

Hence, we need to compute two integrals. We use the parametric description of the sphere.
With 𝑢𝑢 = 𝜑𝜑 and 𝑣𝑣 = 𝜃𝜃, the temperature function becomes 𝑇𝑇(𝑢𝑢, 𝑣𝑣 ) = 10 + 50𝑠𝑠𝑖𝑖𝑖𝑖𝑖𝑖.

49
Integrating the temperature over the sphere using the fact that 𝑑𝑑𝑑𝑑 = |𝒕𝒕𝑢𝑢 × 𝒕𝒕𝑣𝑣 |𝑑𝑑𝑑𝑑𝑑𝑑𝑑𝑑 =
𝑎𝑎2 𝑠𝑠𝑠𝑠𝑠𝑠𝑠𝑠𝑠𝑠𝑠𝑠𝑠𝑠𝑠𝑠, we have:

𝜋𝜋 2𝜋𝜋
�𝑇𝑇(𝑥𝑥, 𝑦𝑦, 𝑧𝑧)𝑑𝑑𝑑𝑑 = � 𝑇𝑇(𝑢𝑢, 𝑣𝑣 )|𝒕𝒕𝑢𝑢 × 𝒕𝒕𝑣𝑣 |𝑑𝑑𝑑𝑑𝑑𝑑𝑑𝑑 = � � (10 + 50𝑠𝑠𝑠𝑠𝑠𝑠𝑠𝑠)𝑎𝑎2 𝑠𝑠𝑠𝑠𝑠𝑠𝑠𝑠𝑠𝑠𝑠𝑠𝑠𝑠𝑠𝑠
𝑆𝑆 𝑅𝑅 0 0
𝜋𝜋
= 2𝜋𝜋𝑎𝑎2 � (10 + 50𝑠𝑠𝑠𝑠𝑠𝑠𝑠𝑠)𝑠𝑠𝑠𝑠𝑠𝑠𝑠𝑠𝑠𝑠𝑠𝑠 = 10𝜋𝜋𝑎𝑎2 (4 + 5𝜋𝜋)
0
The surface area is

𝜋𝜋 2𝜋𝜋
�𝑑𝑑𝑑𝑑 = � |𝒕𝒕𝑢𝑢 × 𝒕𝒕𝑣𝑣 |𝑑𝑑𝑑𝑑𝑑𝑑𝑑𝑑 = � � 𝑎𝑎2 𝑠𝑠𝑠𝑠𝑠𝑠𝑠𝑠𝑠𝑠𝑠𝑠𝑠𝑠𝑠𝑠 = 4𝜋𝜋𝑎𝑎2
𝑆𝑆 𝑅𝑅 0 0

Therefore,

∬𝑆𝑆 𝑇𝑇(𝑥𝑥, 𝑦𝑦, 𝑧𝑧)𝑑𝑑𝑑𝑑 10𝜋𝜋𝑎𝑎2 (4 + 5𝜋𝜋) 5


𝑇𝑇𝑎𝑎𝑎𝑎𝑎𝑎 = = = (4 + 5𝜋𝜋) ≅ 49.3 𝐶𝐶 0 .
∬𝑆𝑆 𝑑𝑑𝑑𝑑 4𝜋𝜋𝑎𝑎2 2

4. A paraboloid reflector with radius 𝑅𝑅 intercepts light of a particular wavelength that is


beamed in the −𝑧𝑧̂ direction. The paraboloid is defined by the following equation: 𝑧𝑧 =
𝑎𝑎(𝑥𝑥 2 + 𝑦𝑦 2 ) where 𝑎𝑎 is a constant. The irradiance (radiation flux density) is given by 𝑰𝑰 =
𝑊𝑊
−𝐼𝐼𝑜𝑜 𝑧𝑧̂ and has units of . Compute the radiation flux intercepted by the paraboloid
𝑚𝑚2
reflector.

Solution

We will use an explicit representation of the surface in which case,

�𝑰𝑰 ∙ 𝒏𝒏𝑺𝑺𝑑𝑑𝑑𝑑 = � −𝐼𝐼𝑜𝑜 〈0,0,1〉 ∙ 𝒕𝒕𝑥𝑥 × 𝒕𝒕𝑦𝑦 𝑑𝑑𝑑𝑑𝑑𝑑𝑑𝑑


𝑆𝑆 𝑅𝑅
and
𝜕𝜕𝜕𝜕 𝜕𝜕𝜕𝜕
𝒕𝒕𝑥𝑥 × 𝒕𝒕𝑦𝑦 = 〈− , − , 1〉
𝜕𝜕𝜕𝜕 𝜕𝜕𝜕𝜕

We are interested in a flux that is downwards, so we need to use −𝒕𝒕𝑥𝑥 × 𝒕𝒕𝑦𝑦


𝜕𝜕𝜕𝜕 𝜕𝜕𝜕𝜕
−𝒕𝒕𝑥𝑥 × 𝒕𝒕𝑦𝑦 = 〈 , , −1〉
𝜕𝜕𝜕𝜕 𝜕𝜕𝜕𝜕
𝜕𝜕𝜕𝜕 𝜕𝜕𝜕𝜕
and = 2𝑎𝑎𝑎𝑎 and = 2𝑎𝑎𝑎𝑎 and the region is defined by 𝑥𝑥 2 + 𝑦𝑦 2 ≤ 𝑅𝑅2 . Therefore, the
𝜕𝜕𝜕𝜕 𝜕𝜕𝜕𝜕
integral becomes:

50
𝑅𝑅 √𝑅𝑅2 −𝑥𝑥 2 𝑅𝑅 √𝑅𝑅2 −𝑥𝑥 2
� � −𝐼𝐼𝑜𝑜 〈0,0,1〉 ∙ 〈2𝑎𝑎𝑎𝑎, 2𝑎𝑎𝑦𝑦, −1〉𝑑𝑑𝑑𝑑𝑑𝑑𝑑𝑑 = � � 𝐼𝐼𝑜𝑜 𝑑𝑑𝑑𝑑𝑑𝑑𝑑𝑑
−𝑅𝑅 −√𝑅𝑅2 −𝑥𝑥 2 −𝑅𝑅 −√𝑅𝑅2 −𝑥𝑥 2

The bounds on the double integral represent a circular region with radius 𝑅𝑅. It is easier to
compute the integral in polar coordinates and therefore the integral becomes:

2𝜋𝜋 𝑅𝑅
� � 𝐼𝐼𝑜𝑜 𝑟𝑟𝑟𝑟𝑟𝑟𝑟𝑟𝑟𝑟 = 𝜋𝜋𝑅𝑅2 𝐼𝐼𝑜𝑜 Watts
0 0

At this point you can work on the following problems which have been extracted from section 17.6
of the textbook:

1.6.6 Problems from Section 17.6: 19,23,27,32,37,41,43,45,53,55,56,69,71

6. (question 19) Surface area using a parametric description Find the area of the following
surface using a parametric description of the surface: The half-cylinder {(𝑟𝑟, 𝜃𝜃, 𝑧𝑧): 𝑟𝑟 = 4,
0 ≤ 𝜃𝜃 ≤ 𝜋𝜋, 0 ≤ 𝑧𝑧 ≤ 7}.
7. (question 23) Surface area using a parametric description Find the area of the following
surface using a parametric description of the surface: A cone with base radius 𝑟𝑟 and height
ℎ, where 𝑟𝑟 and ℎ are positive constants.
8. (question 27) Surface integrals using a parametric description Evaluate the surface
integral ∬𝑆𝑆 𝑓𝑓𝑓𝑓𝑓𝑓 using a parametric description of the surface: 𝑓𝑓 (𝑥𝑥, 𝑦𝑦, 𝑧𝑧) = 𝑥𝑥 where S is the
cylinder 𝑥𝑥 2 + 𝑧𝑧 2 = 1, 0 ≤ 𝑦𝑦 ≤ 3.
9. (question 32) Surface area using an explicit description Find the area of the following
1
surface using an explicit representation of the surface: The trough 𝑧𝑧 = 𝑥𝑥 2 , for −1 ≤ 𝑥𝑥 ≤
2
1, 0 ≤ 𝑦𝑦 ≤ 4.
10. (question 37) Surface integral using an explicit description Evaluate the surface integral
∬𝑆𝑆 𝑓𝑓(𝑥𝑥, 𝑦𝑦, 𝑧𝑧)𝑑𝑑𝑑𝑑 using an explicit description of the surface: 𝑓𝑓 (𝑥𝑥, 𝑦𝑦, 𝑧𝑧) = 25 − 𝑥𝑥 2 − 𝑦𝑦 2, 𝑆𝑆
is the hemisphere centered at the origin with radius 5, for 𝑧𝑧 ≥ 0.
11. (question 41) Average value Find the average value of the function 𝑓𝑓 (𝑥𝑥, 𝑦𝑦, 𝑧𝑧) = 𝑥𝑥𝑥𝑥𝑥𝑥 on
the unit sphere in the first octant.
12. (question 43) Surface integral of a vector field Find the flux of the following vector field
across the given surface with the specified orientation. You may use either an explicit or a
parametric description of the surface: 𝑭𝑭 = 〈0,0, −1〉 across the slanted face of the
tetrahedron 𝑧𝑧 = 4 − 𝑥𝑥 − 𝑦𝑦 in the first octant: normal vector points outward.
13. (question 45) Surface integral of a vector field Find the flux of the following vector field
across the given surface with the specified orientation. You may use either an explicit or a
parametric description of the surface: 𝑭𝑭 = 〈𝑥𝑥, 𝑦𝑦, 𝑧𝑧〉 across the slanted surface of the cone
𝑧𝑧 2 = 𝑥𝑥 2 + 𝑦𝑦 2 for 0 ≤ 𝑧𝑧 ≤ 1: normal vector points upward

51
14. (question 53) Miscellaneous surface integral Evaluate the following integral using a
method of your choice. Assume the normal vector points either outward or
〈𝑥𝑥,0,𝑧𝑧〉
upward:∬𝑆𝑆 ∙ 𝒏𝒏𝑑𝑑𝑑𝑑, where 𝑆𝑆 is the cylinder 𝑥𝑥 2 + 𝑧𝑧 2 = 𝑎𝑎2 , |𝑦𝑦| ≤ 2.
√𝑥𝑥 2 +𝑧𝑧 2
15. (question 55) Cylinder and Sphere Consider the sphere 𝑥𝑥 2 + 𝑦𝑦 2 + 𝑧𝑧 2 = 4 and the
cylinder (𝑥𝑥 − 1)2 + 𝑦𝑦 2 = 1, for 𝑧𝑧 ≥ 0. Find the surface area of the cylinder inside the
sphere.
16. (question 56) Flux on a tetrahedron Find the upward flux of the field: 𝑭𝑭 = 〈𝑥𝑥, 𝑦𝑦, 𝑧𝑧〉 across
𝑥𝑥 𝑦𝑦 𝑧𝑧
the plane + + = 1 in the first octant, where 𝑎𝑎, 𝑏𝑏 and 𝑐𝑐 are positive constants. Show
𝑎𝑎 𝑏𝑏 𝑐𝑐
that the flux equals 𝑐𝑐 times the area of the base of the region. Interpret the result physically.
17. (question 69) Mass and center of mass Let 𝑆𝑆 be a surface that represents a thin shell with
𝑘𝑘𝑘𝑘
surface mass density of 𝜎𝜎: 𝑆𝑆𝑆𝑆 units are 𝑚𝑚2. The moments about the coordinate planes are:
𝑀𝑀𝑦𝑦𝑦𝑦 = ∬𝑆𝑆 𝑥𝑥𝑥𝑥 (𝑥𝑥, 𝑦𝑦, 𝑧𝑧)𝑑𝑑𝑑𝑑 𝑀𝑀𝑦𝑦𝑦𝑦 = ∬𝑆𝑆 𝑧𝑧𝑧𝑧 (𝑥𝑥, 𝑦𝑦, 𝑧𝑧)𝑑𝑑𝑑𝑑 𝑀𝑀𝑥𝑥𝑥𝑥 = ∬𝑆𝑆 𝑦𝑦𝑦𝑦(𝑥𝑥, 𝑦𝑦, 𝑧𝑧)𝑑𝑑𝑑𝑑 .
𝑀𝑀𝑦𝑦𝑦𝑦
The coordinates of the center of mass of the shell are defined as: 𝑥𝑥̅ = 𝑚𝑚
𝑦𝑦� =
𝑀𝑀𝑥𝑥𝑥𝑥 𝑀𝑀𝑦𝑦𝑦𝑦
𝑧𝑧̅ = where 𝑚𝑚 is the mass of the shell. Find the mass and the center of mass of
𝑚𝑚 𝑚𝑚
the following shell. Use symmetry wherever possible: The cylinder 𝑥𝑥 2 + 𝑧𝑧 2 = 𝑎𝑎2 , 0 ≤ 𝑧𝑧 ≤
2, with surface mass density 𝜎𝜎 (𝑥𝑥, 𝑦𝑦, 𝑧𝑧) = 1 + 𝑧𝑧. Note Chapter 16.6 of the textbook goes
into more details on this subject, but this is not required to solve this problem. You are not
responsible for section 16.6 of the textbook.
18. (question 71) Special case of surface integrals of a scalar-valued function Suppose a
surface 𝑆𝑆 is defined as 𝑧𝑧 = 𝑔𝑔(𝑥𝑥, 𝑦𝑦) on a region 𝑅𝑅. Show that 𝒕𝒕𝑥𝑥 × 𝒕𝒕𝑦𝑦 = 〈−𝑧𝑧𝑥𝑥 , −𝑧𝑧𝑦𝑦 , 1〉 and
that ∬𝑆𝑆 𝑓𝑓(𝑥𝑥, 𝑦𝑦, 𝑧𝑧)𝑑𝑑𝑑𝑑 = ∬𝑅𝑅 𝑓𝑓(𝑥𝑥, 𝑦𝑦, 𝑔𝑔(𝑥𝑥, 𝑦𝑦)�1 + 𝑧𝑧𝑥𝑥2 + 𝑧𝑧𝑦𝑦2 𝑑𝑑𝑑𝑑.

You can find the solutions to the problems at the following link: Solutions to Reading Assignment
4 Problems.

52
1.7 Convention on Assigning the Direction of Tangent and Normal
Vectors

Before moving on to the next topic, it is important to note that in defining the tangent vectors and
normal vectors there is a built in ambiguity in terms of which direction to select; clockwise or
counter clockwise for traversing a contour and inwards pointing or outwards pointing when
defining a normal to a contour in a 2d plane or normal to a surface in 3d. Note the 𝑥𝑥𝑥𝑥 plane in 2d
is represented by a normal that points in either the 𝑧𝑧̂ or – 𝑧𝑧̂ direction in 3d.

Generally, we impose a convention, to remove any ambiguity and remain consistent with
everything we have done so far. In Reading Assignment 0 we mentioned an ambiguity when
labeling orthogonal coordinate axes in 3d. One can adopt either a right-handed or left-handed
coordinate system when labeling coordinate axes. In most physical problems a right-handed
coordinate system has been adopted in which case we apply the right-hand rule. The right-hand
rule determines the direction of the 𝑧𝑧 axis with respect to the 𝑥𝑥 and 𝑦𝑦 axes: the right-hand fingers
curl in the direction from the 𝑥𝑥 axis to the 𝑦𝑦 axis and the thumb points in the direction of the 𝑧𝑧
axis. We now summarize the conventions used in the remaining part of the course:

Conventions

• When defining traversal around a contour in 2d, in a counter-clockwise sense, we take the
normal to the contour in 2d to be to our right. This implies that the normal vector always
points outwards from a closed contour in 2d.
• The convention for a 3d geometry is in some sense similar: the normal to a closed surface,
points outwards from the surface.
• The direction vector assigned to the orientation of the area enclosed by a contour traversed
in a counter-clockwise direction is associated with the right-hand convention: fingers of the
right-hand point in the direction of the contour traversal and the thumb points in the direction
of the oriented area.

These conventions are important to remember because they will serve as the backbone once we
introduce the Divergence Theorem and Stokes’ Theorem.

1.8 Properties of a Vector Field

Consider a simple vector field in 2d to illustrate some basic ideas before the ideas are developed
into a framework which allows practical calculations to be considered and to be extended into 3d.

Example: A 2d vector field is defined as follows:

𝑭𝑭 = 〈𝑥𝑥 − 𝑦𝑦, 𝑦𝑦 + 𝑥𝑥 〉

53
We will take a leap in faith and write this vector out as the sum of two vector fields with different
properties, which we will discover.

𝑭𝑭𝑓𝑓𝑓𝑓𝑓𝑓𝑓𝑓 = 〈𝑥𝑥, 𝑦𝑦〉 𝑭𝑭𝑐𝑐𝑐𝑐𝑐𝑐𝑐𝑐 = 〈−𝑦𝑦, 𝑥𝑥 〉 𝑭𝑭 = 𝑭𝑭𝑓𝑓𝑓𝑓𝑓𝑓𝑥𝑥 + 𝑭𝑭𝑐𝑐𝑐𝑐𝑐𝑐𝑐𝑐

The vector 〈𝑥𝑥, 𝑦𝑦〉, as noted earlier in section 1.2, is often referred to as a 2d radial field. A radial
field in 3d would take on the form 〈𝑥𝑥, 𝑦𝑦, 𝑧𝑧〉. Note, a scalar multiplying a radial field is also a radial
field.

The next step is to investigate the flux and circulation vector fields over some closed contour. Take
the contour to be a circle of radius 𝑟𝑟 centred on the origin, to make things simple.

𝑭𝑭𝒇𝒇𝒇𝒇𝒇𝒇𝒇𝒇 𝑭𝑭𝒄𝒄𝒄𝒄𝒄𝒄𝒄𝒄 𝑭𝑭

Fig. 7 Graph of 𝑭𝑭, 𝑭𝑭𝒇𝒇𝒇𝒇𝒇𝒇𝒇𝒇, 𝑭𝑭𝒄𝒄𝒄𝒄𝒄𝒄𝒄𝒄, 𝑻𝑻𝑑𝑑𝑑𝑑 and 𝒏𝒏𝒔𝒔 𝑑𝑑𝑑𝑑

The graphs for the three vectors are shown in Fig. 7 along with the closed contour and the unit
tangent vector 𝑻𝑻 and unit normal vector 𝒏𝒏𝒔𝒔 marked on all three graphs. The physical interpretation
of 𝑭𝑭𝒇𝒇𝒇𝒇𝒇𝒇𝒇𝒇 and 𝑭𝑭𝒄𝒄𝒄𝒄𝒄𝒄𝒄𝒄 will become apparent after the conclusion of the following discussion.

1.8.1 Description of 𝑭𝑭𝒇𝒇𝒇𝒇𝒇𝒇𝒇𝒇

We can traverse the contour in 2d in either a CW or CCW direction. This creates an ambiguity and
therefore we adopt a right-handed convention and choose the direction to be CCW. We look to our
right to determine the orientation of the vector 𝒏𝒏𝒔𝒔 , which would be outwards from the contour.
Traversing a contour in a CW direction implies that the orientation of the vector 𝒏𝒏𝒔𝒔 would be
pointing inwards from the contour, that is, towards the origin. As an aside, normal to a contour in
3d would have no practical meaning since there are an infinite number of vectors that are normal
to a contour in 3d.

54
Upon inspection of 𝑭𝑭𝒇𝒇𝒇𝒇𝒇𝒇𝒇𝒇, it appears that the field lines emanate from a single point. The direction
of 𝒏𝒏𝒔𝒔 with respect to 𝑭𝑭𝒇𝒇𝒇𝒇𝒇𝒇𝒇𝒇 is parallel at all points on the closed contour. If we now integrate the
scalar dot product of 𝒏𝒏𝒔𝒔 and 𝑭𝑭𝒇𝒇𝒇𝒇𝒇𝒇𝒇𝒇, in a CCW direction around the closed contour, we will obtain
a scalar which represents 𝐹𝐹𝐹𝐹𝐹𝐹𝐹𝐹. The procedure for performing this calculation will be discussed
shortly. Had the field lines for 𝑭𝑭𝒇𝒇𝒇𝒇𝒇𝒇𝒇𝒇 been in the opposite direction, the integration would have led
to a sign reversal.

Source: We refer to a source if the field lines emanate from or diverge from a point. Sometimes
we use the word sink to indicate that the field lines converge to a point. If the field lines point
outwards from the source (diverge), then the source takes on a positive value. Alternatively, if the
field lines point inwards towards the source (converge) then the source takes on a negative value.
The source has scalar-like properties which are either zero, positive or negative in value. We can
redraw the circular contour around an arbitrary point centred at (𝑥𝑥𝑜𝑜 , 𝑦𝑦0 ) and in this way compute
the flux at all locations on the 2d plane.

1.8.2 Description of 𝑭𝑭𝒄𝒄𝒄𝒄𝒄𝒄𝒄𝒄

Relabel 𝑭𝑭𝒄𝒄𝒄𝒄𝒄𝒄𝒄𝒄 as 𝑭𝑭𝒄𝒄𝒄𝒄𝒄𝒄𝒄𝒄𝒄𝒄 for the purpose of the following discussion. The suffix 𝑧𝑧 indicates that the
axis of rotation is parallel to the 𝑧𝑧 axis. The magnitude of the field 𝑭𝑭𝒄𝒄𝒄𝒄𝒄𝒄𝒄𝒄𝒄𝒄 = 〈−𝑦𝑦, 𝑥𝑥 〉 = 〈−𝑦𝑦, 𝑥𝑥, 0〉
as you traverse a circle centred on the origin is constant. The direction of 𝑭𝑭𝒄𝒄𝒄𝒄𝒄𝒄𝒄𝒄𝒄𝒄 with respect to 𝑻𝑻,
which is associated with the closed contour, is parallel at all points on the closed contour. The field
lines exhibit the properties of a rotation and, by definition, rotation implies an axis of rotation. The
direction of the axis of rotation is determined using the right-hand rule. The fingers of the right-
hand point in the direction of the field and the thumb points in the direction of the axis of rotation.
Unfortunately, the axis of rotation in 2d can’t be defined in a 2d geometric sense so we consider it
as an imaginary axis in either the 𝑧𝑧 or – 𝑧𝑧 direction, depending on the direction in which the field
is pointing. Therefore, circulation differs from flux in that an orientation is imposed by the axis of
rotation. In 2d, the 𝑥𝑥𝑥𝑥 plane has no orientation and so we must invent an imaginary axis that is
orthogonal to the 𝑥𝑥𝑥𝑥 − plane.

Consider how we can extend the concept of circulation in a 2 plane to circulation in 3d, for the
special case where the components of the vector field are linear in the variables 𝑥𝑥, 𝑦𝑦 and 𝑧𝑧. In this
case, the axis of rotation is no longer imaginary. The 3d vector field is given by 𝑭𝑭𝒄𝒄𝒄𝒄𝒄𝒄𝒄𝒄 =
〈−𝑦𝑦 + 𝑧𝑧, 𝑥𝑥 − 𝑧𝑧, 𝑦𝑦 − 𝑥𝑥 〉. In this example we are interested only in the axis of rotation at the origin.
In 3d, the contour would still be a circle of radius 𝑟𝑟 but there is one circle centred on the origin,
for each principal plane:the 𝑥𝑥𝑥𝑥 −, 𝑦𝑦𝑦𝑦 − and 𝑧𝑧𝑧𝑧 − planes.

55
The first step is to project the vector feld onto the three principle planes in 3d, and assume a right-
handed coordinate system. We arrive at the following result: the field 𝑭𝑭𝒄𝒄𝒄𝒄𝒄𝒄𝒄𝒄𝒄𝒄 = 〈−𝑦𝑦, 𝑥𝑥, 0〉 in the
𝑥𝑥𝑥𝑥 − plane, the field 𝑭𝑭𝒄𝒄𝒄𝒄𝒄𝒄𝒄𝒄𝒄𝒄 = 〈0, −𝑧𝑧, 𝑦𝑦〉 in the 𝑦𝑦𝑦𝑦 − plane and the field 𝑭𝑭𝒄𝒄𝒄𝒄𝒄𝒄𝒄𝒄𝒄𝒄 = 〈𝑧𝑧, 0, −𝑥𝑥 〉 in the
𝑧𝑧𝑧𝑧 − plane. The fingers of the right-hand point in the direction of the field and the thumb points
in the direction of the axis of rotation. Hence the unit vector representing the axis of rotation for
𝑭𝑭𝒄𝒄𝒄𝒄𝒄𝒄𝒄𝒄𝒄𝒄, 𝑭𝑭𝒄𝒄𝒄𝒄𝒄𝒄𝒄𝒄𝒄𝒄 and 𝑭𝑭𝒄𝒄𝒄𝒄𝒄𝒄𝒄𝒄𝒄𝒄 is in the 𝑧𝑧̂ , 𝑦𝑦� and 𝑥𝑥� direction respectively.

The axis of rotation for the 3d vector field 𝑭𝑭𝒄𝒄𝒄𝒄𝒄𝒄𝒄𝒄 = 〈−𝑦𝑦 + 𝑧𝑧, 𝑥𝑥 − 𝑧𝑧, 𝑦𝑦 − 𝑥𝑥 〉 is obtained by
1
superposition. It points in the direction 〈1,1,1〉 (a unit vector), in 3d space.
√3

You can see the cases 𝑭𝑭𝒄𝒄𝒄𝒄𝒄𝒄𝒄𝒄𝒄𝒄 = 〈−𝑦𝑦, 𝑥𝑥, 0〉, 𝑭𝑭𝒄𝒄𝒊𝒊𝒊𝒊𝒊𝒊𝒊𝒊 = 〈0, −𝑧𝑧, 𝑦𝑦〉 and 𝑭𝑭𝒄𝒄𝒄𝒄𝒄𝒄𝒄𝒄𝒄𝒄 = 〈𝑧𝑧, 0, −𝑥𝑥 〉 and 𝑭𝑭𝒄𝒄𝒄𝒄𝒄𝒄𝒄𝒄
if you click on the following link:

https://www.math3d.org/Eh9ZlWlIx

The vector fields in the interactive simulation have been normalized (unit vectors). We will revisit
the 3d case for Circulation later when we discuss the formulation of an expression for the curl of
a 3d vector field.

If we now integrate the scalar dot product of 𝑻𝑻 and 𝑭𝑭𝒄𝒄𝒄𝒄𝒄𝒄𝒄𝒄, in a CCW direction, we will obtain a
number which represents the 𝐶𝐶𝐶𝐶𝐶𝐶𝐶𝐶𝐶𝐶𝐶𝐶𝐶𝐶𝐶𝐶𝐶𝐶𝐶𝐶𝐶𝐶. The procedure for performing this calculation will be
discussed shortly. The integration result has a sign reversal if the field lines for 𝑭𝑭𝒄𝒄𝒄𝒄𝒄𝒄𝒄𝒄 are in the
opposite direction.

Circulation: The circulation integral will give a number that indicates the magnitude of the
circulation. In our example the circulation field 𝑭𝑭𝒄𝒄𝒄𝒄𝒄𝒄𝒄𝒄 is oriented in a CCW direction and is constant
along the periphery of the circle of radius 𝑟𝑟 centred on the origin. The integral of 𝑭𝑭𝒄𝒄𝒄𝒄𝒄𝒄𝒄𝒄 ∙ 𝑻𝑻 around
the closed contour in this case is positive and coincides with an axis of rotation in the same
direction as the fictitious 𝑧𝑧̂ axis. If we reverse the direction of the field, 𝑭𝑭𝒄𝒄𝒄𝒄𝒄𝒄𝒄𝒄 then the value of the
integral will be negative which implies that the axis of rotation is in the same direction as the
fictitious −𝑧𝑧̂ axis. We can redraw the circular contour around an arbitrary point centred at (𝑥𝑥𝑜𝑜 , 𝑦𝑦0 )
and in this way compute the 𝐶𝐶𝐶𝐶𝐶𝐶𝐶𝐶𝐶𝐶𝐶𝐶𝐶𝐶𝐶𝐶𝐶𝐶𝐶𝐶𝐶𝐶 for circles centred at (𝑥𝑥𝑜𝑜 , 𝑦𝑦0 ) in the 2d plane. The axis
of rotation now passes through the point (𝑥𝑥𝑜𝑜 , 𝑦𝑦0 ). We can extend this process to the 3d case by
assuming a sphere of radius 𝑟𝑟 centred on the point (𝑥𝑥𝑜𝑜 , 𝑦𝑦0 , 𝑧𝑧0 ). Three circles of radius 𝑟𝑟 are then
scribed on the sphere with each circle lying on a plane that is parallel to one of the principal planes.
An interactive visualization of this description can be found at the following link.

https://www.math3d.org/axthxGQS4

56
1.8.3 Computing 𝑪𝑪𝑪𝑪𝑪𝑪𝑪𝑪𝑪𝑪𝑪𝑪𝑪𝑪𝑪𝑪𝑪𝑪𝑪𝑪𝑪𝑪 and 𝑭𝑭𝑭𝑭𝑭𝑭𝑭𝑭

We now demonstrate the computation of the four combinations involving contour integrals
assuming either 𝑭𝑭 = 𝑭𝑭𝒄𝒄𝒄𝒄𝒄𝒄𝒄𝒄 or 𝑭𝑭 = 𝑭𝑭𝒇𝒇𝒍𝒍𝒍𝒍𝒍𝒍 . The circular contour 𝐶𝐶 of radius 𝑟𝑟 is parameterized as
follows and is traversed in a counter-clockwise direction:

𝐶𝐶 = {𝑟𝑟⃑(𝑡𝑡) = 𝑟𝑟〈𝑐𝑐𝑐𝑐𝑐𝑐𝑐𝑐, 𝑠𝑠𝑠𝑠𝑠𝑠𝑠𝑠〉, 0 ≤ 𝑡𝑡 ≤ 2𝜋𝜋}

This implies that 𝑻𝑻𝑑𝑑𝑑𝑑 = 𝑟𝑟〈−𝑠𝑠𝑠𝑠𝑠𝑠𝑠𝑠, 𝑐𝑐𝑐𝑐𝑐𝑐𝑐𝑐〉𝑑𝑑𝑑𝑑 and 𝒏𝒏𝒔𝒔 𝑑𝑑𝑑𝑑 = 𝑟𝑟〈𝑐𝑐𝑐𝑐𝑐𝑐𝑐𝑐, 𝑠𝑠𝑠𝑠𝑠𝑠𝑠𝑠〉𝑑𝑑𝑑𝑑. The four integrals then
become:
2𝜋𝜋
𝐶𝐶𝐶𝐶𝐶𝐶𝐶𝐶𝐶𝐶𝐶𝐶𝐶𝐶𝐶𝐶𝐶𝐶𝐶𝐶𝐶𝐶 = �𝑭𝑭𝒄𝒄𝒄𝒄𝒄𝒄𝒄𝒄 ∙ 𝑻𝑻𝑑𝑑𝑑𝑑 = � 𝑟𝑟〈−𝑠𝑠𝑠𝑠𝑠𝑠𝑠𝑠, 𝑐𝑐𝑐𝑐𝑐𝑐𝑐𝑐〉 ∙ 𝑟𝑟〈−𝑠𝑠𝑠𝑠𝑠𝑠𝑠𝑠, 𝑐𝑐𝑐𝑐𝑐𝑐𝑐𝑐〉𝑑𝑑𝑑𝑑 = 2𝜋𝜋𝑟𝑟 2
𝐶𝐶 0

2𝜋𝜋
𝐹𝐹𝐹𝐹𝐹𝐹𝐹𝐹 = �𝑭𝑭𝒄𝒄𝒄𝒄𝒄𝒄𝒄𝒄 ∙ 𝒏𝒏𝒔𝒔 𝑑𝑑𝑑𝑑 = � 𝑟𝑟〈−𝑠𝑠𝑠𝑠𝑠𝑠𝑠𝑠, 𝑐𝑐𝑐𝑐𝑐𝑐𝑐𝑐〉 ∙ 𝑟𝑟〈𝑐𝑐𝑐𝑐𝑐𝑐𝑐𝑐, 𝑠𝑠𝑠𝑠𝑠𝑠𝑠𝑠〉𝑑𝑑𝑑𝑑 = 0
𝐶𝐶 0

2𝜋𝜋
𝐶𝐶𝐶𝐶𝐶𝐶𝐶𝐶𝐶𝐶𝐶𝐶𝐶𝐶𝐶𝐶𝑖𝑖𝑜𝑜𝑜𝑜 = �𝑭𝑭𝒇𝒇𝒇𝒇𝒇𝒇𝒇𝒇 ∙ 𝑻𝑻𝑑𝑑𝑑𝑑 = � 𝑟𝑟〈𝑐𝑐𝑐𝑐𝑐𝑐𝑐𝑐, 𝑠𝑠𝑠𝑠𝑠𝑠𝑠𝑠〉 ∙ 𝑟𝑟〈−𝑠𝑠𝑠𝑠𝑠𝑠𝑠𝑠, 𝑐𝑐𝑐𝑐𝑐𝑐𝑐𝑐〉𝑑𝑑𝑑𝑑 = 0
𝐶𝐶 0

2𝜋𝜋
𝐹𝐹𝐹𝐹𝐹𝐹𝐹𝐹 = �𝑭𝑭𝒇𝒇𝒇𝒇𝒇𝒇𝒇𝒇 ∙ 𝒏𝒏𝒔𝒔 𝑑𝑑𝑑𝑑 = � 𝑟𝑟〈𝑐𝑐𝑐𝑐𝑐𝑐𝑐𝑐, 𝑠𝑠𝑠𝑠𝑠𝑠𝑠𝑠〉 ∙ 𝑟𝑟〈𝑐𝑐𝑐𝑐𝑐𝑐𝑐𝑐, 𝑠𝑠𝑠𝑠𝑠𝑠𝑠𝑠〉𝑑𝑑𝑑𝑑 = 2𝜋𝜋𝑟𝑟 2
𝐶𝐶 0

We can repeat this procedure considering different locations (𝑥𝑥𝑜𝑜 , 𝑦𝑦𝑜𝑜 ) on the 2d plane but then the
circular contour encircles the point (𝑥𝑥𝑜𝑜 , 𝑦𝑦𝑜𝑜 ). The position vector in this case is given by:

𝐶𝐶 = {𝑟𝑟⃑(𝑡𝑡) = 〈𝑥𝑥𝑜𝑜 + 𝑟𝑟𝑟𝑟𝑟𝑟𝑟𝑟𝑟𝑟, 𝑦𝑦𝑜𝑜 + 𝑟𝑟𝑟𝑟𝑟𝑟𝑟𝑟𝑟𝑟〉, 0 ≤ 𝑡𝑡 ≤ 2𝜋𝜋}

This implies that 𝑻𝑻𝑑𝑑𝑑𝑑 = 𝑟𝑟〈−𝑠𝑠𝑠𝑠𝑠𝑠𝑠𝑠, 𝑐𝑐𝑐𝑐𝑐𝑐𝑐𝑐〉𝑑𝑑𝑑𝑑 and 𝒏𝒏𝒔𝒔 𝑑𝑑𝑑𝑑 = 𝑟𝑟〈𝑐𝑐𝑐𝑐𝑐𝑐𝑐𝑐, 𝑠𝑠𝑠𝑠𝑠𝑠𝑡𝑡〉𝑑𝑑𝑑𝑑. The four integrals then
become:
2𝜋𝜋
𝐶𝐶𝐶𝐶𝐶𝐶𝐶𝐶𝐶𝐶𝐶𝐶𝐶𝐶𝐶𝐶𝐶𝐶𝐶𝐶𝐶𝐶 = �𝑭𝑭𝒄𝒄𝒄𝒄𝒄𝒄𝒄𝒄 ∙ 𝑻𝑻𝑑𝑑𝑑𝑑 = � 〈−𝑦𝑦𝑜𝑜 − 𝑟𝑟𝑟𝑟𝑟𝑟𝑟𝑟𝑟𝑟, 𝑥𝑥𝑜𝑜 + 𝑟𝑟𝑟𝑟𝑟𝑟𝑟𝑟𝑟𝑟〉 ∙ 𝑟𝑟〈−𝑠𝑠𝑠𝑠𝑠𝑠𝑠𝑠, 𝑐𝑐𝑐𝑐𝑐𝑐𝑐𝑐〉𝑑𝑑𝑑𝑑 = 2𝜋𝜋𝑟𝑟 2
𝐶𝐶 0

2𝜋𝜋
𝐹𝐹𝐹𝐹𝐹𝐹𝐹𝐹 = �𝑭𝑭𝒄𝒄𝒄𝒄𝒄𝒄𝒄𝒄 ∙ 𝒏𝒏𝒔𝒔 𝑑𝑑𝑑𝑑 = � 〈−𝑦𝑦𝑜𝑜 − 𝑟𝑟𝑟𝑟𝑟𝑟𝑟𝑟𝑟𝑟, 𝑥𝑥𝑜𝑜 + 𝑟𝑟𝑟𝑟𝑟𝑟𝑟𝑟𝑟𝑟〉 ∙ 𝑟𝑟〈𝑐𝑐𝑐𝑐𝑐𝑐𝑐𝑐, 𝑠𝑠𝑠𝑠𝑠𝑠𝑠𝑠〉𝑑𝑑𝑑𝑑 = 0
𝐶𝐶 0
2𝜋𝜋
𝐶𝐶𝐶𝐶𝐶𝐶𝐶𝐶𝐶𝐶𝐶𝐶𝐶𝐶𝐶𝐶𝐶𝐶𝐶𝐶𝐶𝐶 = �𝑭𝑭𝒇𝒇𝒇𝒇𝒇𝒇𝒇𝒇 ∙ 𝑻𝑻𝑑𝑑𝑑𝑑 = � 〈𝑥𝑥𝑜𝑜 + 𝑟𝑟𝑟𝑟𝑟𝑟𝑟𝑟𝑟𝑟, 𝑦𝑦𝑜𝑜 + 𝑟𝑟𝑟𝑟𝑟𝑟𝑟𝑟𝑟𝑟〉 ∙ 𝑟𝑟〈−𝑠𝑠𝑠𝑠𝑠𝑠𝑠𝑠, 𝑐𝑐𝑐𝑐𝑐𝑐𝑐𝑐〉𝑑𝑑𝑑𝑑 = 0
𝐶𝐶 0

2𝜋𝜋
𝐹𝐹𝐹𝐹𝐹𝐹𝐹𝐹 = �𝑭𝑭𝒇𝒇𝒇𝒇𝒇𝒇𝒇𝒇 ∙ 𝒏𝒏𝒔𝒔 𝑑𝑑𝑑𝑑 = � 〈𝑥𝑥𝑜𝑜 + 𝑟𝑟𝑟𝑟𝑟𝑟𝑟𝑟𝑟𝑟, 𝑦𝑦𝑜𝑜 + 𝑟𝑟𝑟𝑟𝑟𝑟𝑟𝑟𝑟𝑟〉 ∙ 𝑟𝑟〈𝑐𝑐𝑐𝑐𝑐𝑐𝑐𝑐, 𝑠𝑠𝑠𝑠𝑠𝑠𝑠𝑠〉𝑑𝑑𝑑𝑑 = 2𝜋𝜋𝑟𝑟 2
𝐶𝐶 0

57
For these specific vector fields, the results are unaffected by location.

1.8.4 Making Circulation and Flux Independent of radius 𝒓𝒓

Recall that the 𝐶𝐶𝐶𝐶𝐶𝐶𝐶𝐶𝐶𝐶𝐶𝐶𝐶𝐶𝐶𝐶𝐶𝐶𝐶𝐶𝐶𝐶 and 𝐹𝐹𝐹𝐹𝐹𝐹𝐹𝐹 depend on the choice of 𝑟𝑟. Consider scaling 𝐶𝐶𝐶𝐶𝐶𝐶𝐶𝐶𝐶𝐶𝐶𝐶𝐶𝐶𝐶𝐶𝐶𝐶𝐶𝐶𝐶𝐶
or 𝐹𝐹𝐹𝐹𝐹𝐹𝐹𝐹 so that they do not depend on the radius 𝑟𝑟 of the contour. The simplest solution is to divide
both expressions by 𝜋𝜋𝜋𝜋 2 which is the 𝐴𝐴𝐴𝐴𝐴𝐴𝐴𝐴 enclosed by the contour. Hence, we obtain the
expressions:

𝐶𝐶𝐶𝐶𝐶𝐶𝐶𝐶𝐶𝐶𝐶𝐶𝐶𝐶𝐶𝐶𝐶𝐶𝐶𝐶𝐶𝐶 𝐹𝐹𝐹𝐹𝐹𝐹𝐹𝐹
=2 𝑎𝑎𝑎𝑎𝑎𝑎 =2
𝐴𝐴𝐴𝐴𝐴𝐴𝐴𝐴 𝐴𝐴𝐴𝐴𝐴𝐴𝐴𝐴

We will use the concept of dividing by an area when we scale 𝐶𝐶𝐶𝐶𝐶𝐶𝐶𝐶𝐶𝐶𝐶𝐶𝐶𝐶𝐶𝐶𝐶𝐶𝐶𝐶𝐶𝐶 and 𝐹𝐹𝐹𝐹𝐹𝐹𝐹𝐹 on a
differential level, that is we let the radius of the contour go to zero and from this we derive two
new vector operators: ∇ ∙ 𝑭𝑭 representing the divergence of a vector field (related to the flux-like
component of a vector field) and ∇ × 𝑭𝑭, representing the curl of a vector field (related to the
circulation-like component of the vector field).

1.8.4.1 Interactive Visualization of Flux/Area

An interactive simulation indicating how the 𝐹𝐹𝐹𝐹𝐹𝐹𝐹𝐹/𝐴𝐴𝐴𝐴𝐴𝐴𝐴𝐴 changes as a function of position (𝑥𝑥𝑜𝑜 , 𝑦𝑦0 )
can be accessed at the following link:

https://www.math3d.org/lTf4mNhJQ

58
Fig. 8 Screenshot of the Interactive Simulation that Pictorially Describes the Concept of the
Flux/Area of a Vector Field in 2d

A screenshot showing the contour encircling a given point (𝑥𝑥𝑜𝑜 , 𝑦𝑦0 ) is shown in Fig. 8. One of the
position vectors is to the given point (𝑥𝑥𝑜𝑜 , 𝑦𝑦0 ) and the other position vector traces out the contour
with respect to the contour’s centre at (𝑥𝑥𝑜𝑜 , 𝑦𝑦0 ). The area enclosed by the contour has a colour
shading that depends on the numerical value of the 𝐹𝐹𝐹𝐹𝐹𝐹𝐹𝐹/𝐴𝐴𝐴𝐴𝐴𝐴𝐴𝐴. In our example, the 𝐹𝐹𝐹𝐹𝐹𝐹𝐹𝐹/𝐴𝐴𝐴𝐴𝐴𝐴𝑎𝑎
is constant at every position (𝑥𝑥𝑜𝑜 , 𝑦𝑦0 ). The radius of the circle should be made vanishingly small
but here we left the radius large enough so that you can observe the colour representing the
magnitude of the 𝐹𝐹𝐹𝐹𝐹𝐹𝐹𝐹/𝐴𝐴𝐴𝐴𝐴𝐴𝐴𝐴 at the position (𝑥𝑥𝑜𝑜 , 𝑦𝑦0 ).

The interactive simulation has two sliders that can be manipulated.

𝑷𝑷: controls the motion of the black arrow which traces out the contour of the circle.
𝑻𝑻: controls the position (𝑥𝑥𝑜𝑜 , 𝑦𝑦0 ) of the center of the circle.

The intensity of the pink shaded region is constant irrespective of position (𝑥𝑥𝑜𝑜 , 𝑦𝑦0 ) since the
Flux/Area is the same everywhere but if the 𝐹𝐹𝐹𝐹𝐹𝐹𝐹𝐹/𝐴𝐴𝐴𝐴𝐴𝐴𝐴𝐴 had been a function of position, then the
intensity of the shaded region would vary with position. This irrotational field has a 𝐹𝐹𝐹𝐹𝐹𝐹𝐹𝐹/𝐴𝐴𝐴𝐴𝐴𝐴𝐴𝐴 of
2 but no 𝐶𝐶𝐶𝐶𝐶𝐶𝐶𝐶𝐶𝐶𝐶𝐶𝐶𝐶𝐶𝐶𝐶𝐶𝐶𝐶𝐶𝐶/𝐴𝐴𝐴𝐴𝐴𝐴𝐴𝐴.

1.8.4.2 Interactive Visualization of Circulation/Area

An interactive simulation indicating how the 𝐶𝐶𝐶𝐶𝐶𝐶𝐶𝐶𝐶𝐶𝐶𝐶𝐶𝐶𝐶𝐶𝐶𝐶𝐶𝐶𝐶𝐶/𝐴𝐴𝐴𝐴𝐴𝐴𝐴𝐴 changes as a function of position


(𝑥𝑥𝑜𝑜 , 𝑦𝑦0 ) can be accessed at the following link:

https://www.math3d.org/ytrzsUli3

Fig. 9 Screenshot of the Interactive Simulation that Pictorially Describes the Concept of the
Circulation/Area of a Vector Field in 2d

59
A screenshot showing the contour encircling a given point (𝑥𝑥𝑜𝑜 , 𝑦𝑦0 ) is shown in Fig. 9. One of the
position vectors is to the given point (𝑥𝑥𝑜𝑜 , 𝑦𝑦0 ) and the other position vector traces out the contour
with respect to the contour’s centre at (𝑥𝑥𝑜𝑜 , 𝑦𝑦0 ).

The interactive simulation has two sliders that can be manipulated.

𝑷𝑷: controls the motion of the black arrow which traces out the contour of the circle.
𝑻𝑻: controls the position (𝑥𝑥𝑜𝑜 , 𝑦𝑦0 ) of the center of the circle.

The red arrows (vector) represent the 𝐶𝐶𝐶𝐶𝐶𝐶𝐶𝐶𝐶𝐶𝐶𝐶𝐶𝐶𝐶𝐶𝐶𝐶𝐶𝐶𝐶𝐶/𝐴𝐴𝐴𝐴𝐴𝐴𝐴𝐴 of the vector field shown in blue. The
radius of the circle is vanishingly small but large enough so that you can observe the length of the
vector representing the magnitude of the 𝐶𝐶𝐶𝐶𝐶𝐶𝐶𝐶𝐶𝐶𝐶𝐶𝐶𝐶𝐶𝐶𝐶𝐶𝐶𝐶𝐶𝐶/𝐴𝐴𝐴𝐴𝐴𝐴𝐴𝐴 at the position (𝑥𝑥𝑜𝑜 , 𝑦𝑦0 ).

The axis of rotation points in the 𝑧𝑧̂ direction and has a constant magnitude (length of red arrows)
irrespective of position (𝑥𝑥𝑜𝑜 , 𝑦𝑦0 ) since the 𝐶𝐶𝐶𝐶𝐶𝐶𝐶𝐶𝐶𝐶𝐶𝐶𝐶𝐶𝐶𝐶𝐶𝐶𝐶𝐶𝐶𝐶/𝐴𝐴𝐴𝐴𝐴𝐴𝐴𝐴 of the vector field is the same
everywhere, but if the 𝐶𝐶𝐶𝐶𝐶𝐶𝐶𝐶𝐶𝐶𝐶𝐶𝐶𝐶𝐶𝐶𝐶𝐶𝐶𝐶𝐶𝐶/𝐴𝐴𝐴𝐴𝐴𝐴𝐴𝐴 of the vector field had been a function of position, then
the magnitude of the circulation (length of the red arrows) would vary with position. This flux-
free field has a 𝐶𝐶𝐶𝐶𝐶𝐶𝐶𝐶𝐶𝐶𝐶𝐶𝐶𝐶𝐶𝐶𝐶𝐶𝐶𝐶𝐶𝐶/𝐴𝐴𝐴𝐴𝐴𝐴𝐴𝐴 of 2 but no 𝐹𝐹𝐹𝐹𝐹𝐹𝐹𝐹/𝐴𝐴𝐴𝐴𝐴𝐴𝐴𝐴.

We have demonstrated using a planar area, that 𝐶𝐶𝐶𝐶𝑟𝑟𝑟𝑟𝑟𝑟𝑟𝑟𝑟𝑟𝑟𝑟𝑟𝑟𝑟𝑟𝑟𝑟/𝐴𝐴𝐴𝐴𝐴𝐴𝐴𝐴 has a vector-like quality but it
is not a vector in the strictest sense of the word. It is typically referred to as a pseudovector or
spinor. The word fragment spin is associated with circulation/rotation/curl which implies an axis
of rotation.

This concludes our simple visual interpretation of Circulation/Area and Flux/Area which were
expressed mathematically in the form of closed contour integrals. Next, we will define expressions
which are based on differentials that are connected to the concept of 𝐶𝐶𝐶𝐶𝐶𝐶𝐶𝐶𝐶𝐶𝐶𝐶𝐶𝐶𝐶𝐶𝐶𝐶𝐶𝐶𝐶𝐶/𝐴𝐴𝐴𝐴𝐴𝐴𝐴𝐴 and
𝐹𝐹𝐹𝐹𝐹𝐹𝐹𝐹/𝐴𝐴𝐴𝐴𝐴𝐴𝐴𝐴.

1.9 Definitions for the Divergence and Curl Operators in 2d

In this section we will define the divergence of a vector field and the curl of a vector field in 2d.
The curl operation has no meaning in 2d so we will assign a 0 to the 𝑧𝑧 component of an equivalent
3d vector, that is, 〈𝑓𝑓(𝑥𝑥, 𝑦𝑦), 𝑔𝑔(𝑥𝑥, 𝑦𝑦)〉 in 2d becomes 〈𝑓𝑓 (𝑥𝑥, 𝑦𝑦), 𝑔𝑔(𝑥𝑥, 𝑦𝑦), 0〉 in 3d.

Consider the 2d vector field 𝑭𝑭 = 〈𝑥𝑥 − 𝑦𝑦, 𝑦𝑦 + 𝑥𝑥 〉 introduced in Section 1.8. In 3d, this vector can
be written as 𝑭𝑭 = 〈𝑥𝑥 − 𝑦𝑦, 𝑦𝑦 + 𝑥𝑥, 0〉. With this modification we can compute the curl and
divergence of 𝑭𝑭, 𝑭𝑭𝒇𝒇𝒇𝒇𝒇𝒇𝒇𝒇 and 𝑭𝑭𝒄𝒄𝒄𝒄𝒄𝒄𝒄𝒄 that were described in Section 1.8.

60
1.9.1 Divergence of a Vector Field in 2d

The symbol ∇ ∙ is defined as the divergence operator. The symbol ∇ is a vector consisting of the
partial derivatives which operate on a vector field according to the rules for a dot product. The
resultant of a dot product is a scalar entity. The operator ∇ ∙ operating on 𝑭𝑭 , 𝑭𝑭𝒇𝒇𝒇𝒇𝒇𝒇𝒇𝒇 and 𝑭𝑭𝒄𝒄𝒄𝒄𝒄𝒄𝒄𝒄
results in

𝜕𝜕 𝜕𝜕 𝜕𝜕
∇ ∙ 𝑭𝑭 = 〈 , , 〉 ∙ 〈𝑥𝑥 − 𝑦𝑦, 𝑦𝑦 + 𝑥𝑥, 0〉 = 2
𝜕𝜕𝜕𝜕 𝜕𝜕𝜕𝜕 𝜕𝜕𝜕𝜕
𝜕𝜕 𝜕𝜕 𝜕𝜕
∇ ∙ 𝑭𝑭𝒇𝒇𝒇𝒇𝒇𝒇𝒇𝒇 = 〈 , , 〉 ∙ 〈𝑥𝑥, 𝑦𝑦, 0〉 = 2
𝜕𝜕𝜕𝜕 𝜕𝜕𝜕𝜕 𝜕𝜕𝜕𝜕
𝜕𝜕 𝜕𝜕 𝜕𝜕
∇ ∙ 𝑭𝑭𝒄𝒄𝒄𝒄𝒄𝒄𝒄𝒄 = 〈 , , 〉 ∙ 〈−𝑦𝑦, 𝑥𝑥, 0〉 = 0
𝜕𝜕𝜕𝜕 𝜕𝜕𝜕𝜕 𝜕𝜕𝜕𝜕

Note that the results for ∇ ∙ 𝑭𝑭𝒇𝒇𝒇𝒇𝒇𝒇𝒇𝒇 are equivalent to what we obtained in Section 1.8.4 when we
divided the 𝐹𝐹𝐹𝐹𝐹𝐹𝐹𝐹 by the 𝐴𝐴𝐴𝐴𝐴𝐴𝐴𝐴 of the circle with radius 𝑟𝑟.

1.9.2 Curl of a Vector Field in 2d

The symbol ∇ × is defined as the curl operator. The symbol ∇ is a vector consisting of the three
partial derivatives which operate on a vector field according to the rules for a cross product. The
resultant of a cross product is a vector entity. The operator ∇ × operating on 𝑭𝑭 , 𝑭𝑭𝒇𝒇𝒇𝒇𝒇𝒇𝒇𝒇 and 𝑭𝑭𝒄𝒄𝒄𝒄𝒄𝒄𝒄𝒄
results in

𝑥𝑥� 𝑦𝑦� 𝑧𝑧̂


𝜕𝜕 𝜕𝜕 𝜕𝜕
∇ × 𝑭𝑭 = � � = 〈0,0,2〉 = 2𝑧𝑧̂
𝜕𝜕𝜕𝜕 𝜕𝜕𝜕𝜕 𝜕𝜕𝜕𝜕
𝑥𝑥 − 𝑦𝑦 𝑦𝑦 + 𝑥𝑥 0 𝑑𝑑𝑑𝑑𝑑𝑑

𝑥𝑥� 𝑦𝑦� 𝑧𝑧̂


𝜕𝜕 𝜕𝜕 𝜕𝜕
∇ × 𝑭𝑭𝒇𝒇𝒇𝒇𝒇𝒇𝒇𝒇 =� � = 〈0,0,0〉 = 0𝑧𝑧̂
𝜕𝜕𝜕𝜕 𝜕𝜕𝜕𝜕 𝜕𝜕𝜕𝜕
𝑥𝑥 𝑦𝑦 0 𝑑𝑑𝑑𝑑𝑑𝑑

𝑥𝑥� 𝑦𝑦� 𝑧𝑧̂


𝜕𝜕 𝜕𝜕 𝜕𝜕
∇ × 𝑭𝑭𝒄𝒄𝒄𝒄𝒄𝒄𝒄𝒄 =� � = 〈0,0,2〉 = 2𝑧𝑧̂
𝜕𝜕𝜕𝜕 𝜕𝜕𝜕𝜕 𝜕𝜕𝜕𝜕
−𝑦𝑦 𝑥𝑥 0 𝑑𝑑𝑑𝑑𝑑𝑑

61
Note that the results for ∇ × 𝑭𝑭𝒄𝒄𝒄𝒄𝒄𝒄𝒄𝒄 are equivalent to what we obtained in Section 1.8.4 when we
divided the 𝐶𝐶𝐶𝐶𝐶𝐶𝐶𝐶𝐶𝐶𝐶𝐶𝐶𝐶𝐶𝐶𝐶𝐶𝐶𝐶𝐶𝐶 by the 𝐴𝐴𝐴𝐴𝐴𝐴𝐴𝐴 of the circle with radius 𝑟𝑟. In general 𝐶𝐶𝐶𝐶𝐶𝐶𝐶𝐶𝐶𝐶𝐶𝐶𝐶𝐶𝐶𝐶𝐶𝐶𝐶𝐶𝐶𝐶/𝐴𝐴𝐴𝐴𝐴𝐴𝐴𝐴
can depend on the radius of the circle (as well as position). In constrast, ∇ × 𝑭𝑭 only depends on
position.

As just stated, the divergence of a vector field is a scalar which is an agreement with how we
defined a source. The scalar can be negative or positive. If the scalar is positive, then field lines
for 𝑭𝑭𝒇𝒇𝒇𝒇𝒇𝒇𝒇𝒇 emanate from the point source. If the scalar is negative, then the field lines for 𝑭𝑭𝒇𝒇𝒇𝒇𝒇𝒇𝒇𝒇
point inwards to the point source. The choice of the word divergence should now be apparent; the
field lines for 𝑭𝑭𝒇𝒇𝒇𝒇𝒇𝒇𝒇𝒇 diverge as you move outwards from the source. The computed value of 2 for
∇ ∙ 𝑭𝑭𝒇𝒇𝒇𝒇𝒇𝒇𝒇𝒇 is just a number with no practical absolute meaning, for the time being. The convention
of field lines pointing outward for a positive source agrees with the convention that was discussed
in section 1.8. Also, of note, is that the divergence of the circulation field 𝑭𝑭𝒄𝒄𝒄𝒄𝒄𝒄𝒄𝒄 is zero. This
suggests that the divergence operator sifts out only that part of the field that can be attributed
to having source like behavior.

In contrast, the curl of a vector field produces a new vector field. The direction of the resultant
vector field is consistent with the relationship between the direction of circulation and the direction
of the axis of rotation discussed in Section 1.8. Notice that the curl of the flux field 𝑭𝑭𝒇𝒇𝒇𝒇𝒇𝒇𝒇𝒇 is zero.
This suggests that the curl operator sifts out only that part of the field that can be attributed
to having a circulation like behavior.

You can now work on the following problems from section 17.5 of the textbook.

1.9.3 Problems from Section 17.5: 3, 6, 11, 15, 19, 21, 23, 29, 33, 39, 49, 59, 60, 63

1. (question 3) What does it mean if the divergence of a vector field is zero throughout a
region?
2. (question 6) What does it mean if the curl of a vector field is zero throughout a region?
3. (question 11) Divergence of a vector field Find the divergence of the following vector
field: 𝑭𝑭 = 〈12𝑥𝑥, −6𝑦𝑦, −6𝑧𝑧〉.
4. (question 15) Divergence of a vector field Find the divergence of the following vector
〈𝑥𝑥,𝑦𝑦,𝑧𝑧〉
field: 𝑭𝑭 = 1+𝑥𝑥2 +𝑦𝑦2.
5. (question 19) Divergence of a radial field Calculate the divergence of the following radial
field. Express the result in terms of the position vector 𝒓𝒓 and its length |𝒓𝒓|: 𝑭𝑭 =
〈𝑥𝑥,𝑦𝑦,𝑧𝑧〉 𝒓𝒓
(1++𝑦𝑦 2+𝑧𝑧 2 )2
= |𝒓𝒓|4
6. (question 21) Divergence and flux from a graph Consider the following vector field, the
circle 𝐶𝐶 , and two points 𝑃𝑃 and 𝑄𝑄.

62
a. Without computing the divergence, does the graph suggest that the divergence is
positive or negative at 𝑃𝑃 and 𝑄𝑄? Justify your answer.
b. Compute the divergence and confirm your conjecture in part(a).
c. On what part of 𝐶𝐶 is the flux outward? Inward?
d. Is the net outward flux across 𝐶𝐶 positive or negative?
𝑭𝑭 = 〈𝑥𝑥, 𝑥𝑥 + 𝑦𝑦〉

7. (question 23) Curl of a rotation field Consider the following vector field 𝑭𝑭 = 〈1,0,0〉 × 𝒓𝒓
where 𝒓𝒓 = 〈𝑥𝑥, 𝑦𝑦, 𝑧𝑧〉
a. Compute the curl of the vector field and verify that it has the same direction as the
axis of rotation.
b. Compute the magnitude of the curl of the vector field.
8. (question 29) Curl of a vector field Compute the curl of the following vector field: 𝑭𝑭 =
〈𝑥𝑥 2 − 𝑧𝑧 2 , 1,2𝑥𝑥𝑥𝑥〉
9. (question 33) Curl of a vector field Compute the curl of the following vector field: 𝑭𝑭 =
〈𝑧𝑧 2 𝑠𝑠𝑠𝑠𝑠𝑠𝑠𝑠, 𝑥𝑥𝑥𝑥 2 𝑐𝑐𝑐𝑐𝑐𝑐𝑐𝑐, 2𝑥𝑥𝑥𝑥𝑥𝑥𝑥𝑥𝑥𝑥𝑥𝑥〉.
10. (question 39) Explain why or why not Determine whether the following statements are
true and give and explanation or counterexample.
a. For a function f of a single variable, if 𝑓𝑓 ′(𝑥𝑥 ) = 0 for all x in the domain, then 𝑓𝑓 is
a constant function. If ∇ ∙ 𝑭𝑭 = 0 for all points in the domain, then 𝑭𝑭 is a constant.
b. If ∇ × 𝑭𝑭 = 0 then 𝑭𝑭 is a constant.
c. A vector field consisting of parallel vectors has zero curl.
d. A vector field consisting of parallel vectors has zero divergence.
e. curl 𝑭𝑭 is orthogonal to 𝑭𝑭.
11. (question 49) Zero component of the curl For what vectors 𝒏𝒏 is (𝑐𝑐𝑐𝑐𝑐𝑐𝑐𝑐 𝑭𝑭) ∙ 𝒏𝒏 = 0 when
𝑭𝑭 = 〈𝑦𝑦, −2𝑧𝑧, −𝑥𝑥 〉?

63
12. (question 59) Gravitational potential The potential function for the gravitational force
𝐺𝐺𝐺𝐺𝐺𝐺
field due to a mass M at the origin acting on a mass is 𝜙𝜙 = |𝒓𝒓| where 𝒓𝒓 = 〈𝑥𝑥, 𝑦𝑦, 𝑧𝑧〉 is the
position vector of mass 𝑚𝑚 , and 𝐺𝐺 is the gravitational constant.
a. Compute the gravitational force field 𝑭𝑭 = −∇𝜙𝜙.
b. Show that the field is irrotational, that is show that ∇ × 𝑭𝑭 = 𝟎𝟎: Note the different
definition of an irrotational field in the textook
13. (question 60) Electric potential The potential function for the force field is due to a charge
1 𝑞𝑞
𝑞𝑞 at the origin is 𝜙𝜙 = |𝒓𝒓|
, where 𝒓𝒓 = 〈𝑥𝑥, 𝑦𝑦, 𝑧𝑧〉 is the position vector of a point in the
4𝜋𝜋𝜀𝜀𝑜𝑜
field and 𝜀𝜀𝑜𝑜 is the permittivity of free space.
a. Compute the gravitational force field 𝑭𝑭 = −∇𝜙𝜙
b. Show that the field is irrotational, that is show that ∇ × 𝑭𝑭 = 𝟎𝟎: Note the different
definition of an irrotational field in the textbook.
14. (question 63) Ampere’s Law One of Maxwell’s equations for electromagnetic waves is
𝜕𝜕𝑬𝑬
∇ × 𝑯𝑯 = 𝐶𝐶 , where 𝑬𝑬 is the electric field, 𝑯𝑯 is the magnetic field and 𝐶𝐶 is a constant.
𝜕𝜕𝜕𝜕
a. Show that the fields 𝑬𝑬(𝑧𝑧, 𝑡𝑡) = 𝐴𝐴 sin(𝑘𝑘𝑘𝑘 − 𝜔𝜔𝜔𝜔) 𝒊𝒊 and 𝑯𝑯(𝑧𝑧, 𝑡𝑡) = 𝐴𝐴𝐴𝐴𝐴𝐴𝐴𝐴(𝑘𝑘𝑘𝑘 − 𝜔𝜔𝜔𝜔)𝒋𝒋
𝑘𝑘
satisfy the equations for constant 𝐴𝐴, 𝑘𝑘 and 𝜔𝜔 provided 𝜔𝜔 = 𝐶𝐶 .
b. Make a rough sketch showing the directions of 𝑬𝑬 and 𝑯𝑯.

You can find the solutions to the problems at the following link: Solutions to Reading Assignment
4 Problems.

In the next section we unify the concepts of circulation and flux in the form of a theorem referred
to as Helmholtz’s Decomposition Theorem.

1.10 Introduction to Helmholtz’s Decomposition Theorem

From the previous discussion, it should be clear that if the field is rotation free, then the curl of the
vector field is zero. On the other hand, if the field is source free, then the divergence of the vector
field is zero. For any general vector field, the divergence and curl will both exist. In the most
general case, any vector field is the superposition of a vector field which is source free (𝑭𝑭𝑐𝑐𝑐𝑐𝑐𝑐𝑐𝑐 ) and
a vector field which is curl free (𝑭𝑭𝑓𝑓𝑓𝑓𝑓𝑓𝑓𝑓 ). The total field is 𝑭𝑭 = 𝑭𝑭𝑓𝑓𝑓𝑓𝑓𝑓𝑓𝑓 + 𝑭𝑭𝑐𝑐𝑐𝑐𝑐𝑐𝑐𝑐 . This decomposition
of a vector field in terms of its irrotational field component and its source free component is
referred to as Helmholtz’s Decomposition Theorem.

Helmholtz’s Decomposition Theorem

𝑭𝑭 = −∇𝜙𝜙 + ∇ × 𝑨𝑨
↑ ↑
𝑭𝑭 = 𝑭𝑭𝑓𝑓𝑓𝑓𝑓𝑓𝑓𝑓 + 𝑭𝑭𝑐𝑐𝑐𝑐𝑐𝑐𝑐𝑐

64
The source free field (also sometimes referred to as a solenoidal field) is defined by the vector
expression ∇ × 𝑨𝑨 and the irrotational field (also sometimes referred to as a conservative field)
is defined by the vector expression −∇𝜙𝜙. 𝑨𝑨 is defined as the vector potential and 𝜙𝜙 as the scalar
potential. Note: the negative sign appearing in front of ∇𝜙𝜙 is part of the formal definition
but a scalar multiple of 𝛁𝛁𝝓𝝓 is also a conservative field.

Let us now perform the curl and divergence operation on the vector field 𝑭𝑭.

∇ ∙ 𝑭𝑭 = −∇ ∙ ∇𝜙𝜙 + ∇ ∙ (∇ × 𝑨𝑨) = −∇ ∙ ∇𝜙𝜙 = 𝜌𝜌


∇ × 𝑭𝑭 = −∇ × ∇𝜙𝜙 + ∇ × (∇ × 𝑨𝑨) = ∇ × (∇ × 𝑨𝑨) = 𝑱𝑱

Here 𝜌𝜌 is a scalar and 𝑱𝑱 is a vector. If 𝜌𝜌 = 0 then the field 𝑭𝑭 is source free. If 𝑱𝑱 = 𝟎𝟎, then the field
𝑭𝑭 is irrotational.

For a given vector field 𝑭𝑭, the choice of 𝜙𝜙 and 𝑨𝑨 is not unique. In fact, there are an infinite number
of ways to select 𝜙𝜙 and 𝑨𝑨. However, if certain conditions are placed on 𝜌𝜌, 𝑱𝑱 and the boundary of
the domain of interest, then there exists one and only one solution for 𝜙𝜙 and 𝑨𝑨. More details are
provided in sections 4.5 and 4.6 of the expanded set of notes.

Helmholtz’s Decomposition Theorem is the root from which applications in vector calculus are
constructed. We will explore the application aspects in Reading Assignment 5. A more rigorous
theoretical understanding of an irrotational and source free field is outlined in sections 4.3 and 4.4
respectively, in the expanded set of notes. We do not have time in this course to cover the material
in sections 4.3 and 4.4, therefore we summarize the important relationships, in Table 8, but you
are not responsible for it.

Table 8 Differential and Integral Relationships for Irrotational and Source Free Fields

Differential Integral
Irrotational ∇ × 𝑭𝑭 = 0 2d
�𝑭𝑭 ∙ 𝑻𝑻𝑑𝑑𝑑𝑑 = 0
(2d&3d): 𝑭𝑭 = 〈𝑓𝑓 (𝑥𝑥, 𝑦𝑦), 𝑔𝑔(𝑥𝑥, 𝑦𝑦), 0〉 𝐶𝐶
Closed where 𝑭𝑭 = −∇𝜙𝜙
contour
∇ × 𝑭𝑭 = 0 3d
𝑭𝑭 = 〈𝑓𝑓(𝑥𝑥, 𝑦𝑦, 𝑧𝑧), 𝑔𝑔(𝑥𝑥, 𝑦𝑦, 𝑧𝑧), ℎ(𝑥𝑥, 𝑦𝑦, 𝑧𝑧)〉

Irrotational ∇ × 𝑭𝑭 = 0 2d 𝐵𝐵

(2d&3d): 𝑭𝑭 = 〈𝑓𝑓 (𝑥𝑥, 𝑦𝑦), 𝑔𝑔(𝑥𝑥, 𝑦𝑦), 0〉 � 𝑭𝑭 ∙ 𝑻𝑻𝑑𝑑𝑑𝑑 = 𝜙𝜙(𝐵𝐵) − 𝜙𝜙(𝐴𝐴)


𝐴𝐴
open where 𝑭𝑭 = −∇𝜙𝜙
contour

65
∇ × 𝑭𝑭 = 0 3d
𝑭𝑭 = 〈𝑓𝑓(𝑥𝑥, 𝑦𝑦, 𝑧𝑧), 𝑔𝑔(𝑥𝑥, 𝑦𝑦, 𝑧𝑧), ℎ(𝑥𝑥, 𝑦𝑦, 𝑧𝑧)〉

Source free ∇ ∙ 𝑭𝑭 = ∇ ∙ (∇ × 𝑨𝑨) = 0


�𝑭𝑭 ∙ 𝒏𝒏𝑑𝑑𝑑𝑑 = 0
(2d): closed 𝑨𝑨 = 〈0,0, 𝜑𝜑(𝑥𝑥, 𝑦𝑦)〉 𝐶𝐶
contour 𝜕𝜕𝜕𝜕 𝜕𝜕𝜕𝜕 where 𝑭𝑭 = ∇ × 𝑨𝑨
∇ × 𝑨𝑨 = 〈 , − 〉
𝜕𝜕𝜕𝜕 𝜕𝜕𝜕𝜕

Source free ∇ ∙ 𝑭𝑭 = ∇ ∙ (∇ × 𝑨𝑨) = 0 𝐵𝐵

(2d): open 𝑨𝑨 = 〈0,0, 𝜑𝜑(𝑥𝑥, 𝑦𝑦)〉 � 𝑭𝑭 ∙ 𝒏𝒏𝑑𝑑𝑑𝑑 = 𝜑𝜑(𝐵𝐵) − 𝜑𝜑(𝐴𝐴)


𝐴𝐴
contour 𝜕𝜕𝜕𝜕 𝜕𝜕𝜕𝜕 where 𝑭𝑭 = ∇ × 𝑨𝑨
∇ × 𝑨𝑨 = 〈 , − 〉
𝜕𝜕𝜕𝜕 𝜕𝜕𝜕𝜕

Source free ∇ ∙ 𝑭𝑭 = ∇ ∙ (∇ × 𝑨𝑨) = 0


�𝑭𝑭 ∙ 𝒏𝒏𝑑𝑑𝑑𝑑 = 0
(3d): closed 𝑨𝑨 = 〈𝐴𝐴𝑥𝑥 , 𝐴𝐴𝑦𝑦 , 𝐴𝐴𝑧𝑧 〉 𝑆𝑆
surface 𝜕𝜕𝐴𝐴𝑧𝑧 𝜕𝜕𝐴𝐴𝑦𝑦 𝜕𝜕𝐴𝐴𝑥𝑥 𝜕𝜕𝐴𝐴𝑧𝑧 𝜕𝜕𝐴𝐴𝑦𝑦 𝜕𝜕𝐴𝐴𝑥𝑥 where 𝑭𝑭 = ∇ × 𝑨𝑨
𝛁𝛁 × 𝑨𝑨 = 〈 − , − , − 〉
Source free 𝜕𝜕𝜕𝜕 𝜕𝜕𝜕𝜕 𝜕𝜕𝜕𝜕 𝜕𝜕𝜕𝜕 𝜕𝜕𝜕𝜕 𝜕𝜕𝜕𝜕 �𝑭𝑭 ∙ 𝒏𝒏𝑑𝑑𝑑𝑑 = �𝑨𝑨 ∙ 𝑻𝑻𝑑𝑑𝑑𝑑
(3d): open 𝑆𝑆 𝐶𝐶
surface where 𝑭𝑭 = ∇ × 𝑨𝑨
Application of Stokes’
Theorem

Note 1: 𝜙𝜙 is defined as a scalar potential function: 𝜙𝜙(𝑥𝑥, 𝑦𝑦) in 2𝑑𝑑 𝜙𝜙(𝑥𝑥, 𝑦𝑦, 𝑧𝑧) in 3𝑑𝑑
Note 2: 𝜑𝜑(𝑥𝑥, 𝑦𝑦) is defined as a 2d scalar stream function.
Note 3: In 3d it is not possible to define a unique stream function 𝜑𝜑, however it is possible to
define the rotational component of the vector field in terms of two independent stream functions
𝜑𝜑1 and 𝜑𝜑2 , which are functions of 𝑥𝑥, 𝑦𝑦 and 𝑧𝑧, as follows: ∇ × 𝑨𝑨 = ∇𝜑𝜑1 × ∇𝜑𝜑2 . You can show
using a vector identity that ∇ ∙ (∇𝜑𝜑1 × ∇𝜑𝜑2 ) = 0.

1.11 Course Content Going Forward

The Divergence Theorem and Stokes’ theorem are the theorems we will apply most often. Practical
applications of the Divergence Theorem and Stokes’ Theorem are generally posed as 3d problems.
A subset of a general class of problems are problems that exhibit symmetry are considered, in more
detail, in Reading Assignment 5. These problems have geometries which allow you to extract
analytical solutions using the mathematical knowledge you have acquired so far. We also introduce
the reader to Delta Dirac functions since they are required to model point sources, line sources and
surface sources. Finally, we work out examples applying Green’s Circulation Theorem (Stokes’
Theorem in 2d) and Green’s Flux Theorem (Divergence Theorem in 2d), in the second last week
of lectures.

66
2 Part 2: A Formal Description of First Order Differential
Operators

We have now introduced the following three derivative operations: ∇𝜙𝜙, ∇ ∙ 𝑭𝑭 and ∇ × 𝑭𝑭. ∇, ∇ ∙
and ∇ ×, are referred to as operators. They operate on an entity to produce a new entity. The
gradient, which is represented in the form of a vector operator, operates on a scalar function to
produce a vector valued function. The vector operator ∇ ∙ operates on a vector to create a scalar.
The vector operator ∇ × operates on a vector to create a new vector. Note the ∇ symbol is
𝜕𝜕 𝜕𝜕 𝜕𝜕 𝜕𝜕 𝜕𝜕
represented as follows: ∇= 〈 , 〉, (the gradient operator in 2d) and ∇= 〈 , , 〉 (the gradient
𝜕𝜕𝜕𝜕 𝜕𝜕𝜕𝜕 𝜕𝜕𝜕𝜕 𝜕𝜕𝜕𝜕 𝜕𝜕𝜕𝜕
operator in 3d). The gradient is the only operation that has been discussed in any depth, from a
geometric perspective so far.

The purpose of Part 2 of these notes is to provide the rigorous formal definition for the divergence
and curl operations using geometric arguments.

We begin with a short summary of the gradient operator, from a geometric perspective.

2.1 Gradient of a Scalar Potential: 𝛁𝛁𝝓𝝓

The gradient, from a geometrical perspective, is orthogonal to a level curve for a scalar function
of 2 variables and orthogonal to a level surface for a scalar function of 3 variables. From a practical
perspective, it is used in conjunction with the vectors that define the tangential plane, at a given
point on a curved surface, to establish a local coordinate system, which is typically taken to be
Cartesian but not always. The localized coordinate system, which in our example we assume to be
Cartesian, is then used to solve practical problems in kinematics, for example. One can also
interpret the gradient of a function as the direction in which the slope, at a given point, is either
minimized or maximized (equivalent to the magnitude of the gradient vector).

2.2 Divergence and Curl of a Vector Field

Figure 10a (i) shows an infinitesimal 2d square with one vertex located at the point (𝑥𝑥, 𝑦𝑦) in the
first quadrant and the remaining three vertices also located in the first quadrant. Fig. 10a (ii) shows
an infinitesimal cube with one vertex located at the point (𝑥𝑥, 𝑦𝑦, 𝑧𝑧) in the first octant and the
remaining seven vertices also located in the first octant. The infinitesimal cube has 6 faces and
therefore 6 oriented exterior surface areas. Let the size of the cube go to zero. Then the volume of
the cube goes to zero and the surface area of the six faces bounding the cube also goes to zero.

67
Fig. 10a (i) Diagram used to explain concept of divergence and curl of a vector field in 2d

Fig. 10a (ii) Diagram used to explain concept of divergence and curl of a vector field in 3d

We have chosen the reference point of the square and cube to be associated with the vertex that is
located closest to the origin, that is, position (𝑥𝑥, 𝑦𝑦) and (𝑥𝑥, 𝑦𝑦, 𝑧𝑧) respectively. However, the choice
of reference point is irrelevant. It may be convenient to place the reference point at the centroid of
the cube or square. As the size of the square or cube shrinks, the object collapses onto the reference
point, irrespective of the location that represented the reference point for the cube/square. The
infinitesimal cube can exist at any point in 3d space. Likewise, an infinitesimal square can exist at
any point in 2d space.

To formulate the divergence and curl expressions for both cases we start with a position vector
that points from the origin to the vertex of the cube, which in Fig 10a is the vertex located at
(𝑥𝑥, 𝑦𝑦, 𝑧𝑧) in 3d or (𝑥𝑥, 𝑦𝑦) in 2d. Hence, we can state that the position vector to any point inside the
square/cube or on its boundary can be decomposed into two separate vectors: one vector from the
origin to the reference point, which is fixed, and one vector from the reference point to the
boundary of the infinitesimal square or cube or to a point interior to the boundary of the
infinitesimal cube or square. The boundary in 2d is a vertex point or line whereas in 3d the
boundary is a vertex point, line/edge, or points on a surface. Formally we have:

𝑟𝑟⃑ = 𝑟𝑟⃑𝑣𝑣𝑣𝑣𝑣𝑣𝑣𝑣𝑣𝑣𝑣𝑣 + 𝑟𝑟⃑𝑠𝑠𝑠𝑠𝑠𝑠𝑠𝑠𝑠𝑠𝑠𝑠 2d problem

68
𝑟𝑟⃑ = 𝑟𝑟⃑𝑣𝑣𝑣𝑣𝑣𝑣𝑣𝑣𝑣𝑣𝑣𝑣 + 𝑟𝑟⃑𝑐𝑐𝑐𝑐𝑐𝑐𝑐𝑐 3d problem

where 𝑟𝑟⃑𝑣𝑣𝑣𝑣𝑣𝑣𝑣𝑣𝑣𝑣𝑣𝑣 is the vector that points from the physical origin to the reference vertex point
(𝑥𝑥, 𝑦𝑦) of (𝑥𝑥, 𝑦𝑦, 𝑧𝑧) of the square/cube and 𝑟𝑟⃑𝑠𝑠𝑠𝑠𝑠𝑠𝑠𝑠𝑠𝑠𝑠𝑠 or 𝑟𝑟⃑𝑐𝑐𝑐𝑐𝑐𝑐𝑐𝑐 points from the reference vertex point of
the square/cube to any part of the square’s/cube’s exterior boundary (line/line or surface) or interior
(area/volume).

A summary of the position vectors and associated geometries which we will investigate are
summarized below:

2d line 𝑟𝑟⃑𝑣𝑣𝑣𝑣𝑣𝑣𝑣𝑣𝑣𝑣𝑣𝑣 = 〈𝑥𝑥, 𝑦𝑦〉 𝑟𝑟⃑𝑠𝑠𝑠𝑠𝑠𝑠𝑠𝑠𝑠𝑠𝑠𝑠 = 〈𝑥𝑥(𝑡𝑡)𝑠𝑠𝑠𝑠𝑠𝑠𝑠𝑠𝑠𝑠𝑠𝑠 , 𝑦𝑦(𝑡𝑡)𝑠𝑠𝑠𝑠𝑠𝑠𝑠𝑠𝑠𝑠𝑠𝑠 〉


parameterization 𝑎𝑎 ≤ 𝑡𝑡 ≤ 𝑏𝑏

2d surface 𝑟𝑟⃑𝑣𝑣𝑣𝑣𝑣𝑣𝑣𝑣𝑣𝑣𝑣𝑣 = 〈𝑥𝑥, 𝑦𝑦〉 𝑟𝑟⃑𝑠𝑠𝑠𝑠𝑠𝑠𝑠𝑠𝑠𝑠𝑠𝑠 = 〈𝑥𝑥(𝑢𝑢, 𝑣𝑣)𝑠𝑠𝑠𝑠𝑠𝑠𝑠𝑠𝑠𝑠𝑠𝑠 , 𝑦𝑦(𝑢𝑢, 𝑣𝑣)𝑠𝑠𝑠𝑠𝑠𝑠𝑠𝑠𝑠𝑠𝑠𝑠 〉


parameterization 𝑎𝑎 ≤ 𝑢𝑢 ≤ 𝑏𝑏 & 𝑐𝑐 ≤ 𝑣𝑣 ≤ 𝑑𝑑

In both 2d parametrization cases, the vector 〈𝑥𝑥, 𝑦𝑦〉 has constant coefficients and depends on the
location of the reference vertex point of the infinitesimal square.
3d line 𝑟𝑟⃑𝑣𝑣𝑣𝑣𝑣𝑣𝑣𝑣𝑣𝑣𝑣𝑣 = 〈𝑥𝑥, 𝑦𝑦, 𝑧𝑧〉 𝑟𝑟⃑𝑐𝑐𝑐𝑐𝑐𝑐𝑐𝑐 = 〈𝑥𝑥(𝑡𝑡)𝑐𝑐𝑐𝑐𝑐𝑐𝑐𝑐 , 𝑦𝑦(𝑡𝑡)𝑐𝑐𝑐𝑐𝑐𝑐𝑐𝑐 , 𝑧𝑧(𝑡𝑡)𝑐𝑐𝑐𝑐𝑐𝑐𝑐𝑐 〉
parameterization 𝑎𝑎 ≤ 𝑡𝑡 ≤ 𝑏𝑏

3d exterior 𝑟𝑟⃑𝑣𝑣𝑣𝑣𝑣𝑣𝑣𝑣𝑣𝑣𝑣𝑣 = 〈𝑥𝑥, 𝑦𝑦, 𝑧𝑧〉 𝑟𝑟⃑𝑐𝑐𝑐𝑐𝑐𝑐𝑐𝑐 = 〈𝑥𝑥(𝑢𝑢, 𝑣𝑣)𝑐𝑐𝑐𝑐𝑐𝑐𝑐𝑐 , 𝑦𝑦(𝑢𝑢, 𝑣𝑣)𝑐𝑐𝑐𝑐𝑐𝑐𝑐𝑐 , 𝑧𝑧(𝑢𝑢, 𝑣𝑣)𝑐𝑐𝑐𝑐𝑐𝑐𝑐𝑐 〉
surface 𝑎𝑎 ≤ 𝑢𝑢 ≤ 𝑏𝑏 & 𝑐𝑐 ≤ 𝑣𝑣 ≤ 𝑑𝑑
parameterization

3d volume 𝑟𝑟⃑𝑣𝑣𝑣𝑣𝑣𝑣𝑣𝑣𝑣𝑣𝑣𝑣 = 〈𝑥𝑥, 𝑦𝑦, 𝑧𝑧〉 𝑟𝑟⃑𝑐𝑐𝑐𝑐𝑐𝑐𝑐𝑐 = 〈𝑥𝑥(𝑢𝑢, 𝑣𝑣, 𝑤𝑤)𝑐𝑐𝑐𝑐𝑐𝑐𝑐𝑐 , 𝑦𝑦(𝑢𝑢, 𝑣𝑣, 𝑤𝑤)𝑐𝑐𝑐𝑐𝑏𝑏𝑒𝑒 , 𝑧𝑧(𝑢𝑢, 𝑣𝑣, 𝑤𝑤)𝑐𝑐𝑐𝑐𝑐𝑐𝑐𝑐 〉
parameterization 𝑎𝑎 ≤ 𝑢𝑢 ≤ 𝑏𝑏 & 𝑐𝑐 ≤ 𝑣𝑣 ≤ 𝑑𝑑 & 𝑒𝑒 ≤ 𝑤𝑤 ≤ 𝑓𝑓

In all 3d parametrization cases, the vector 〈𝑥𝑥, 𝑦𝑦, 𝑧𝑧〉 has constant coefficients and depends on the
location of the reference vertex point of the infinitesimal cube.

In defining the curl of a vector field, we need to establish the three basis vectors. Choose three
planar surfaces whose direction vectors are orthogonal to each other and are related to the
infinitesimal geometry that was chosen. In the case of a cube, the simplest planes to choose are the
planes associated with the faces of the cube that is the 𝑥𝑥𝑥𝑥 plane, the 𝑦𝑦𝑦𝑦 plane and the 𝑥𝑥𝑥𝑥 plane.

2.3 Definition of Divergence of a Vector Field: 𝛁𝛁 ∙ 𝑭𝑭

Figures 10b and 10c are used to assist in the derivation of the expressions for the divergence of a
vector field 𝑭𝑭 for the infinitesimal square (2d) and infinitesimal cube (3d) respectively.

69
The divergence of a vector field is written as ∇ ∙ 𝑭𝑭 where 𝑭𝑭 = 〈𝑓𝑓 (𝑥𝑥, 𝑦𝑦, 𝑧𝑧), 𝑔𝑔(𝑥𝑥, 𝑦𝑦, 𝑧𝑧), ℎ(𝑥𝑥, 𝑦𝑦, 𝑧𝑧)〉 in
3d and 𝑭𝑭 = 〈𝑓𝑓(𝑥𝑥, 𝑦𝑦), 𝑔𝑔(𝑥𝑥, 𝑦𝑦)〉 in 2d proper or 𝑭𝑭 = 〈𝑓𝑓(𝑥𝑥, 𝑦𝑦), 𝑔𝑔(𝑥𝑥, 𝑦𝑦), 0〉 in pseudo 2d form: takes a
vector valued function and generates a scalar valued function.

2.3.1 2d vector field


𝑭𝑭 = 〈𝑓𝑓(𝑥𝑥, 𝑦𝑦), 𝑔𝑔(𝑥𝑥, 𝑦𝑦), 0〉

Recall from section 1.8.4 that we divided flux ∮𝑪𝑪 𝑭𝑭 ∙ 𝒏𝒏𝒔𝒔 𝑑𝑑𝑑𝑑 by the area 𝐴𝐴 of the enclosure (a circle
in our case) and obtained a number. In effect we were computing an entity related to the divergence
of the vector field, that is ∇ ∙ 𝑭𝑭. In calculus, we define things formally in terms of limits so ∇ ∙ 𝑭𝑭
represents the following in 2d:

∮𝑪𝑪 𝑭𝑭 ∙ 𝒏𝒏𝒔𝒔 𝑑𝑑𝑑𝑑


∇ ∙ 𝑭𝑭 = lim
𝐴𝐴→0 𝐴𝐴
We must integrate over four discrete lines since the exterior contour is not smooth hence

�𝑭𝑭 ∙ 𝒏𝒏𝒔𝒔 𝑑𝑑𝑑𝑑 = � 𝑭𝑭 ∙ 𝒏𝒏𝒔𝒔𝒙𝒙 𝑑𝑑𝑑𝑑 + � 𝑭𝑭 ∙ 𝒏𝒏𝒔𝒔𝒙𝒙+𝒅𝒅𝒅𝒅 𝑑𝑑𝑑𝑑 + � 𝑭𝑭 ∙ 𝒏𝒏𝒔𝒔𝒚𝒚 𝑑𝑑𝑑𝑑 + � 𝑭𝑭 ∙ 𝒏𝒏𝒔𝒔𝒚𝒚+𝒅𝒅𝒅𝒅 𝑑𝑑𝑑𝑑
𝑪𝑪 𝒔𝒔𝒙𝒙 𝒔𝒔𝒙𝒙+𝒅𝒅𝒅𝒅 𝒔𝒔𝒚𝒚 𝒔𝒔𝒚𝒚+𝒅𝒅𝒅𝒅

as per Figure 10b.

Writing out term by term and dividing by 𝐴𝐴 = 𝑑𝑑𝑑𝑑𝑑𝑑𝑑𝑑 we obtain:

∮𝑪𝑪 𝑭𝑭 ∙ 𝒏𝒏𝒔𝒔 𝑑𝑑𝑑𝑑 𝐹𝐹𝑥𝑥 (𝑥𝑥 + 𝑑𝑑𝑑𝑑, 𝑦𝑦)𝑑𝑑𝑑𝑑 − 𝐹𝐹𝑥𝑥 (𝑥𝑥, 𝑦𝑦)𝑑𝑑𝑑𝑑 𝐹𝐹𝑦𝑦 (𝑥𝑥, 𝑦𝑦 + 𝑑𝑑𝑑𝑑)𝑑𝑑𝑑𝑑 − 𝐹𝐹𝑦𝑦 (𝑥𝑥, 𝑦𝑦)𝑑𝑑𝑑𝑑 𝜕𝜕𝐹𝐹𝑥𝑥 𝜕𝜕𝐹𝐹𝑦𝑦
lim = + = +
𝐴𝐴→0 𝐴𝐴 𝑑𝑑𝑑𝑑𝑑𝑑𝑑𝑑 𝑑𝑑𝑑𝑑𝑑𝑑𝑑𝑑 𝜕𝜕𝜕𝜕 𝜕𝜕𝜕𝜕

Fig. 10b Pictorial description of the Divergence theorem in 2d

where 𝒏𝒏𝑺𝑺𝒙𝒙 , 𝒏𝒏𝑺𝑺𝒙𝒙+𝒅𝒅𝒅𝒅 , 𝒏𝒏𝑺𝑺𝒚𝒚 and 𝒏𝒏𝑺𝑺𝒚𝒚+𝒅𝒅𝒅𝒅 represent the unit vectors which are normal and pointing
outwards from the respective four discrete lines.

70
∮𝑪𝑪 𝑭𝑭 ∙ 𝒏𝒏𝒔𝒔 𝑑𝑑𝑑𝑑 represents the flux emanating outward from the closed contour traversed in a counter-
clockwise direction and A represents the area bounded by the closed contour.

2.3.2 3d vector field


𝑭𝑭 = 〈𝑓𝑓 (𝑥𝑥, 𝑦𝑦), 𝑔𝑔(𝑥𝑥, 𝑦𝑦), ℎ(𝑥𝑥, 𝑦𝑦)〉

Geometrically ∇ ∙ 𝑭𝑭 represents the following in 3d:

∯𝑺𝑺 𝑭𝑭 ∙ 𝒏𝒏𝑺𝑺 𝑑𝑑𝑑𝑑


∇ ∙ 𝑭𝑭 = lim
𝑉𝑉→0 𝑉𝑉
We need to integrate over six surfaces since the exterior surface is not smooth hence

�𝑭𝑭 ∙ 𝒏𝒏𝑺𝑺 𝑑𝑑𝑑𝑑 = � 𝑭𝑭 ∙ 𝒏𝒏𝑺𝑺𝒙𝒙 𝑑𝑑𝑑𝑑𝑑𝑑𝑑𝑑 + � 𝑭𝑭 ∙ 𝒏𝒏𝑺𝑺𝒙𝒙+𝒅𝒅𝒅𝒅 𝑑𝑑𝑑𝑑𝑑𝑑𝑑𝑑 + � 𝑭𝑭 ∙ 𝒏𝒏𝑺𝑺𝒚𝒚 𝑑𝑑𝑑𝑑𝑑𝑑𝑑𝑑 +


𝑺𝑺 𝑺𝑺𝒙𝒙 𝑺𝑺𝒙𝒙+𝒅𝒅𝒅𝒅 𝑺𝑺𝒚𝒚

� 𝑭𝑭 ∙ 𝒏𝒏𝑺𝑺𝒚𝒚+𝒅𝒅𝒅𝒅 𝑑𝑑𝑑𝑑𝑑𝑑𝑑𝑑 + � 𝑭𝑭 ∙ 𝒏𝒏𝑺𝑺𝒛𝒛 𝑑𝑑𝑑𝑑𝑑𝑑𝑑𝑑 + � 𝑭𝑭 ∙ 𝒏𝒏𝑺𝑺𝒛𝒛+𝒅𝒅𝒅𝒅 𝑑𝑑𝑑𝑑𝑑𝑑𝑑𝑑


𝑺𝑺𝒚𝒚+𝒅𝒅𝒅𝒅 𝑺𝑺𝒛𝒛 𝑺𝑺𝒛𝒛+𝒅𝒅𝒅𝒅

as per Figure 10c. Writing out term by term and dividing by 𝑉𝑉 = 𝑑𝑑𝑑𝑑𝑑𝑑𝑑𝑑𝑑𝑑𝑑𝑑 we obtain:

∯𝑺𝑺 𝑭𝑭 ∙ 𝒏𝒏𝑺𝑺 𝑑𝑑𝑑𝑑


lim
𝑉𝑉→0 𝑉𝑉
𝐹𝐹𝑥𝑥 (𝑥𝑥 + 𝑑𝑑𝑑𝑑, 𝑦𝑦, 𝑧𝑧)𝑑𝑑𝑑𝑑𝑑𝑑𝑑𝑑 − 𝐹𝐹𝑥𝑥 (𝑥𝑥, 𝑦𝑦, 𝑧𝑧)𝑑𝑑𝑑𝑑𝑑𝑑𝑑𝑑
=
𝑑𝑑𝑑𝑑𝑑𝑑𝑑𝑑𝑑𝑑𝑑𝑑
𝐹𝐹𝑦𝑦 𝑥𝑥, 𝑦𝑦 + 𝑑𝑑𝑑𝑑, 𝑧𝑧 𝑑𝑑𝑑𝑑𝑑𝑑𝑑𝑑 − 𝐹𝐹𝑦𝑦 (𝑥𝑥, 𝑦𝑦, 𝑧𝑧)𝑑𝑑𝑑𝑑𝑑𝑑𝑑𝑑
( )
+
𝑑𝑑𝑑𝑑𝑑𝑑𝑑𝑑𝑑𝑑𝑑𝑑
𝐹𝐹𝑧𝑧 (𝑥𝑥, 𝑦𝑦, 𝑧𝑧 + 𝑑𝑑𝑑𝑑)𝑑𝑑𝑑𝑑𝑑𝑑𝑑𝑑 − 𝐹𝐹𝑧𝑧 (𝑥𝑥, 𝑦𝑦, 𝑧𝑧)𝑑𝑑𝑑𝑑𝑑𝑑𝑑𝑑 𝜕𝜕𝐹𝐹𝑥𝑥 𝜕𝜕𝐹𝐹𝑦𝑦 𝜕𝜕𝐹𝐹𝑧𝑧
+ = + +
𝑑𝑑𝑑𝑑𝑑𝑑𝑑𝑑𝑑𝑑𝑑𝑑 𝜕𝜕𝜕𝜕 𝜕𝜕𝜕𝜕 𝜕𝜕𝜕𝜕

where 𝒏𝒏𝑺𝑺𝒙𝒙 , 𝒏𝒏𝑺𝑺𝒙𝒙+𝒅𝒅𝒅𝒅 , 𝒏𝒏𝑺𝑺𝒚𝒚 , 𝒏𝒏𝑺𝑺𝒚𝒚+𝒅𝒅𝒅𝒅 , 𝒏𝒏𝑺𝑺𝒛𝒛 and 𝒏𝒏𝑺𝑺𝒛𝒛+𝒅𝒅𝒅𝒅 represent the unit vectors which are normal and
pointing outwards from the respective six discrete surfaces. ∯𝑺𝑺 𝑭𝑭 ∙ 𝒏𝒏𝑺𝑺𝑑𝑑𝑑𝑑 represents the flux
emanating outwards from the closed surface that bounds the interior volume V.

In practical terms, the divergence of a vector valued function in the limit indicates if a source exists
at a specific point either in a 2d or 3d form. The physical meaning of the source can take on a
variety of attributes such as a mass or a charge.

71
Fig. 10c Pictorial description of the Divergence theorem in 3d

2.4 Definition of Curl of a Vector Field: 𝛁𝛁 × 𝑭𝑭

2.4.1 2d vector field


𝑭𝑭 = 〈𝑓𝑓(𝑥𝑥, 𝑦𝑦), 𝑔𝑔(𝑥𝑥, 𝑦𝑦), 0〉

Note, the curl operator is only defined in 3d since the curl of a vector field generates a vector field
which is orthogonal to the plane of rotation. So, in 2d, rotation is in the 𝑥𝑥𝑥𝑥 plane but the axis of
rotation is orthogonal to this plane and is directed along the z axis which does not exist in 2d. One
way of still using the curl operator, under these circumstances, is to convert a 2d vector field into
a 3d vector field by assigning zero to the z component. In essence we have created a 3d vector
field which has the properties of a pseudo 2d vector field. The reason for having to resort to a work
around solution stems from the fact that there were two competing theories of vector calculus in
the late 1800s.

Recall from section 1.8.4 that we divided circulation ∮𝐶𝐶 𝑭𝑭 ∙ 𝑻𝑻𝑑𝑑𝑑𝑑 by the area 𝐴𝐴 of the enclosure (a
circle in our case) and obtained a number. In effect we were computing an entity related to the curl
of the vector field, that is ∇ × 𝑭𝑭. In calculus, we define things formally in terms of limits so ∇ × 𝑭𝑭
represents the following in 2d.

You take a vector valued function and generate another vector valued function that represents the
magnitude of rotation about the axis of rotation which is in the fictitious 𝑧𝑧 direction at a given
point on the 𝑥𝑥𝑥𝑥 plane. The formal definition of the 𝑧𝑧 − component of the curl of a vector field is
as follows:

72
∮𝐶𝐶 𝑭𝑭 ∙ 𝑻𝑻𝑑𝑑𝑑𝑑
(∇ × 𝑭𝑭)𝑧𝑧 = lim
𝐴𝐴𝑧𝑧� →0 𝐴𝐴𝑧𝑧̂

One must integrate over four-line segments since the exterior contour is not smooth. Hence

�𝑭𝑭 ∙ 𝑻𝑻𝑑𝑑𝑑𝑑 = � 𝑭𝑭 ∙ 𝑻𝑻𝒔𝒔𝒙𝒙 𝑑𝑑𝑑𝑑 + � 𝑭𝑭 ∙ 𝑻𝑻𝒔𝒔𝒙𝒙+𝒅𝒅𝒅𝒅 𝑑𝑑𝑑𝑑 + � 𝑭𝑭 ∙ 𝑻𝑻𝒔𝒔𝒚𝒚 𝑑𝑑𝑑𝑑 + � 𝑭𝑭 ∙ 𝑻𝑻𝒔𝒔𝒚𝒚+𝒅𝒅𝒅𝒅 𝑑𝑑𝑑𝑑
𝑪𝑪 𝒔𝒔𝒙𝒙 𝒔𝒔𝒙𝒙+𝒅𝒅𝒅𝒅 𝒔𝒔𝒚𝒚 𝒔𝒔𝒚𝒚+𝒅𝒅𝒅𝒅

Fig. 10d Pictorial description of Stokes’ theorem in 2d: (𝛁𝛁 × 𝑭𝑭)𝒛𝒛

as per Figure 10d. ∮𝐶𝐶 𝑭𝑭 ∙ 𝑻𝑻𝑑𝑑𝑑𝑑 represents the circulation when traversing the complete contour in a
counter-clockwise direction and 𝐴𝐴𝑧𝑧̂ is the area bounded by the closed contour. In the limit as 𝐴𝐴𝑧𝑧̂ →
0, we compute the 𝑧𝑧 − component of the circulation at a given point on the 𝑥𝑥𝑥𝑥 plane. The axis of
rotation in the 2d case is pointing in the 𝑧𝑧 − direction which is consistent with the right-hand
convention.

Writing out term by term and dividing by 𝐴𝐴𝑧𝑧̂ = 𝑑𝑑𝑑𝑑𝑑𝑑𝑑𝑑 we obtain:

∮𝐶𝐶 𝑭𝑭 ∙ 𝑻𝑻𝑑𝑑𝑑𝑑 𝐹𝐹𝑦𝑦 (𝑥𝑥 + 𝑑𝑑𝑑𝑑, 𝑦𝑦, 0)𝑑𝑑𝑑𝑑 − 𝐹𝐹𝑦𝑦 (𝑥𝑥, 𝑦𝑦, 0)𝑑𝑑𝑑𝑑 𝐹𝐹𝑥𝑥 (𝑥𝑥, 𝑦𝑦 + 𝑑𝑑𝑑𝑑, 0)𝑑𝑑𝑑𝑑 − 𝐹𝐹𝑥𝑥 (𝑥𝑥, 𝑦𝑦, 0)𝑑𝑑𝑑𝑑
lim = −
𝐴𝐴𝑧𝑧� →0 𝐴𝐴𝑧𝑧̂ 𝑑𝑑𝑑𝑑𝑑𝑑𝑑𝑑 𝑑𝑑𝑑𝑑𝑑𝑑𝑑𝑑
𝜕𝜕𝐹𝐹𝑦𝑦 𝜕𝜕𝐹𝐹𝑥𝑥
= −
𝜕𝜕𝜕𝜕 𝜕𝜕𝜕𝜕

where 𝑻𝑻 represents the unit vector tangent to the closed contour and oriented in a counter-
clockwise sense. The integration must be broken into 4 discrete segments (the tangent vector 𝑻𝑻 is
not smooth (discontinuous)).

2.4.2 3d vector field


𝑭𝑭 = 〈𝑓𝑓(𝑥𝑥, 𝑦𝑦, 𝑧𝑧), 𝑔𝑔(𝑥𝑥, 𝑦𝑦, 𝑧𝑧), ℎ(𝑥𝑥, 𝑦𝑦, 𝑧𝑧)〉

73
The geometrical interpretation of ∇ × 𝑭𝑭 is equivalent to what was described above however the
contour is no longer constrained to the 𝑥𝑥𝑥𝑥 plane. To properly identify the vector, we need to
compute the area component pointing in the 𝑥𝑥�, 𝑦𝑦� and 𝑧𝑧̂ direction. The calculation for the
component in the 𝑥𝑥� direction is performed next where 𝐴𝐴𝑥𝑥� represents the area of the face whose
direction vector is in the 𝑥𝑥� direction. Refer to Fig 10e (i) for the pictorial description.

∮𝐶𝐶 𝑭𝑭 ∙ 𝑻𝑻𝑑𝑑𝑑𝑑
(∇ × 𝑭𝑭)𝑥𝑥 = lim
𝐴𝐴𝑥𝑥� →0 𝐴𝐴𝑥𝑥�

where the direction of the contour C is chosen based on the right-hand rule to be consistent with
the normal vector 𝑥𝑥�

We must integrate over four lines since the exterior contour is not smooth hence

�𝑭𝑭 ∙ 𝑻𝑻𝑑𝑑𝑑𝑑 = � 𝑭𝑭 ∙ 𝑻𝑻𝒔𝒔𝒛𝒛 𝑑𝑑𝑑𝑑 + � 𝑭𝑭 ∙ 𝑻𝑻𝒔𝒔𝒛𝒛+𝒅𝒅𝒅𝒅 𝑑𝑑𝑑𝑑 + � 𝑭𝑭 ∙ 𝑻𝑻𝒔𝒔𝒚𝒚 𝑑𝑑𝑑𝑑 + � 𝑭𝑭 ∙ 𝑻𝑻𝒔𝒔𝒚𝒚+𝒅𝒅𝒅𝒅 𝑑𝑑𝑑𝑑
𝑪𝑪 𝒔𝒔𝒛𝒛 𝒔𝒔𝒛𝒛+𝒅𝒅𝒅𝒅 𝒔𝒔𝒚𝒚 𝒔𝒔𝒚𝒚+𝒅𝒅𝒅𝒅

∮𝐶𝐶 𝑭𝑭 ∙ 𝑻𝑻𝑑𝑑𝑑𝑑 represents the circulation when traversing the complete contour in a counterclockwise
direction and 𝐴𝐴𝑥𝑥� is the area in the 𝑦𝑦𝑦𝑦 plane bounded by the closed contour.

Writing out term by term and dividing by 𝐴𝐴𝑥𝑥� = 𝑑𝑑𝑑𝑑𝑑𝑑𝑑𝑑 we obtain:

∮𝐶𝐶 𝑭𝑭 ∙ 𝑻𝑻𝑑𝑑𝑑𝑑 𝐹𝐹𝑧𝑧 (𝑥𝑥, 𝑦𝑦 + 𝑑𝑑𝑑𝑑, 𝑧𝑧)𝑑𝑑𝑑𝑑 − 𝐹𝐹𝑧𝑧 (𝑥𝑥, 𝑦𝑦, 𝑧𝑧)𝑑𝑑𝑑𝑑 𝐹𝐹𝑦𝑦 (𝑥𝑥, 𝑦𝑦, 𝑧𝑧 + 𝑑𝑑𝑑𝑑)𝑑𝑑𝑑𝑑 − 𝐹𝐹𝑦𝑦 (𝑥𝑥, 𝑦𝑦, 𝑧𝑧)𝑑𝑑𝑑𝑑
lim = −
𝐴𝐴𝑥𝑥� →0 𝐴𝐴𝑥𝑥� 𝑑𝑑𝑑𝑑𝑑𝑑𝑑𝑑 𝑑𝑑𝑑𝑑𝑑𝑑𝑑𝑑
𝜕𝜕𝐹𝐹𝑧𝑧 𝜕𝜕𝐹𝐹𝑦𝑦
= −
𝜕𝜕𝜕𝜕 𝜕𝜕𝜕𝜕

where 𝑻𝑻 represents the unit vector which is tangent to the closed contour and is oriented in a
counter-clockwise sense. The integration must be broken into 4 discrete segments (contour is
discontinuous) and only two of these segments contribute to a non-zero dot product 𝑭𝑭 ∙ 𝑻𝑻.

The expressions for (∇ × 𝑭𝑭)𝑦𝑦 and (∇ × 𝑭𝑭)𝑧𝑧 are found in the same way, with the assistance of Fig.
10e (ii) and Fig. 10e (iii) respectively. It is left as an exercise for the student to write out the
expressions for (∇ × 𝑭𝑭)𝑦𝑦 and (∇ × 𝑭𝑭)𝑧𝑧 . In summary, we have probed the vectorial components of
the circulation at a given point (limiting process) in all three orthogonal directions. The sum of the
three orthogonal vectors is the resultant vector representing the curl of the vector field, that is
∇ × 𝑭𝑭. We chose a simple way to illustrate the derivation of the curl of a vector field, but one could
have used other shapes as discussed briefly in section 2.6, of the expanded set of notes.

74
(i) The x component of the curl (𝛁𝛁 × 𝑭𝑭)𝒙𝒙

(ii) The y component of the curl (𝛁𝛁 × 𝑭𝑭)𝒚𝒚

(iii) The z component of curl (𝛁𝛁 × 𝑭𝑭)𝒛𝒛

Fig. 10e Pictorial description of Stokes’ theorem in 3d

2.5 Exercise: Compute (𝛁𝛁 × 𝑭𝑭)𝒚𝒚 and (𝛁𝛁 × 𝑭𝑭)𝒛𝒛

Write out the expressions for (∇ × 𝑭𝑭)𝑦𝑦 and (∇ × 𝑭𝑭)𝑧𝑧 : refer to the Archived Tutorial 7 Video.

75
2.6 Impact of the Infinitesimal Volume’s Shape

As previously mentioned, the shape of the infinitesimal volume used to develop the expressions
for the divergence and curl of a vector field is irrelevant. A cubic structure turned out be simple
since along any line or surface only one field component contributed to the dot product, however
four segments were required to describe a closed contour and six planar faces were required to
represent the closed surface.

Alternatively, if we were to use an infinitesimal sphere, rather than a cube, as shown in Fig. 11,
then any closed contour on the exterior of the sphere can be represented by one smooth curve. The
sphere which is made infinitesimal in size has its centre located at some arbitrary position 𝑟𝑟⃑𝑜𝑜 =
〈𝑥𝑥𝑜𝑜 , 𝑦𝑦𝑜𝑜 , 𝑧𝑧𝑜𝑜 〉 in space.

More details are provided in the expanded set of notes including two optional questions involving
the computation of the divergence and curl of the vector field assuming an infinitesimal spherical
volume.

Fig. 11 Interpretation of 𝛁𝛁 × 𝑭𝑭 in 3d

3 Part 3: Vector Identities

Throughout this course we will be using identities in a proof or as a utility to simplify a calculation.
The identities are grouped in terms of zero order derivative operations, first order derivative
operations, second order derivative operations and third order derivative operations. These
identities have been discovered either by accident or by manipulating expressions to look for
underlying patterns. Once these expressions are found it becomes a relatively easy task to formally

76
prove them by working out the details of the left-hand side and right-hand side of the expressions
and see that they are equivalent to each other.

Vector operators apply in any coordinate system however the detailed expressions for each
component of the vector field will look different depending on what coordinate system is used. In
this course we will only work in a Cartesian coordinate system however in ECE221S next term
you will be considering the vector operators in other coordinate systems, for example, in
cylindrical coordinates. The specifics of converting vector operators from one coordinate system
to another is described in more detail in the expanded set of notes in Section 4 of Reading
Assignment 2.

A sampling of the frequently used identities is introduced next, and the expressions are coordinate
independent. The identities marked in red are the ones we will refer to more frequently in the
course and so having them all stated here serves as a point of reference. There are exercises at the
end of this section where you are asked to work out the proofs, for the examples marked in red.

3.1 Scalars or Vectors Generated by Combinations of Addition, Dot


Products and Cross Products

𝑨𝑨 + 𝑩𝑩 = 𝑩𝑩 + 𝑨𝑨

𝑨𝑨 ∙ 𝑩𝑩 = 𝑩𝑩 ∙ 𝑨𝑨

𝑨𝑨 × 𝑩𝑩 = −𝑩𝑩 × 𝑨𝑨

(𝑨𝑨 + 𝑩𝑩) ∙ 𝑪𝑪 = 𝑨𝑨 ∙ 𝑪𝑪 + 𝑩𝑩 ∙ 𝑪𝑪

(𝑨𝑨 + 𝑩𝑩) × 𝑪𝑪 = 𝑨𝑨 × 𝑪𝑪 + 𝑩𝑩 × 𝑪𝑪

𝑨𝑨 ∙ (𝑩𝑩 × 𝑪𝑪) = 𝑩𝑩 ∙ (𝑪𝑪 × 𝑨𝑨) = 𝑪𝑪 ∙ (𝑨𝑨 × 𝑩𝑩) (scalar triple product)

𝑨𝑨 × (𝑩𝑩 × 𝑪𝑪) = (𝑨𝑨 ∙ 𝑪𝑪)𝑩𝑩 − (𝑨𝑨 ∙ 𝑩𝑩)𝑪𝑪 (vector triple product)

(𝑨𝑨 × 𝑩𝑩) × 𝑪𝑪 = (𝑨𝑨 ∙ 𝑪𝑪)𝑩𝑩 − (𝑩𝑩 ∙ 𝑪𝑪)A (vector triple product)

𝑨𝑨 × (𝑩𝑩 × 𝑪𝑪) = (𝑨𝑨 × 𝑩𝑩) × 𝑪𝑪 + 𝑩𝑩 × (𝑨𝑨 × 𝑪𝑪) (Jacobi identity)

𝑨𝑨 × (𝑩𝑩 × 𝑪𝑪) + 𝑪𝑪 × (𝑨𝑨 × 𝑩𝑩) + 𝑩𝑩 × (𝑪𝑪 × 𝑨𝑨) = 0 (Jacobi identity)

(𝑨𝑨 × 𝑩𝑩) ∙ (𝑪𝑪 × 𝑫𝑫) = (𝑨𝑨 ∙ 𝑪𝑪)(𝑩𝑩 ∙ 𝑫𝑫) − (𝑩𝑩 ∙ 𝑪𝑪)(𝑨𝑨 ∙ 𝑫𝑫)

�𝑨𝑨 ∙ (𝑩𝑩 × 𝑪𝑪)�𝑫𝑫 = (𝑨𝑨 ∙ 𝑫𝑫)(𝑩𝑩 × 𝑪𝑪) + (𝑩𝑩 ∙ 𝑫𝑫)(𝑪𝑪 × 𝑨𝑨) + (𝑪𝑪 ∙ 𝑫𝑫)(𝑨𝑨 × 𝑩𝑩)

(𝑨𝑨 × 𝑩𝑩) × (𝑪𝑪 × 𝑫𝑫) = �𝑨𝑨 ∙ (𝑩𝑩 × 𝑫𝑫)�𝑪𝑪 − �𝑨𝑨 ∙ (𝑩𝑩 × 𝑪𝑪)�𝑫𝑫

77
3.2 Differentiation

Differentiation here means operations performed using a gradient operator (converting a scalar
function into a vector valued function), divergence operator (converting a vector valued function
into a scalar function) and the curl operator (converting a vector valued function into another vector
valued function.

3.2.1 Gradient

∇(𝜓𝜓 + ϕ) = ∇(𝜓𝜓) + ∇(ϕ)

∇(𝜓𝜓ϕ) = ϕ∇(𝜓𝜓) + 𝜓𝜓∇(ϕ)

∇(𝑨𝑨 ∙ 𝑩𝑩) = (𝑨𝑨 ∙ ∇)𝑩𝑩 + (𝑩𝑩 ∙ ∇)𝑨𝑨 + 𝑨𝑨 × (∇ × 𝑩𝑩) + 𝑩𝑩 × (∇ × 𝑨𝑨)

Note that (𝑩𝑩 ∙ ∇)𝑨𝑨 or (𝑨𝑨 ∙ ∇)𝑩𝑩 is not the same as (∇ ∙ 𝑩𝑩)𝑨𝑨 or (∇ ∙ 𝑨𝑨)𝑩𝑩. In the former case, for
example (𝑩𝑩 ∙ ∇)𝑨𝑨, the dot product acts as an operator that operates on every component of the
vector 𝑨𝑨. Written out in full we obtain:

𝜕𝜕 𝜕𝜕 𝜕𝜕 𝜕𝜕 𝜕𝜕 𝜕𝜕
(𝑩𝑩 ∙ ∇)𝑨𝑨 = �〈𝐵𝐵𝑥𝑥 , 𝐵𝐵𝑦𝑦 , 𝐵𝐵𝑧𝑧 〉 ∙ 〈 , , 〉� 〈𝐴𝐴𝑥𝑥 , 𝐴𝐴𝑦𝑦 , 𝐴𝐴𝑧𝑧 〉 = �𝐵𝐵𝑥𝑥 + 𝐵𝐵𝑦𝑦 + 𝐵𝐵𝑧𝑧 � 〈𝐴𝐴𝑥𝑥 , 𝐴𝐴𝑦𝑦 , 𝐴𝐴𝑧𝑧 〉
𝜕𝜕𝜕𝜕 𝜕𝜕𝜕𝜕 𝜕𝜕𝜕𝜕 𝜕𝜕𝜕𝜕 𝜕𝜕𝜕𝜕 𝜕𝜕𝜕𝜕

𝜕𝜕𝐴𝐴𝑥𝑥 𝜕𝜕𝐴𝐴𝑥𝑥 𝜕𝜕𝐴𝐴𝑥𝑥 𝜕𝜕𝐴𝐴𝑦𝑦 𝜕𝜕𝐴𝐴𝑦𝑦 𝜕𝜕𝐴𝐴𝑦𝑦 𝜕𝜕𝐴𝐴𝑧𝑧 𝜕𝜕𝐴𝐴𝑧𝑧 𝜕𝜕𝐴𝐴𝑧𝑧


= 〈𝐵𝐵𝑥𝑥 + 𝐵𝐵𝑦𝑦 + 𝐵𝐵𝑧𝑧 , 𝐵𝐵𝑥𝑥 + 𝐵𝐵𝑦𝑦 + 𝐵𝐵𝑧𝑧 , 𝐵𝐵𝑥𝑥 + 𝐵𝐵𝑦𝑦 + 𝐵𝐵𝑧𝑧 〉
𝜕𝜕𝜕𝜕 𝜕𝜕𝜕𝜕 𝜕𝜕𝜕𝜕 𝜕𝜕𝜕𝜕 𝜕𝜕𝜕𝜕 𝜕𝜕𝜕𝜕 𝜕𝜕𝜕𝜕 𝜕𝜕𝜕𝜕 𝜕𝜕𝜕𝜕

In the latter case, ∇ ∙ 𝑩𝑩 produces a scalar which multiplies each component of the vector 𝑨𝑨. Written
out in full we obtain:

𝜕𝜕 𝜕𝜕 𝜕𝜕 𝜕𝜕𝐵𝐵𝑥𝑥 𝜕𝜕𝐵𝐵𝑦𝑦 𝜕𝜕𝐵𝐵𝑧𝑧


(∇ ∙ 𝑩𝑩)𝑨𝑨 = �〈 , , 〉 ∙ 〈𝐵𝐵𝑥𝑥 , 𝐵𝐵𝑦𝑦 , 𝐵𝐵𝑧𝑧 〉� 〈𝐴𝐴𝑥𝑥 , 𝐴𝐴𝑦𝑦 , 𝐴𝐴𝑧𝑧 〉 = � + + � 〈𝐴𝐴𝑥𝑥 , 𝐴𝐴𝑦𝑦 , 𝐴𝐴𝑧𝑧 〉
𝜕𝜕𝜕𝜕 𝜕𝜕𝜕𝜕 𝜕𝜕𝜕𝜕 𝜕𝜕𝜕𝜕 𝜕𝜕𝜕𝜕 𝜕𝜕𝜕𝜕
𝜕𝜕𝐵𝐵𝑥𝑥 𝜕𝜕𝐵𝐵𝑦𝑦 𝜕𝜕𝐵𝐵𝑧𝑧 𝜕𝜕𝐵𝐵𝑥𝑥 𝜕𝜕𝐵𝐵𝑦𝑦 𝜕𝜕𝐵𝐵𝑧𝑧 𝜕𝜕𝐵𝐵𝑥𝑥 𝜕𝜕𝐵𝐵𝑦𝑦 𝜕𝜕𝐵𝐵𝑧𝑧
= 〈� + + � 𝐴𝐴𝑥𝑥 , � + + � 𝐴𝐴𝑦𝑦 , � + + � 𝐴𝐴𝑧𝑧 〉
𝜕𝜕𝜕𝜕 𝜕𝜕𝜕𝜕 𝜕𝜕𝜕𝜕 𝜕𝜕𝜕𝜕 𝜕𝜕𝜕𝜕 𝜕𝜕𝜕𝜕 𝜕𝜕𝜕𝜕 𝜕𝜕𝜕𝜕 𝜕𝜕𝜕𝜕

Notice that the operation (𝑩𝑩 ∙ ∇)𝑨𝑨 and (∇ ∙ 𝑩𝑩)𝑨𝑨 result in vectors which are not the same.

3.2.2 Divergence

∇ ∙ (𝑨𝑨 + 𝑩𝑩) = ∇ ∙ (𝑨𝑨) + ∇ ∙ (𝑩𝑩) (linearity identity/superposition)

∇ ∙ (𝜓𝜓𝑨𝑨) = 𝜓𝜓∇ ∙ 𝑨𝑨 + 𝑨𝑨 ∙ ∇𝜓𝜓

∇ ∙ (𝑨𝑨 × 𝑩𝑩) = 𝑩𝑩 ∙ (∇ × 𝑨𝑨) − 𝐀𝐀 ∙ (∇ × 𝑩𝑩)

78
3.2.3 Curl

∇ × (𝑨𝑨 + 𝑩𝑩) = ∇ × 𝑨𝑨 + ∇ × 𝑩𝑩 (linearity identity/superposition)

∇ × (𝜓𝜓𝑨𝑨) = 𝜓𝜓∇ × 𝑨𝑨 + ∇𝜓𝜓 × 𝑨𝑨

∇ × (𝑨𝑨 × 𝑩𝑩) = 𝑨𝑨(∇ ∙ 𝑩𝑩) − 𝑩𝑩(∇ ∙ 𝑨𝑨) + (𝑩𝑩 ∙ ∇)𝑨𝑨 − (𝑨𝑨 ∙ ∇)𝑩𝑩

3.2.4 Second derivatives

∇ × (𝜓𝜓∇ϕ) = ∇𝜓𝜓 × ∇ϕ

∇ ∙ (∇ × 𝑨𝑨) = 𝟎𝟎 (source free field)

∇ × (∇𝜓𝜓) = 𝟎𝟎 (rotation free field)

∇ ∙ (∇𝜓𝜓) = ∇2 𝜓𝜓 (scalar Laplacian)

∇(∇ ∙ 𝑨𝑨) − ∇ × (∇ × 𝑨𝑨) = ∇2 𝑨𝑨 (vector Laplacian or Lagrange’s formula for ∇)

∇ ∙ (ϕ∇𝜓𝜓) = ϕ∇2 𝜓𝜓 + ∇𝜓𝜓 ∙ ∇ϕ

𝜓𝜓∇2 ϕ − ϕ∇2 𝜓𝜓 = ∇ ∙ (𝜓𝜓∇ϕ − ϕ∇𝜓𝜓)

∇2 (ϕ𝜓𝜓) = ϕ∇2 𝜓𝜓 + 2∇ϕ ∙ ∇𝜓𝜓 + 𝜓𝜓∇2 ϕ

∇2 (𝜓𝜓𝑨𝑨) = 𝑨𝑨∇2 𝜓𝜓 + 2(∇𝜓𝜓 ∙ ∇)𝑨𝑨 + 𝜓𝜓∇2 𝑨𝑨

∇2 (𝑨𝑨 ∙ 𝑩𝑩) = 𝑨𝑨 ∙ ∇2 𝑩𝑩 − 𝑩𝑩 ∙ ∇2 𝑨𝑨 + 2∇ ∙ �(𝑩𝑩 ∙ 𝛁𝛁)𝑨𝑨 + 𝑩𝑩 × 𝛁𝛁 × 𝑨𝑨� (Green’s vector identity)

3.2.5 Third derivatives

∇2 (∇𝜓𝜓) = ∇�∇ ∙ (∇𝜓𝜓)� = ∇(∇2 𝜓𝜓)

∇2 (∇ ∙ 𝑨𝑨) = ∇ ∙ �∇(∇ ∙ 𝑨𝑨)� = ∇ ∙ (∇2 𝑨𝑨)

∇2 (∇ × 𝑨𝑨) = −∇ × �∇ × (∇ × 𝑨𝑨)� = ∇ × (∇2 𝑨𝑨)

Miscellaneous

�(𝑪𝑪 ∙ ∇)𝑩𝑩� ∙ 𝑨𝑨 − (𝑪𝑪 ∙ 𝑨𝑨)(∇ ∙ 𝑩𝑩) = 𝑪𝑪 ∙ �(𝑨𝑨 × ∇) × 𝑩𝑩�

3.3 Exercises to Prove Vector Identities

Prove the following identities by expanding out the right-hand and left-hand sides for each
expression and equating the results:

79
1. 𝑨𝑨 ∙ (𝑩𝑩 × 𝑪𝑪) = 𝑩𝑩 ∙ (𝑪𝑪 × 𝑨𝑨) = 𝑪𝑪 ∙ (𝑨𝑨 × 𝑩𝑩) (scalar triple product)
2. ∇ ∙ (𝑨𝑨 + 𝑩𝑩) = ∇ ∙ (𝑨𝑨) + 𝛁𝛁 ∙ (𝑩𝑩) (linearity identity)
3. ∇ × (𝑨𝑨 + 𝑩𝑩) = ∇ × 𝑨𝑨 + ∇ × 𝑩𝑩 (linearity identity)
4. ∇ ∙ (∇ × 𝑨𝑨) = 0 (source free field)
5. ∇ × (∇𝜓𝜓) = 𝟎𝟎 (rotation free field)
2
6. ∇ ∙ ∇𝜓𝜓 = ∇ 𝜓𝜓 (scalar Laplacian
( )
7. ∇(∇ ∙ 𝑨𝑨) − ∇ × (∇ × 𝑨𝑨) = ∇2 𝑨𝑨 (vector Laplacian or Lagrange’s formula for ∇)
8. ∇(𝑨𝑨 ∙ 𝑩𝑩) = (𝑨𝑨 ∙ ∇)𝑩𝑩 + (𝑩𝑩 ∙ ∇)𝑨𝑨 + 𝑨𝑨 × (∇ × 𝑩𝑩) + 𝑩𝑩 × (∇ × 𝑨𝑨)
9. �(𝑪𝑪 ∙ ∇)𝑩𝑩� ∙ 𝑨𝑨 − (𝑪𝑪 ∙ 𝑨𝑨)(∇ ∙ 𝑩𝑩) = 𝑪𝑪 ∙ �(𝑨𝑨 × ∇) × 𝑩𝑩�
10. (question 45 from Section 17.5 of textbook) Curl of the rotation field For the general
rotation field 𝑭𝑭 = 𝒂𝒂 × 𝒓𝒓 where 𝒂𝒂 = 〈𝑎𝑎1 , 𝑎𝑎2 , 𝑎𝑎3 〉 is a nonzero constant vector and 𝒓𝒓 =
〈𝑥𝑥, 𝑦𝑦, 𝑧𝑧〉, show that curl 𝑭𝑭 = 2𝒂𝒂.
11. (question 61 from Section 17.5 of textbook) Navier-Stokes equation The Navier-Stokes
equation is the fundamental equation of fluid dynamics that models the flow in everything
from bathtubs to oceans. In one of its many forms, (incompressible viscous flow), the
𝜕𝜕𝑽𝑽
equation is 𝜌𝜌 � + (𝑽𝑽 ∙ ∇)𝑽𝑽� = −∇𝑝𝑝 + 𝜇𝜇(∇ ∙ ∇)𝑽𝑽. In this notation, 𝑽𝑽 =
𝜕𝜕𝜕𝜕
〈𝑢𝑢(𝑥𝑥, 𝑦𝑦, 𝑧𝑧), 𝑣𝑣(𝑥𝑥, 𝑦𝑦, 𝑧𝑧), 𝑤𝑤(𝑥𝑥, 𝑦𝑦, 𝑧𝑧)〉 is the three-dimensional velocity field, 𝑝𝑝 is the (scalar
pressure), 𝜌𝜌 is the constant mass density of the fluid and 𝜇𝜇 is the constant viscosity. Write
out the three component equations of this vector equation.

You can find the solutions to the problems at the following link: Solutions to Reading Assignment
4 Problems.

4 Part 4: The Divergence Theorem and Stokes’ Theorem,


Geometric Characterization of Fields, and a Sampling of Physical
Laws
In this section of the notes, we introduce the basic theorems that constitute Vector Calculus. The
key theorems and caveats to be considered are:

• The Divergence Theorem in 3d and Green’s Flux Theorem in 2d


• Stokes’ Theorem in 3d and Green’s Circulation Theorem in 2d
• The Fundamental Theorem for Line Integrals
 Irrotational field (2d and 3d)
 Source free field (2d)
• Techniques for applying the Divergence Theorem, Stokes’ Theorem and Green’s
Theorems under Problematic Conditions

80
In applying these theorems, we will need to apply conventions, which are due to our adoption of a
right-handed coordinate system. The theorems, conventions and an illustration that complements
each convention is shown in Table 11.

Each theorem stems from our observation of the two patterns exhibited by a vector field, that is
𝐶𝐶𝐶𝐶𝐶𝐶𝐶𝐶𝐶𝐶𝐶𝐶𝐶𝐶𝐶𝐶𝐶𝐶𝐶𝐶𝐶𝐶 and 𝐹𝐹𝐹𝐹𝐹𝐹𝐹𝐹, as described in Section 1.8. Also, we add to this mix the interesting
geometric observation that two endpoints bound a line, a closed contour bounds an area, and a
closed surface bounds a volume.

You may notice that there is no Fundamental Theorem for Line Integrals for the 3d source free
case. We will briefly discuss this in section 4.2.1 when we discuss Stokes’ Theorem.

In summary, we have the following expressions which relate to closed contours or closed surfaces
when the field is conservative or source-free.

Conservative field (2d&3d) Source-free field (2d) Source-free field (3d)


�𝑭𝑭 ∙ 𝑻𝑻𝑑𝑑𝑑𝑑 = 0 �𝑭𝑭 ∙ 𝒏𝒏𝒔𝒔 𝑑𝑑𝑑𝑑 = 0 �𝑭𝑭 ∙ 𝒏𝒏𝑺𝑺𝑑𝑑𝑑𝑑 = 0
𝐶𝐶 𝐶𝐶 𝑆𝑆

The formal proof for the Fundamental Theorem of Line Integrals (for the Conservative and Source-
free field cases) are given in section 4.3 and 4.4 respectively of the expanded set of notes, for
which you are not responsible. The formal proofs are not given for Stokes’ theorem and the
Divergence theorem since these proofs are outside the scope of this course. Instead, we will work
out some examples that will help convince you that these theorems are correct.

81
Table 11 Summary of Theorems, Conventions, and Illustrations

Theorem Convention or Note Illustration


The direction of the unit vector 𝒏𝒏𝑺𝑺 is
�𝑭𝑭 ∙ 𝒏𝒏𝑺𝑺 𝑑𝑑𝑑𝑑 = � ∇ ∙ 𝑭𝑭𝑑𝑑𝑑𝑑
𝑆𝑆 𝑉𝑉 associated with the differential surface area. It
Divergence Theorem: 3d points outwards from the surface 𝑆𝑆 (blue grid
lines) that encloses the volume 𝑉𝑉 (blue).

Apply the right-hand rule: The fingers of the


� 𝑭𝑭 ∙ 𝑻𝑻𝑑𝑑𝑑𝑑 = �∇ × 𝑭𝑭 ∙ 𝒏𝒏𝑺𝑺 𝑑𝑑𝑑𝑑
𝐶𝐶 𝑆𝑆 right-hand point in the direction of the unit
Stokes’ Theorem: 3d tangent vector 𝑻𝑻 and the thumb points in the
direction of the unit vector 𝒏𝒏𝑺𝑺 associated with
the differential surface area. The surface 𝑆𝑆
(blue) is enclosed by the closed contour 𝐶𝐶
(red).

𝜕𝜕𝜕𝜕 𝜕𝜕𝜕𝜕 The fingers of the right-hand point in the


� 𝑭𝑭𝟐𝟐𝟐𝟐 ∙ 𝑻𝑻𝟐𝟐𝟐𝟐 𝑑𝑑𝑑𝑑 = � � − � 𝑑𝑑𝑑𝑑
𝐶𝐶 𝐴𝐴 𝜕𝜕𝜕𝜕 𝜕𝜕𝜕𝜕 direction of the unit tangent vector 𝑻𝑻𝟐𝟐𝟐𝟐, which
𝑭𝑭𝟐𝟐𝟐𝟐 = 〈𝑓𝑓(𝑥𝑥, 𝑦𝑦), 𝑔𝑔(𝑥𝑥, 𝑦𝑦)〉 is oriented in a counterclockwise direction.
𝑻𝑻𝟐𝟐𝟐𝟐 → 𝑻𝑻 The planar surface 𝐴𝐴 (blue) is enclosed by the
Green’s Circulation Theorem: Stokes’ closed contour 𝐶𝐶 (red).
Theorem in 2d

𝜕𝜕𝜕𝜕 𝜕𝜕𝜕𝜕 The fingers of the right-hand point in the


� 𝑭𝑭𝟐𝟐𝟐𝟐 ∙ 𝒏𝒏𝟐𝟐𝟐𝟐 𝑑𝑑𝑑𝑑 = � � + � 𝑑𝑑𝑑𝑑
𝐶𝐶 𝐴𝐴 𝜕𝜕𝜕𝜕 𝜕𝜕𝜕𝜕 direction of the unit tangent vector 𝑻𝑻𝟐𝟐𝟐𝟐,
𝑭𝑭𝟐𝟐𝟐𝟐 = 〈𝑓𝑓(𝑥𝑥, 𝑦𝑦), 𝑔𝑔(𝑥𝑥, 𝑦𝑦)〉 oriented in a counterclockwise direction. The
𝒏𝒏𝟐𝟐𝟐𝟐 → 𝒏𝒏𝒔𝒔 and 𝑻𝑻𝟐𝟐𝟐𝟐 → 𝑻𝑻 unit vector 𝒏𝒏𝟐𝟐𝟐𝟐 is perpendicular to the contour
Green’s Flux Theorem: Divergence 𝐶𝐶 and is pointing towards the right. The planar
Theorem in 2d surface 𝐴𝐴 (blue) is enclosed by the closed
contour 𝐶𝐶(red).

𝐵𝐵
∫𝐶𝐶 𝑭𝑭𝒇𝒇𝒇𝒇𝒇𝒇𝒇𝒇 ∙ 𝑻𝑻𝑑𝑑𝑑𝑑 = ∫𝐴𝐴 𝑭𝑭𝒇𝒇𝒇𝒇𝒇𝒇𝒇𝒇 ∙ 𝑻𝑻𝑑𝑑𝑑𝑑 = 𝜙𝜙(𝐵𝐵) − Note: The result does not depend on the path
𝜙𝜙(𝐴𝐴) taken from point 𝐴𝐴 to point 𝐵𝐵.
𝜙𝜙(𝑥𝑥, 𝑦𝑦)or 𝜙𝜙(𝑥𝑥, 𝑦𝑦, 𝑧𝑧) → potential function &
𝑭𝑭𝒇𝒇𝒇𝒇𝒇𝒇𝒇𝒇 = ∇𝜙𝜙
Fundamental Theorem of Line Integrals
for a Conservative 2d or 3d Field
𝐵𝐵 Note: The result does not depend on the path
� 𝑭𝑭𝒄𝒄𝒄𝒄𝒄𝒄𝒄𝒄 ∙ 𝒏𝒏𝒔𝒔 𝑑𝑑𝑑𝑑 = � 𝑭𝑭𝒄𝒄𝒄𝒄𝒄𝒄𝒄𝒄 ∙ 𝒏𝒏𝒔𝒔 𝑑𝑑𝑑𝑑
𝐶𝐶 𝐴𝐴
taken from point 𝐴𝐴 to point 𝐵𝐵.
= 𝜑𝜑(𝐵𝐵) − 𝜑𝜑(𝐴𝐴)
𝜑𝜑(𝑥𝑥, 𝑦𝑦) → stream function
𝜕𝜕𝜕𝜕(𝑥𝑥,𝑦𝑦) 𝜕𝜕𝜕𝜕(𝑥𝑥,𝑦𝑦)
& 𝑭𝑭𝒄𝒄𝒄𝒄𝒄𝒄𝒄𝒄 = 〈 ,− 〉
𝜕𝜕𝜕𝜕 𝜕𝜕𝜕𝜕
Fundamental Theorem of Line Integrals
for a Source free 2d Field

82
4.1 Divergence Theorem
In this section we describe the Divergence Theorem formulation for the 3d and 2d case.

Here we assume that:

• The surface enclosing the volume, in 3d, or the line enclosing a planar area, in 2d, is
continuous.
• The components of the vector valued function 𝑭𝑭 at each point on the surface, in 3d, or the
at each point on the contour, in 2d, are continuous,
• The closed surface encloses the entire region, in 3d, and the closed contour encloses the
entire planar region, in 2d,
• The partial derivatives of the vector valued function 𝑭𝑭 at every point in the region enclosed
by the surface in 3d and enclosed by the contour in 2d are defined.

In section 4.7 we show how we can address the problem of a region which is not fully enclosed,
such as a region with voids, or an enclosed region containing one or more points where one or
more of the partial derivatives of the vector valued function 𝑭𝑭 is not defined. We define these cases
as problematic cases.

4.1.1 3d formulation

The Divergence Theorem in 3d is given by ∯𝑆𝑆 𝑭𝑭 ∙ 𝑑𝑑𝑺𝑺 = ∭𝑉𝑉 ∇ ∙ 𝑭𝑭𝑑𝑑𝑑𝑑. Fig. 12 shows a sampling of
geometries which we will focus on in this course.

(a) 1 continuous surface (b) 3 distinct surfaces (c) 6 distinct surfaces

(a) ∯𝑆𝑆 𝑭𝑭 ∙ 𝑑𝑑𝑺𝑺


(b) ∯𝑆𝑆 𝑭𝑭 ∙ 𝑑𝑑𝑺𝑺 = ∬𝑆𝑆 𝑭𝑭 ∙ 𝑑𝑑𝑺𝑺𝟏𝟏 + ∬𝑆𝑆 𝑭𝑭 ∙ 𝑑𝑑𝑺𝑺𝟐𝟐 + ∬𝑆𝑆 𝑭𝑭 ∙ 𝑑𝑑𝑺𝑺𝟑𝟑 where 𝑆𝑆 = 𝑆𝑆1 ∪ 𝑆𝑆2 ∪ 𝑆𝑆3
1 2 3

(c) ∯𝑆𝑆 𝑭𝑭 ∙ 𝑑𝑑𝑺𝑺 = ∬𝑆𝑆 𝑭𝑭 ∙ 𝑑𝑑𝑺𝑺𝟏𝟏 + ∬𝑆𝑆 𝑭𝑭 ∙ 𝑑𝑑𝑺𝑺𝟐𝟐 + ∬𝑆𝑆 𝑭𝑭 ∙ 𝑑𝑑𝑺𝑺𝟑𝟑 ∬𝑆𝑆 𝑭𝑭 ∙ 𝑑𝑑𝑺𝑺𝟒𝟒 + ∬𝑆𝑆 𝑭𝑭 ∙ 𝑑𝑑𝑺𝑺𝟓𝟓 + ∬𝑆𝑆 𝑭𝑭 ∙ 𝑑𝑑𝑺𝑺𝟔𝟔
1 2 3 4 5 6
where 𝑆𝑆 = 𝑆𝑆1 ∪ 𝑆𝑆2 ∪ 𝑆𝑆3 ∪ 𝑆𝑆4 ∪ 𝑆𝑆5 ∪ 𝑆𝑆6
Fig. 12 Interpretation of Divergence Theorem in 3d: ∯𝑺𝑺 𝑭𝑭 ∙ 𝒅𝒅𝒅𝒅 illustrated for three
different examples
The convention as already pointed out is that the surface area vector always points outwards from
the surface 𝑆𝑆 that encloses the volume 𝑉𝑉.

83
4.1.2 Exercises Illustrating the Use of the Divergence Theorem

This set of exercises gives you the ability to understand (a) how the divergence theorem can be
utilized and (b) how to apply the correct conventions. Applications of the Divergence Theorem
will be explored in greater detail in Assignment 5. Our main objective here is for you as a student
to understand the Divergence theorem as a tool.

1. Let 𝑭𝑭(𝑥𝑥, 𝑦𝑦, 𝑧𝑧) = 𝑥𝑥 4 𝒊𝒊 − 𝑥𝑥 3 𝑧𝑧 2 𝒋𝒋 + 4𝑥𝑥𝑦𝑦 2 𝑧𝑧𝒌𝒌. Find the outward flux of F across the closed
surface 𝑆𝑆 which is the surface of the solid bounded by the cylinder 𝑥𝑥 2 + 𝑦𝑦 2 = 1 and the
planes 𝑧𝑧 = 𝑥𝑥 + 2 and 𝑧𝑧 = 0 (refer to Archived Lecture Video 23A)

Note: you have two ways of solving this problem using the Divergence Theorem: solving
a closed surface integral problem or solving an enclosed volume integral. Usually, one of
these is easier. Here we will solve the problem both ways and then you can judge which of
the two was the easiest way.

Solution

• Yellow surface: the plane 𝑧𝑧 = 𝑥𝑥 + 2.


• Red surface: the plane 𝑧𝑧 = 0.
• Blue surface: the cylinder 𝑥𝑥 2 + 𝑦𝑦 2 = 1.
• 𝒏𝒏𝒛𝒛=𝒙𝒙+𝟐𝟐𝑑𝑑𝑑𝑑 is the orientation vector for the
slanted planar surface,
• 𝒏𝒏𝒛𝒛=𝟎𝟎 𝑑𝑑𝑑𝑑 is the orientation vector for the
planar surface on the 𝑥𝑥𝑥𝑥 − plane.
• 𝒏𝒏𝒄𝒄𝒄𝒄𝒄𝒄 𝑑𝑑𝑑𝑑 is the orientation vector for the
outside of the cylinder.
• Apply convention: the orientation vector
for all three surfaces point outwards from
the enclosing surface.
• Explore this graph interactively by
clicking on the following link.

https://www.math3d.org/rYt4CaiO

You can easily show that planes 𝑧𝑧 = 𝑥𝑥 + 2 and 𝑧𝑧 = 0 intersect outside the cylinder defined by
𝑥𝑥 2 + 𝑦𝑦 2 = 1: i.e., 𝑧𝑧 = 𝑥𝑥 + 2 is the top surface and 𝑧𝑧 = 0 is the bottom surface of the solid.

Let 𝑉𝑉 be the region bounded by 𝑆𝑆. Then, according to the Divergence theorem, the total flux is:

�𝑭𝑭 ∙ 𝒏𝒏𝑑𝑑𝑑𝑑 = � ∇ ∙ 𝑭𝑭𝑑𝑑𝑑𝑑


𝑆𝑆 𝑉𝑉

84
Computing flux using the left-hand side

There are three discrete surfaces making up the closed surface thus

�𝑭𝑭 ∙ 𝒏𝒏𝑑𝑑𝑑𝑑 = � 𝑭𝑭 ∙ 𝒏𝒏𝒄𝒄𝒄𝒄𝒄𝒄𝑑𝑑𝑑𝑑 + � 𝑭𝑭 ∙ 𝒏𝒏𝒛𝒛=𝟎𝟎 𝑑𝑑𝑑𝑑 + � 𝑭𝑭 ∙ 𝒏𝒏𝒛𝒛=𝒙𝒙+𝟐𝟐𝑑𝑑𝑑𝑑


𝑆𝑆 𝑐𝑐𝑐𝑐𝑐𝑐 𝑧𝑧=0 𝑧𝑧=𝑥𝑥+2

The normal vectors for each surface point outwards with respect to the closed surfaces.

For the top and bottom surfaces, we can use the explicit representation of a surface.

1 √1−𝑥𝑥 2
� 𝑭𝑭 ∙ 𝒏𝒏𝒛𝒛=𝟎𝟎 𝑑𝑑𝑑𝑑 = � � 〈𝑥𝑥 4 , 0, 0〉 ∙ 〈0,0, −1〉𝑑𝑑𝑑𝑑𝑑𝑑𝑑𝑑
𝑧𝑧=0 −1 −√1−𝑥𝑥 2

1 √1−𝑥𝑥 2
� 𝑭𝑭 ∙ 𝒏𝒏𝒛𝒛=𝒙𝒙+𝟐𝟐𝑑𝑑𝑑𝑑 = � � 〈𝑥𝑥 4 , −𝑥𝑥 3 (𝑥𝑥 + 2)2 , 4𝑥𝑥𝑦𝑦 2 (𝑥𝑥 + 2)〉 ∙ 〈−1,0,1〉𝑑𝑑𝑑𝑑𝑑𝑑𝑑𝑑
𝑧𝑧=𝑥𝑥+2 −1 −√1−𝑥𝑥 2

Surface parameterization is used for the outer surface of the cylinder. Let the position vector be:

𝑟𝑟⃑ = 〈𝑐𝑐𝑐𝑐𝑐𝑐𝑐𝑐, 𝑠𝑠𝑠𝑠𝑠𝑠𝑠𝑠, 𝑣𝑣(𝑐𝑐𝑐𝑐𝑐𝑐𝑐𝑐 + 2)〉 0 ≤ 𝜃𝜃 < 2𝜋𝜋 0 ≤ 𝑣𝑣 ≤ 1

Then

𝒏𝒏𝒄𝒄𝒄𝒄𝒄𝒄 𝑑𝑑𝑑𝑑 = 𝑡𝑡���⃑ ���⃑𝑧𝑧 𝑑𝑑𝑑𝑑𝑑𝑑𝑑𝑑 = 〈𝑐𝑐𝑐𝑐𝑐𝑐𝑐𝑐(𝑐𝑐𝑐𝑐𝑐𝑐𝑐𝑐 + 2), 𝑠𝑠𝑠𝑠𝑠𝑠𝑠𝑠(𝑐𝑐𝑐𝑐𝑐𝑐𝑐𝑐 + 2), 0 〉𝑑𝑑𝑑𝑑𝑑𝑑𝑑𝑑
𝜃𝜃 × 𝑡𝑡

and 𝑭𝑭 = 〈𝑐𝑐𝑐𝑐𝑐𝑐 4𝜃𝜃, −𝑣𝑣 2𝑐𝑐𝑐𝑐𝑐𝑐 3𝜃𝜃(𝑐𝑐𝑐𝑐𝑐𝑐𝑐𝑐 + 2)2 , 4𝑣𝑣𝑣𝑣𝑣𝑣𝑣𝑣𝑣𝑣𝑠𝑠𝑠𝑠𝑠𝑠2 𝜃𝜃 (𝑐𝑐𝑐𝑐𝑐𝑐𝑐𝑐 + 2)〉.

The final expression is

� 𝑭𝑭 ∙ 𝒏𝒏𝒄𝒄𝒄𝒄𝒄𝒄𝑑𝑑𝑑𝑑 =
𝑐𝑐𝑐𝑐𝑐𝑐

2𝜋𝜋 1
� � 〈𝑐𝑐𝑐𝑐𝑐𝑐 4 𝜃𝜃, −𝑣𝑣 2𝑐𝑐𝑐𝑐𝑐𝑐 3 𝜃𝜃(𝑐𝑐𝑐𝑐𝑐𝑐𝑐𝑐 + 2)2 , 4𝑣𝑣𝑣𝑣𝑣𝑣𝑣𝑣𝑣𝑣𝑠𝑠𝑠𝑠𝑠𝑠2 𝜃𝜃 (𝑐𝑐𝑐𝑐𝑐𝑐𝑐𝑐 + 2)〉
0 0
∙ 〈𝑐𝑐𝑐𝑐𝑐𝑐𝑐𝑐(𝑐𝑐𝑐𝑐𝑐𝑐𝑐𝑐 + 2), 𝑠𝑠𝑠𝑠𝑠𝑠𝑠𝑠 (𝑐𝑐𝑐𝑐𝑐𝑐𝑐𝑐 + 2), 0 〉𝑑𝑑𝑑𝑑𝑑𝑑𝑑𝑑

Although we have formulated the problem, the last integral is complicated to solve, but you can
do it as an exercise. We will stop at this point and instead solve the problem using the right-hand
side of the Divergence Theorem.

Computing flux using the right-hand side

1 √1−𝑥𝑥 2 𝑥𝑥+2
∭𝑉𝑉 ∇ ∙ 𝑭𝑭𝑑𝑑𝑑𝑑 = ∫−1 ∫−√1−𝑥𝑥2 ∫0 4𝑥𝑥 (𝑥𝑥 2 + 𝑦𝑦 2 )𝑑𝑑𝑑𝑑𝑑𝑑𝑑𝑑𝑑𝑑𝑑𝑑

Convert to cylindrical coordinates:

85
2π 1 𝑟𝑟𝑟𝑟𝑟𝑟𝑟𝑟𝑟𝑟+2 2𝜋𝜋 1
3
2
� � � (4𝑟𝑟 𝑐𝑐𝑐𝑐𝑐𝑐𝑐𝑐 )𝑑𝑑𝑑𝑑𝑑𝑑𝑑𝑑𝑑𝑑𝑑𝑑𝑑𝑑 = � � (4𝑟𝑟 5 𝑐𝑐𝑐𝑐𝑐𝑐 2𝜃𝜃 + 8𝑟𝑟 4 𝑐𝑐𝑐𝑐𝑐𝑐𝑐𝑐)𝑑𝑑𝑑𝑑𝑑𝑑𝑑𝑑 = 𝜋𝜋
0 0 0 0 0 3

A side note

The original problem could have been formulated differently. The objective could have been to
compute the flux from the cylindrical surface. In this case, we use the calculated results and
rearrange the Divergence Theorem to get an expression for the flux through the cylindrical surface:

� 𝑭𝑭 ∙ 𝒏𝒏𝒄𝒄𝒄𝒄𝒄𝒄 𝑑𝑑𝑑𝑑 = � ∇ ∙ 𝑭𝑭𝑑𝑑𝑉𝑉 − � 𝑭𝑭 ∙ 𝒏𝒏𝒛𝒛=𝟎𝟎 𝑑𝑑𝑑𝑑 − � 𝑭𝑭 ∙ 𝒏𝒏𝒛𝒛=𝒙𝒙+𝟐𝟐𝑑𝑑𝑑𝑑


𝑐𝑐𝑐𝑐𝑐𝑐 𝑉𝑉 𝑧𝑧=0 𝑧𝑧=𝑥𝑥+2

2. Let 𝑭𝑭(𝑥𝑥, 𝑦𝑦, 𝑧𝑧) = 𝑧𝑧𝑡𝑡𝑡𝑡𝑡𝑡−1 (𝑦𝑦 2 )𝑥𝑥� − 𝑧𝑧 3 ln (𝑥𝑥 2 + 1)𝑦𝑦� + 𝑧𝑧𝑧𝑧̂ . Find the flux of F across the part of
the paraboloid 𝑥𝑥 2 + 𝑦𝑦 2 + 𝑧𝑧 = 2 that lies above the plane 𝑧𝑧 = 1 and is oriented upwards.

Solution

• Greyed out area is the projection of the region


bounded by the intersection line between surfaces 𝑆𝑆1
(shaded in blue) and 𝑆𝑆2 , (shaded in red).
• 𝒏𝒏𝑺𝑺𝑺𝑺 𝑑𝑑𝑑𝑑 is the orientation vector for the paraboloid
surface.
• 𝒏𝒏𝑺𝑺𝑺𝑺 𝑑𝑑𝑑𝑑 is the orientation vector for the circular disk.
• Convention applied: orientation vector points
outwards from the enclosing surface.
• Explore this graph interactively by clicking on the
following link.

https://www.math3d.org/1NL9fIgm

We look at the closed surface bounded by 𝑆𝑆 formed by two separate surfaces 𝑆𝑆1 and 𝑆𝑆2 . 𝑆𝑆1 is the
part of the paraboloid 𝑥𝑥 2 + 𝑦𝑦 2 + 𝑧𝑧 = 2 that lies above the plane 𝑧𝑧 = 1 and 𝑆𝑆2 is the disk 𝑥𝑥 2 +
𝑦𝑦 2 = 1 on the plane 𝑧𝑧 = 1 which is oriented downward. Computing the flux through surface 𝑆𝑆1
may be computationally more intensive. Alternatively, we can use the Divergence Theorem to
simplify the calculation process.

Let 𝑉𝑉 be the region bounded by S. Then according to the Divergence theorem

�𝑭𝑭 ∙ 𝒏𝒏𝑑𝑑𝑑𝑑 = � 𝑭𝑭 ∙ 𝒏𝒏𝑺𝑺𝑺𝑺 𝑑𝑑𝑑𝑑 + � 𝑭𝑭 ∙ 𝒏𝒏𝑺𝑺𝑺𝑺 𝑑𝑑𝑑𝑑 = � ∇ ∙ 𝑭𝑭𝑑𝑑𝑑𝑑


𝑆𝑆 𝑆𝑆1 𝑆𝑆2 𝑉𝑉

86
Since the disk 𝑆𝑆2 is oriented downward, then the vector 𝒏𝒏𝑺𝑺𝑺𝑺 𝑑𝑑𝑑𝑑 is given by

𝒏𝒏𝑺𝑺𝑺𝑺 𝑑𝑑𝑑𝑑 = −𝑧𝑧̂ 𝑑𝑑𝑑𝑑𝑑𝑑𝑑𝑑 and 𝑭𝑭 on the plane 𝑧𝑧 = 1 is

𝑭𝑭 = 𝑡𝑡𝑡𝑡𝑡𝑡−1 (𝑦𝑦 2 )𝑥𝑥� − ln (𝑥𝑥 2 + 1)𝑦𝑦� + 𝑧𝑧̂

Therefore

𝑭𝑭 ∙ −𝑧𝑧̂ 𝑑𝑑𝑑𝑑𝑑𝑑𝑑𝑑 = −1𝑑𝑑𝑑𝑑𝑑𝑑𝑑𝑑

and

1 √1−𝑥𝑥 2
� 𝑭𝑭 ∙ 𝒏𝒏𝑺𝑺𝑺𝑺 𝑑𝑑𝑑𝑑 = � � −𝑑𝑑𝑑𝑑𝑑𝑑𝑑𝑑
𝑆𝑆2 −1 −√1−𝑥𝑥 2

It is easier to solve this integral in polar coordinates in which case we obtain the following result

1 √1−𝑥𝑥 2 2𝜋𝜋 1
� � −𝑑𝑑𝑑𝑑𝑑𝑑𝑑𝑑 = − � � 𝑟𝑟𝑟𝑟𝑟𝑟𝑟𝑟𝑟𝑟 = −𝜋𝜋
−1 −√1−𝑥𝑥 2 0 0

If we compute the right-hand side of the Divergence Theorem, we obtain.


1 2𝜋𝜋 2−𝑟𝑟 2
𝜋𝜋
� ∇ ∙ 𝑭𝑭𝑑𝑑𝑑𝑑 = � � � 𝑑𝑑𝑑𝑑𝑑𝑑𝑑𝑑𝑑𝑑𝑑𝑑𝑑𝑑 =
𝐸𝐸 0 0 1 2

Thus, the flux of F across 𝑆𝑆1 is

� 𝑭𝑭 ∙ 𝒏𝒏𝑺𝑺𝑺𝑺 𝑑𝑑𝑑𝑑 = � ∇ ∙ 𝑭𝑭𝑑𝑑𝑑𝑑 − � 𝑭𝑭 ∙ 𝒏𝒏𝑺𝑺𝑺𝑺 𝑑𝑑𝑑𝑑


𝑆𝑆1 𝑉𝑉 𝑆𝑆2

Therefore,

𝜋𝜋 3𝜋𝜋
� ∇ ∙ 𝑭𝑭𝑑𝑑𝑑𝑑 − � 𝑭𝑭 ∙ 𝒏𝒏𝑑𝑑𝑆𝑆2 = − (−𝜋𝜋) =
𝑉𝑉 𝑆𝑆2 2 2

At this point you can work on the following problems which have been extracted from section 17.8
of the textbook:

4.1.3 Problems from Section 17.8: 1, 3, 9, 11, 17, 21, 29, 31, 35, 39 (a, c, d, e), 47

1. (question 1) Explain the meaning of the surface integral in the Divergence Theorem.
2. (question 3) Explain the meaning of the Divergence Theorem
3. (question 9) Verifying the Divergence Theorem Evaluate both integrals of the Divergence
Theorem for the following vector field and region. Check for agreement: 𝑭𝑭 =
〈2𝑥𝑥, 3𝑦𝑦, 4𝑧𝑧〉: 𝐷𝐷 = {(𝑥𝑥, 𝑦𝑦, 𝑧𝑧): 𝑥𝑥 2 + 𝑦𝑦 2 + 𝑧𝑧 2 ≤ 4}

87
4. (question 11) Verifying the Divergence Theorem Evaluate both integrals of the Divergence
Theorem for the following vector field and region. Check for agreement: 𝑭𝑭 =
𝑥𝑥 2 𝑦𝑦 2 𝑧𝑧 2
〈𝑧𝑧 − 𝑦𝑦, 𝑥𝑥, −𝑥𝑥 〉: 𝐷𝐷 = �(𝑥𝑥, 𝑦𝑦, 𝑧𝑧): + + 12 ≤ 1�
4 8
5. (question 17) Computing flux Use the Divergence Theorem to compute the net outward flux
of the following field across the given surface 𝑆𝑆: 𝑭𝑭 = 〈𝑥𝑥, −2𝑦𝑦, 3𝑧𝑧〉: 𝑆𝑆 is the surface
{(𝑥𝑥, 𝑦𝑦, 𝑧𝑧): 𝑥𝑥 2 + 𝑦𝑦 2 + 𝑧𝑧 2 = 6}
6. (question 21) Computing flux Use the Divergence Theorem to compute the net outward flux
of the following field across the given surface 𝑆𝑆: 𝑭𝑭 = 〈𝑦𝑦 − 2𝑥𝑥, 𝑥𝑥 3 − 𝑦𝑦, 𝑦𝑦 2 − 𝑧𝑧〉: 𝑆𝑆 is the
surface {(𝑥𝑥, 𝑦𝑦, 𝑧𝑧): 𝑥𝑥 2 + 𝑦𝑦 2 + 𝑧𝑧 2 = 4}
7. (question 29) Divergence Theorem for more general regions Use the Divergence Theorem
to compute the net outward flux of the following vector field across the boundary of the given
region 𝐷𝐷: 𝑭𝑭 = 〈𝑥𝑥 2 , −𝑦𝑦 2 , 𝑧𝑧 2 〉: 𝐷𝐷 is the region in the first octant between the planes 𝑧𝑧 = 4 −
𝑥𝑥 − 𝑦𝑦 and 𝑧𝑧 = 2 − 𝑥𝑥 − 𝑦𝑦.
8. (question 31) Explain why or why not Determine whether the following statements are true
and give an explanation or counterexample.
a. If ∇ ∙ 𝑭𝑭 = 0 at all points of a region 𝐷𝐷, then 𝑭𝑭 ∙ 𝒏𝒏 = 0 at all points of the boundary of 𝐷𝐷.
b. If ∯𝑆𝑆 𝑭𝑭 ∙ 𝒏𝒏𝑑𝑑𝑑𝑑 = 0 on all closed surfaces in ℝ3 , then 𝑭𝑭 is constant.
c. If |𝑭𝑭| ≤ 1, then �∭𝐷𝐷 ∇ ∙ 𝑭𝑭𝑑𝑑𝑑𝑑 � is less than the area of the surface of 𝐷𝐷.
9. (question 35) Flux integrals Compute the outward flux of the following vector field across
the given surface 𝑆𝑆. You should decide which integral of the Divergence Theorem to use: 𝑭𝑭 =
〈𝑥𝑥𝑥𝑥𝑥𝑥𝑥𝑥𝑥𝑥, −𝑐𝑐𝑐𝑐𝑐𝑐𝑐𝑐, 𝑧𝑧𝑧𝑧𝑧𝑧𝑧𝑧𝑧𝑧〉: 𝑆𝑆 is the boundary of the region bounded by the planes 𝑥𝑥 = 1, 𝑦𝑦 = 0,
𝜋𝜋
𝑦𝑦 = , 𝑧𝑧 = 0 and 𝑧𝑧 = 𝑥𝑥.
2
10. (question 39 a, c, d, e) Gauss’ Law (Divergence Theorem) for electric fields The electric
𝑄𝑄 𝒓𝒓
field due to a point charge 𝑄𝑄 is 𝑬𝑬 = |𝒓𝒓|3
, where 𝒓𝒓 = 〈𝑥𝑥, 𝑦𝑦, 𝑧𝑧〉 and 𝜀𝜀𝑜𝑜 is a constant.
4𝜋𝜋𝜀𝜀𝑜𝑜
a. Show that the flux of the field across a sphere of radius 𝑎𝑎 centered at the origin is
𝑄𝑄
∯𝑆𝑆 𝑬𝑬 ∙ 𝒏𝒏𝑑𝑑𝑑𝑑 = . 𝜀𝜀𝑜𝑜
b. Suppose there is a distribution of charge within a region 𝐷𝐷. Let 𝑞𝑞(𝑥𝑥, 𝑦𝑦, 𝑧𝑧) be the charge
density (charge per unit volume). Interpret the statement that ∯𝑆𝑆 𝑬𝑬 ∙ 𝒏𝒏𝑑𝑑𝑑𝑑 =
1
𝜀𝜀𝑜𝑜
∭𝐷𝐷 𝑞𝑞(𝑥𝑥, 𝑦𝑦, 𝑧𝑧)𝑑𝑑𝑑𝑑.
c. Assuming 𝑬𝑬 satisfies the conditions of the Divergence Theorem on 𝐷𝐷, conclude from part
𝑞𝑞
(b) that ∇ ∙ 𝑬𝑬 = .
𝜀𝜀𝑜𝑜
d. The electric field can be expressed in the following form: 𝑬𝑬 = −∇𝜙𝜙 where 𝜙𝜙 is a scalar
𝑞𝑞
function. From part (c) conclude that ∇2 𝜙𝜙 = − .
𝜀𝜀𝑜𝑜
11. (question 47) A beautiful flux integral Consider the potential function 𝜙𝜙(𝑥𝑥, 𝑦𝑦, 𝑧𝑧) = 𝐺𝐺(𝜌𝜌), G
is any twice differentiable function and 𝜌𝜌 = �𝑥𝑥 2 + 𝑦𝑦 2 + 𝑧𝑧 2 : therefore, 𝐺𝐺 depends only on the
distance from the origin.
𝒓𝒓
a. Show that the gradient vector field associated with 𝜙𝜙 is given by: 𝑭𝑭 = ∇𝜙𝜙 = 𝐺𝐺 ′(𝜌𝜌) ,
𝜌𝜌
where 𝒓𝒓 = 〈𝑥𝑥, 𝑦𝑦, 𝑧𝑧〉 and 𝜌𝜌 = |𝒓𝒓|.

88
b. Let 𝑆𝑆 be the sphere of radius 𝑎𝑎 centered at the origin and let 𝐷𝐷 be the region enclosed by
𝑆𝑆. Show that the flux of 𝑭𝑭 across 𝑆𝑆 is ∯𝑆𝑆 𝑭𝑭 ∙ 𝒏𝒏𝑑𝑑𝑑𝑑 = 4𝜋𝜋𝑎𝑎2 𝐺𝐺 ′(𝑎𝑎).
2𝐺𝐺 ′(𝜌𝜌)
c. Show that ∇ ∙ 𝑭𝑭 = ∇ ∙ ∇𝜙𝜙 = 𝜌𝜌
+ 𝐺𝐺 " (𝜌𝜌).
d. Use part (c) to show that the flux across 𝑆𝑆 (as given in part(b)) is also obtained by the
volume integral ∭𝐷𝐷 ∇ ∙ 𝑭𝑭𝑑𝑑𝑑𝑑 . (Hint: use spherical coordinates and integrate by parts).

12. Suppose that 𝐹𝐹 = 〈𝐹𝐹𝑥𝑥 , 𝐹𝐹𝑦𝑦 , 𝐹𝐹𝑧𝑧 〉 (subscripts here indicating vector components, not partial
derivatives) is a continuously differentiable vector field. Let 𝐶𝐶ℎ be the solid cylinder:
ℎ ℎ
𝐶𝐶ℎ = �(𝑥𝑥, 𝑦𝑦, 𝑧𝑧) | 𝑥𝑥 2 + 𝑦𝑦 2 ≤ 1, − ≤ 𝑧𝑧 ≤ � and let 𝐷𝐷 be the 2-dimensional disc of radius
2 2
1 centred at the origin in the 𝑥𝑥𝑥𝑥 − plane.

𝐷𝐷 = {(𝑥𝑥, 𝑦𝑦, 𝑧𝑧) | 𝑥𝑥 2 + 𝑦𝑦 2 ≤ 1, 𝑧𝑧 = 0}

Show from first principles (i.e., without using the divergence theorem) that

1 𝜕𝜕𝐹𝐹𝑧𝑧
lim � 𝑭𝑭 ∙ 𝒏𝒏𝑑𝑑𝑑𝑑 = � 𝑑𝑑𝑑𝑑 + � 𝑭𝑭 ∙ 〈𝑥𝑥, 𝑦𝑦, 0〉 𝑑𝑑𝑑𝑑
ℎ→0 ℎ 𝜕𝜕𝜕𝜕 𝐷𝐷 𝜕𝜕𝜕𝜕 𝜕𝜕𝜕𝜕

You can find the solutions to the problems at the following link: Solutions to Reading Assignment
4 Problems.

4.1.4 2d formulation

The Divergence Theorem in 2d is given by ∬𝐴𝐴 ∇ ∙ 𝑭𝑭𝟐𝟐𝟐𝟐 𝑑𝑑𝑑𝑑 = ∮𝐶𝐶 𝑭𝑭𝟐𝟐𝟐𝟐 ∙ 𝒏𝒏𝟐𝟐𝟐𝟐 𝑑𝑑𝑑𝑑 and is referred to as
Green’s Flux theorem. Chapter 17.4 of the book devotes a special section to Green’s theorem but
you do not need to look at this section unless you want a different perspective. We will now prove
Green’s Flux theorem from the Divergence theorem.

Proof

Let 𝑭𝑭 = 〈𝑓𝑓 (𝑥𝑥, 𝑦𝑦), 𝑔𝑔(𝑥𝑥, 𝑦𝑦), 0〉 . The divergence theorem, when applied to the example in Figure 12,
becomes:

∆𝑍𝑍
2
� � ∇ ∙ 𝑭𝑭𝑑𝑑𝑑𝑑𝑑𝑑𝑑𝑑𝑑𝑑𝑑𝑑 = �𝑭𝑭 ∙ 𝑑𝑑𝑺𝑺 = � 𝑭𝑭 ∙ 𝑑𝑑𝑺𝑺𝒃𝒃𝒃𝒃𝒃𝒃 + � 𝑭𝑭 ∙ 𝑑𝑑𝑺𝑺𝒕𝒕𝒕𝒕𝒕𝒕 + � 𝑭𝑭 ∙ 𝑑𝑑𝑺𝑺𝒕𝒕𝒕𝒕𝒕𝒕
∆𝑍𝑍
− 𝐴𝐴 𝑆𝑆 𝑆𝑆𝑏𝑏𝑏𝑏𝑏𝑏 𝑆𝑆𝑡𝑡𝑡𝑡𝑡𝑡 𝑆𝑆𝑡𝑡𝑡𝑡𝑡𝑡
2

89
Fig. 12 Interpretation of Divergence Theorem in 2d (Greens Flux Theorem)

The height 𝛥𝛥𝛥𝛥 of the volume is made arbitrarily thin and 𝑑𝑑𝑺𝑺𝒃𝒃𝒃𝒃𝒃𝒃 = −𝑑𝑑𝑺𝑺𝒕𝒕𝒕𝒕𝒕𝒕. More importantly, 𝑭𝑭
is perpendicular to 𝑑𝑑𝑺𝑺𝒃𝒃𝒃𝒃𝒃𝒃 and 𝑑𝑑𝑺𝑺𝒕𝒕𝒕𝒕𝒕𝒕 since 𝑭𝑭 is only defined in the 𝑥𝑥𝑥𝑥 plane (no 𝑧𝑧 field component).
Consequently,

� 𝑭𝑭 ∙ 𝑑𝑑𝑺𝑺𝒃𝒃𝒃𝒃𝒃𝒃 = � 𝑭𝑭 ∙ 𝒅𝒅𝑺𝑺𝒕𝒕𝒕𝒕𝒕𝒕 = 0
𝑆𝑆𝑏𝑏𝑏𝑏𝑏𝑏 𝑆𝑆𝑡𝑡𝑡𝑡𝑡𝑡

and consequently
∆𝑍𝑍
2
� � ∇ ∙ 𝑭𝑭𝑑𝑑𝑑𝑑𝑑𝑑𝑑𝑑𝑑𝑑𝑑𝑑 = � 𝑭𝑭 ∙ 𝒅𝒅𝑺𝑺𝒕𝒕𝒕𝒕𝒕𝒕
∆𝑍𝑍
− 𝐴𝐴 𝑆𝑆𝑡𝑡𝑡𝑡𝑡𝑡
2

where 𝒅𝒅𝑺𝑺𝒕𝒕𝒕𝒕𝒕𝒕 represents the surface area vector which is transverse to the 𝑥𝑥𝑥𝑥 plane. The vertical
surface 𝑑𝑑𝑺𝑺𝒕𝒕𝒕𝒕𝒕𝒕 of the volume can be parameterized as follows:

𝑥𝑥 = 𝑥𝑥(𝜃𝜃) 𝑦𝑦 = 𝑦𝑦(𝜃𝜃) 𝑧𝑧 = 𝑧𝑧

0 < 𝜃𝜃 ≤ 2𝜋𝜋

∆𝑍𝑍 ∆𝑍𝑍
− < 𝑧𝑧 ≤
2 2
The steps to computing 𝑑𝑑𝑺𝑺𝒕𝒕𝒕𝒕𝒕𝒕 representing the side surface are as follows: compute 𝒕𝒕𝜃𝜃 , 𝒕𝒕𝑧𝑧 and
𝒕𝒕𝜃𝜃 × 𝒕𝒕𝑧𝑧 .

90
𝚤𝚤̂ 𝚥𝚥̂ 𝑘𝑘�
𝜕𝜕𝜕𝜕 𝜕𝜕𝜕𝜕 𝜕𝜕𝜕𝜕 𝜕𝜕𝜕𝜕 𝜕𝜕𝜕𝜕 𝜕𝜕𝜕𝜕 𝜕𝜕𝜕𝜕 𝜕𝜕𝜕𝜕 𝜕𝜕𝜕𝜕
𝒕𝒕𝜃𝜃 = 〈 , , 〉 𝒕𝒕𝑧𝑧 = 〈 , , 〉 𝒕𝒕𝜃𝜃 × 𝒕𝒕𝑧𝑧 = ��𝜕𝜕𝜕𝜕 𝜕𝜕𝜕𝜕 𝜕𝜕𝜕𝜕�
� 𝒅𝒅𝑺𝑺𝒕𝒕𝒕𝒕𝒕𝒕 = 𝒕𝒕𝜃𝜃 × 𝒕𝒕𝑧𝑧 𝑑𝑑𝑑𝑑𝑑𝑑𝑑𝑑
𝜕𝜕𝜕𝜕 𝜕𝜕𝜕𝜕 𝜕𝜕𝜕𝜕 𝜕𝜕𝜕𝜕 𝜕𝜕𝜕𝜕 𝜕𝜕𝜕𝜕
𝜕𝜕𝜕𝜕 𝜕𝜕𝜕𝜕 𝜕𝜕𝜕𝜕
𝜕𝜕𝜕𝜕 𝜕𝜕𝜕𝜕 𝜕𝜕𝜕𝜕 𝑑𝑑𝑑𝑑𝑑𝑑

𝜕𝜕𝜕𝜕 𝜕𝜕𝜕𝜕 𝜕𝜕𝜕𝜕


where = 𝑥𝑥 ′ (𝜃𝜃 ), = 𝑦𝑦 ′(𝜃𝜃 ) and = 0 and 𝒅𝒅𝑺𝑺𝒕𝒕𝒕𝒕𝒕𝒕 is pointing outwards from the surface.
𝜕𝜕𝜕𝜕 𝜕𝜕𝜕𝜕 𝜕𝜕𝜕𝜕

Making the substitutions we arrive at the following result:

𝑑𝑑𝑺𝑺𝒕𝒕𝒕𝒕𝒕𝒕 = 〈 𝑦𝑦 ′(𝜃𝜃 ), −𝑥𝑥 ′ (𝜃𝜃 ), 0〉 ∙ 𝑑𝑑𝑑𝑑𝑑𝑑𝑑𝑑 = 𝒏𝒏𝒕𝒕𝒕𝒕𝒕𝒕 𝑑𝑑𝑆𝑆𝑡𝑡𝑡𝑡𝑡𝑡

〈 𝑦𝑦 ′ (𝜃𝜃),−𝑥𝑥 ′ (𝜃𝜃),0〉
where 𝒏𝒏𝒕𝒕𝒕𝒕𝒕𝒕 = and 𝑑𝑑𝑆𝑆𝑡𝑡𝑡𝑡𝑡𝑡 = �𝑥𝑥 ′ 2 (𝜃𝜃 ) + 𝑦𝑦 ′ 2 (𝜃𝜃 )𝑑𝑑𝑑𝑑𝑑𝑑𝑑𝑑
�𝑥𝑥 ′ 2 (𝜃𝜃)+𝑦𝑦 ′2 (𝜃𝜃)

Note: the vector tangent to the contour is 〈𝑥𝑥 ′(𝜃𝜃 ), 𝑦𝑦 ′(𝜃𝜃 ), 0〉 and is oriented in a CCW direction.
〈 𝑥𝑥 ′ (𝜃𝜃 ), 𝑦𝑦 ′(𝜃𝜃 ), 0〉 ∙ 〈 𝑦𝑦 ′(𝜃𝜃 ), −𝑥𝑥 ′(𝜃𝜃 ), 0〉 = 0, therefore, 〈 𝑦𝑦 ′(𝜃𝜃 ), −𝑥𝑥 ′ (𝜃𝜃 ), 0〉 is orthogonal to the
contour and points outwards.
The next step is to substitute the expressions into the right-hand side of the Divergence theorem
equation.
∆𝑍𝑍 ∆𝑍𝑍 ∆𝑍𝑍
2𝜋𝜋 2𝜋𝜋
2 2 2
� �∇ ∙ 𝑭𝑭𝑑𝑑𝑑𝑑𝑑𝑑𝑑𝑑𝑑𝑑𝑑𝑑 = � � 𝑭𝑭 ∙ 〈𝑦𝑦 ′(𝜃𝜃 ), −𝑥𝑥 ′(𝜃𝜃 ), 0〉𝑑𝑑𝑑𝑑𝑑𝑑𝑑𝑑 = � � 𝑭𝑭 ∙ 𝒏𝒏𝒕𝒕𝒕𝒕𝒕𝒕 𝑑𝑑𝑑𝑑𝑑𝑑𝑑𝑑
∆𝑍𝑍 ∆𝑍𝑍 ∆𝑍𝑍
− 𝐴𝐴 − 0 − 0
2 2 2

∆𝑍𝑍
2
=� �𝑭𝑭 ∙ 𝒏𝒏𝒕𝒕𝒕𝒕𝒕𝒕 𝑑𝑑𝑑𝑑𝑑𝑑𝑑𝑑
∆𝑍𝑍
− 𝐶𝐶
2

𝑭𝑭 is not a function of 𝑧𝑧 therefore integrating, over 𝑧𝑧, on the left-hand side and right-hand side gives
a multiplying factor of 𝛥𝛥𝛥𝛥 for both integrals. Then dividing through by 𝛥𝛥𝛥𝛥 we arrive at the result

� ∇ ∙ 𝑭𝑭𝑑𝑑𝑑𝑑𝑑𝑑𝑑𝑑 = �∇ ∙ 𝑭𝑭𝟐𝟐𝟐𝟐 𝑑𝑑𝑑𝑑 = �𝑭𝑭 ∙ 〈𝑦𝑦 ′(𝜃𝜃 ), −𝑥𝑥 ′ (𝜃𝜃 ), 0〉 ∙ 𝑑𝑑𝑑𝑑 = �𝑭𝑭𝟐𝟐𝟐𝟐 ∙ 𝒏𝒏𝟐𝟐𝟐𝟐 𝑑𝑑𝑑𝑑
𝐴𝐴 𝐴𝐴 𝐶𝐶 𝐶𝐶

〈𝑦𝑦 ′ (𝜃𝜃),−𝑥𝑥 ′ (𝜃𝜃)〉 2 2


where 𝑭𝑭𝟐𝟐𝟐𝟐 = 〈𝑓𝑓 (𝑥𝑥, 𝑦𝑦), 𝑔𝑔(𝑥𝑥, 𝑦𝑦)〉 , 𝒏𝒏𝟐𝟐𝟐𝟐 = and 𝑑𝑑𝑑𝑑 = ��𝑥𝑥 ′ (𝜃𝜃 )� + �𝑦𝑦 ′(𝜃𝜃 )� 𝑑𝑑𝑑𝑑
2 2
��𝑥𝑥 ′ (𝜃𝜃)� +�𝑦𝑦 ′ (𝜃𝜃)�

The expression ∮𝐶𝐶 𝑭𝑭𝟐𝟐𝟐𝟐 ∙ 𝒏𝒏𝟐𝟐𝟐𝟐𝑑𝑑𝑑𝑑 = ∬𝐴𝐴 ∇ ∙ 𝑭𝑭𝟐𝟐𝟐𝟐 𝑑𝑑𝑑𝑑 is also referred to as Green’s Flux Theorem ∎.

We will discuss Green’s Flux Theorem during the second last week of term.

91
4.1.5 Green’s Flux Theorem

Green’s Flux Theorem is not used very often so the discussion below provides some worked-out
examples.

4.1.6 Example Problems

1. Evaluate ∮𝐶𝐶(4𝑥𝑥 3 + 𝑠𝑠𝑠𝑠𝑠𝑠𝑦𝑦 2 )𝑑𝑑𝑑𝑑 − (4𝑦𝑦 3 + 𝑐𝑐𝑐𝑐𝑐𝑐𝑥𝑥 2 )𝑑𝑑𝑑𝑑, where 𝐶𝐶 is the boundary of the disk
𝑅𝑅 = {(𝑥𝑥, 𝑦𝑦): 𝑥𝑥 2 + 𝑦𝑦 2 ≤ 4} oriented counterclockwise.

Solution

Letting 𝑓𝑓(𝑥𝑥, 𝑦𝑦) = 4𝑥𝑥 3 + 𝑠𝑠𝑠𝑠𝑠𝑠𝑦𝑦 2 and 𝑔𝑔(𝑥𝑥, 𝑦𝑦) = 4𝑦𝑦 3 + 𝑐𝑐𝑐𝑐𝑐𝑐𝑥𝑥 2 , Green’s Flux Theorem takes
the form:

𝜕𝜕𝜕𝜕 𝜕𝜕𝜕𝜕
� (4𝑥𝑥 3 + 𝑠𝑠𝑠𝑠𝑠𝑠𝑦𝑦 2 )𝑑𝑑𝑑𝑑 − (4𝑦𝑦 3 + 𝑐𝑐𝑐𝑐𝑐𝑐𝑥𝑥 2 )𝑑𝑑𝑑𝑑 = � � + � 𝑑𝑑𝑑𝑑 = � (12𝑥𝑥 2 + 12𝑦𝑦 2 )𝑑𝑑𝑑𝑑
𝐶𝐶 𝑅𝑅 𝜕𝜕𝜕𝜕 𝜕𝜕𝜕𝜕 𝑅𝑅
2𝜋𝜋 2 2𝜋𝜋 4 2 2𝜋𝜋
2
𝑟𝑟
=� � 𝑟𝑟 𝑟𝑟𝑟𝑟𝑟𝑟𝑟𝑟𝑟𝑟 = 12 � � 𝑑𝑑𝑑𝑑 = 48 � 𝑑𝑑𝑑𝑑 = 96𝜋𝜋
0 0 0 4 0 0

2. Consider the example from section 1.8 which included a pure source field 𝑭𝑭 = 〈𝑥𝑥, 𝑦𝑦〉.
Assume a disk 𝑅𝑅 = {(𝑥𝑥, 𝑦𝑦): 𝑥𝑥 2 + 𝑦𝑦 2 ≤ 𝑟𝑟 2 }. We can confirm that the integral ∮𝐶𝐶 𝑭𝑭 ∙ 𝒏𝒏𝒔𝒔 𝑑𝑑𝑑𝑑 =
2𝜋𝜋𝑟𝑟 2 , as discussed in Archived Lecture Video 21A. 𝐶𝐶 is oriented counter-clockwise and is
the boundary of 𝑅𝑅. Now confirm this result using Green’s Flux Theorem.

Solution
𝜕𝜕𝜕𝜕 𝜕𝜕𝜕𝜕
Note that 𝑓𝑓 (𝑥𝑥, 𝑦𝑦) = 𝑥𝑥 and 𝑔𝑔(𝑥𝑥, 𝑦𝑦) = 𝑦𝑦; therefore, the divergence of 𝑭𝑭 is + = 2. By
𝜕𝜕𝜕𝜕 𝜕𝜕𝜕𝜕
Green’s Flux Theorem,
2𝜋𝜋 𝑟𝑟
𝜕𝜕𝜕𝜕 𝜕𝜕𝜕𝜕
�𝑭𝑭 ∙ 𝒏𝒏𝒔𝒔 𝑑𝑑𝑑𝑑 = � � + � 𝑑𝑑𝑑𝑑 = � 2𝑑𝑑𝑑𝑑 = 2 � � 𝑟𝑟 ′𝑑𝑑′𝑟𝑟𝑟𝑟𝑟𝑟 = 2𝜋𝜋𝑟𝑟 2
𝐶𝐶 𝐴𝐴 𝜕𝜕𝜕𝜕 𝜕𝜕𝜕𝜕 𝑅𝑅 0 0

At this point you can work on the following problems which have been extracted from section 17.4
of the textbook.

4.1.7 Problems from Section 17.4: 7, 43 (flux), 47 (flux), 52, 65

1. (question 7) Sketch a two-dimensional vector field that has zero divergence everywhere in
the plane.

92
2. (question 43) Flux For the following vector field, compute the flux on the boundary of the
region. Assume boundary curves are oriented counterclockwise: 𝑭𝑭 = 〈2𝑥𝑥 + 𝑦𝑦, 𝑥𝑥 − 4𝑦𝑦〉: 𝑅𝑅
𝜋𝜋
is the quarter annulus �(𝑟𝑟, 𝜃𝜃 ): 1 ≤ 𝑟𝑟 ≤ 4, 0 ≤ 𝜃𝜃 ≤ �
2
3. (question 47) Flux For the following vector field, compute the flux on the boundary of the
region. Assume boundary curves are oriented counterclockwise: 𝑭𝑭 = 〈𝑥𝑥 + 𝑦𝑦 2 , 𝑥𝑥 2 − 𝑦𝑦〉:
𝑅𝑅 = {(𝑥𝑥, 𝑦𝑦): 𝑦𝑦 2 ≤ 𝑥𝑥 ≤ 2 − 𝑦𝑦 2 }
4. (question 52) Double integral to line integral Use the flux form of Green’s theorem to
evaluate ∬𝑅𝑅(2𝑥𝑥𝑥𝑥 + 4𝑦𝑦 2 )𝑑𝑑𝑑𝑑, where R is the triangle with vertices (0,0), (1,0) and (0,1).
5. (question 65) Green’s Theorem as a Fundamental Theorem of Calculus Show that if
𝑓𝑓(𝑥𝑥)
the flux form of Green’s theorem is applied to the vector field 〈 , 0〉, where 𝑐𝑐 > 0 and
𝑐𝑐
𝑅𝑅 = {(𝑥𝑥, 𝑦𝑦): 𝑎𝑎 ≤ 𝑥𝑥 ≤ 𝑏𝑏, 0 ≤ 𝑦𝑦 ≤ 𝑐𝑐 }, then the result is the Fundamental Theorem of
𝑏𝑏 𝑑𝑑𝑑𝑑
Calculus ∫𝑎𝑎 𝑑𝑑𝑑𝑑 = 𝑓𝑓 (𝑏𝑏) − 𝑓𝑓(𝑎𝑎).
𝑑𝑑𝑑𝑑

You can find the solutions to the problems at the following link: Solutions to Reading Assignment
4 Problems.

4.2 Stokes’ Theorem

In this section, we describe Stokes’ Theorem formulation for the 3d and 2d case.

Here we assume that:

• The vector tangent to the closed contour is continuous.


• The components of the vector valued function 𝑭𝑭 at each point on the closed contour 𝐶𝐶 are
continuous.
• The closed contour encloses the entire open region 𝑆𝑆 in 3d, and region 𝐴𝐴 in 2d.
• The partial derivatives of the vector valued function 𝑭𝑭, at every point in the region (curved
open surface 𝑆𝑆 enclosed by the contour 𝐶𝐶 in 3d, and planar surface 𝐴𝐴 enclosed by the
contour 𝐶𝐶 in 2d), are defined.

In section 4.7 we show how we can address the problem of an open surface in 3d which is not
simply connected. One case is a surface with voids. A second case is an open surface enclosed by
a contour where the surface contains one or more points where the vector valued function 𝑭𝑭 is not
defined. We define these cases as problematic cases.

4.2.1 3d formulation

With reference to Fig. 14, Stokes’ Theorem is given by

93
�𝑭𝑭 ∙ 𝑑𝑑𝒔𝒔 = � ∇ × 𝑭𝑭 ∙ 𝑑𝑑𝑺𝑺𝟏𝟏 = � ∇ × 𝑭𝑭 ∙ 𝑑𝑑𝑺𝑺𝟐𝟐
𝐶𝐶 𝑆𝑆1 𝑆𝑆2

𝒓𝒓̇ (𝑡𝑡)
where 𝑑𝑑𝒔𝒔 = 𝑻𝑻𝑑𝑑𝑑𝑑, 𝑻𝑻 = |𝒓𝒓̇ , 𝑑𝑑𝑑𝑑 = |𝒓𝒓̇ (𝑡𝑡)|𝑑𝑑𝑑𝑑 and thus 𝑻𝑻𝑑𝑑𝑑𝑑 = 𝒓𝒓̇ (𝑡𝑡)𝑑𝑑𝑑𝑑.
(𝑡𝑡)|

Fig. 14 Interpretation of Stokes’ Theorem in 3d

The parameterization of the contour 𝐶𝐶 is given by 𝒓𝒓 = 〈𝑥𝑥(𝑡𝑡), 𝑦𝑦(𝑡𝑡), 𝑧𝑧(𝑡𝑡)〉 and 0 ≤ 𝑡𝑡 < 2𝜋𝜋 assuming
we parameterize the closed contour in terms of functions of 𝑐𝑐𝑐𝑐𝑐𝑐(𝑡𝑡) and 𝑠𝑠𝑠𝑠𝑠𝑠(𝑡𝑡). The right-hand
convention (right hand rule) is applied to determine the relationship between the direction that the
contour is traversed and the orientation of the area vector. Place the right hand of your fingers in
the direction that the contour is traversed, that is in the direction of 𝑻𝑻. Then the thumb on the right-
hand points in the direction of the area vectors 𝑑𝑑𝑺𝑺𝟏𝟏 and 𝑑𝑑𝑺𝑺𝟐𝟐 .

It is clear from Figure 14, that if the contour is a given, the area chosen that is bounded by the
contour can be chosen at free will. This gives the user choices when solving problems, as will be
demonstrated once we consider applications of Stokes’ Theorem in Reading Assignment 5.

4.2.2 Exercises Illustrating the Use of Stokes Theorem

This set of exercises gives you the ability to understand how Stokes’ theorem can be utilized and
understand how to apply the correct conventions. Applications of Stokes’ Theorem will be
explored in greater detail in Assignment 5. The main objective here is to understand Stokes’
theorem as a tool.

1. Compute the line integral ∮𝐶𝐶 (𝑦𝑦 + 𝑠𝑠𝑠𝑠𝑠𝑠𝑠𝑠)𝑑𝑑𝑑𝑑 + (𝑧𝑧 2 + 𝑐𝑐𝑐𝑐𝑐𝑐𝑐𝑐)𝑑𝑑𝑑𝑑 + 𝑥𝑥 3 𝑑𝑑𝑑𝑑. Here C is the curve
𝒓𝒓(𝑡𝑡) = 〈𝑠𝑠𝑠𝑠𝑠𝑠𝑠𝑠, 𝑐𝑐𝑐𝑐𝑐𝑐𝑐𝑐, 𝑠𝑠𝑠𝑠𝑠𝑠2𝑡𝑡〉, 0 ≤ 𝑡𝑡 < 2𝜋𝜋. This curve lies on the surface defined by 𝑧𝑧 = 2𝑥𝑥𝑥𝑥.

94
Note: Any closed line integral can be associated with Stokes’ theorem so this is the first thing
you should recognize. Alternatively, any open surface which is bounded by a closed contour
can also be associated with Stokes’ theorem. You can refer to Archived Lecture Video 24A
for additional information.

Solution

Stokes’ theorem is

�⃑̇𝑑𝑑𝑑𝑑 = �(∇ × 𝑭𝑭) ∙ 𝒏𝒏𝑑𝑑𝑑𝑑


�𝑭𝑭 ∙ 𝑻𝑻𝑑𝑑𝑑𝑑 = �𝑭𝑭 ∙ 𝒓𝒓
𝐶𝐶 𝐶𝐶 𝑆𝑆

Question to ask yourself: is the RHS or LHS of Stokes’ theorem easier to compute?

The first thing to consider is the orientation of the curve 𝒓𝒓(𝑡𝑡). The position vector is given and
therefore the direction of the contour can be determined from the problem statement: it is clockwise
when looked from above.

• Greyed out area is the projection of the surface area in red, bounded by the closed contour
indicated in red, onto the 𝑥𝑥𝑥𝑥 − plane.
• 𝑻𝑻𝑑𝑑𝑑𝑑 shows the vector tangent to the contour in red.
It is traversed in a CW direction.
• 𝒏𝒏𝑺𝑺𝑑𝑑𝑑𝑑 is the orientation vector for the curved
surface, pointing downwards, according to the
right-hand rule convention.
• Explore this graph interactively by clicking on the
following link.
https://www.math3d.org/AZYTAQeTr

• Slider 𝑇𝑇: motion of 𝒏𝒏𝑺𝑺𝑑𝑑𝑑𝑑 on the surface shaded in red. Projection of the position vector
onto the 𝑥𝑥𝑥𝑥 −plane traces out a circle of a given radius determined by Slider 𝑃𝑃.
• Slider 𝑃𝑃: motion of 𝒏𝒏𝑺𝑺 𝑑𝑑𝑑𝑑 on the surface shaded in red. Projection of the position vector
onto the 𝑥𝑥𝑥𝑥 −plane traces out a radial line at a given angle determined by Slider 𝑇𝑇.

LHS of Stokes’ Theorem

Starting with the expression in the problem statement, we have:

� (𝑦𝑦 + 𝑠𝑠𝑠𝑠𝑠𝑠𝑠𝑠 )𝑑𝑑𝑑𝑑 + (𝑧𝑧 2 + 𝑐𝑐𝑐𝑐𝑐𝑐𝑐𝑐)𝑑𝑑𝑑𝑑 + 𝑥𝑥 3 𝑑𝑑𝑑𝑑 = � 〈𝑦𝑦 + 𝑠𝑠𝑠𝑠𝑠𝑠𝑠𝑠, 𝑧𝑧 2 + 𝑐𝑐𝑐𝑐𝑐𝑐𝑐𝑐, 𝑥𝑥 3 〉 ∙ 〈𝑑𝑑𝑑𝑑, 𝑑𝑑𝑑𝑑, 𝑑𝑑𝑑𝑑〉
𝐶𝐶 𝐶𝐶

95
where
𝑭𝑭 = 〈𝑦𝑦 + 𝑠𝑠𝑠𝑠𝑠𝑠𝑠𝑠, 𝑧𝑧 2 + 𝑐𝑐𝑐𝑐𝑐𝑐𝑐𝑐, 𝑥𝑥 3 〉

and

𝒓𝒓̇ 𝑑𝑑𝑑𝑑 = 〈𝑑𝑑𝑑𝑑, 𝑑𝑑𝑑𝑑, 𝑑𝑑𝑑𝑑〉


𝑻𝑻𝑑𝑑𝑑𝑑 = �⃑

The integral expression looks messy so instead we will explore the right-hand side for calculation
purposes.

RHS of Stokes’ Theorem

Looking down onto the 𝑥𝑥𝑥𝑥 plane, the contour is traversed in a clockwise direction, therefore, the
right-hand convention would stipulate that the area vector 𝒏𝒏𝑑𝑑𝑑𝑑 has a 𝑧𝑧 component which points in
the −𝑧𝑧̂ direction. The surface 𝑧𝑧 = 𝑓𝑓 (𝑥𝑥, 𝑦𝑦) = 2𝑥𝑥𝑥𝑥 is given in explicit form so the surface area vector
pointing downwards can be expressed in explicit form, that is:

𝒏𝒏𝑑𝑑𝑑𝑑 = 〈𝑓𝑓𝑥𝑥 , 𝑓𝑓𝑦𝑦 , −1〉𝑑𝑑𝑑𝑑𝑑𝑑𝑑𝑑

The vector pointing normal to the surface and downward is thus 𝒏𝒏𝑑𝑑𝑑𝑑 = 〈2𝑦𝑦, 2𝑥𝑥, −1〉𝑑𝑑𝑑𝑑𝑑𝑑𝑑𝑑.

∇ × 𝑭𝑭 = 〈−2𝑧𝑧, −3𝑥𝑥 2 , −1〉

We need to evaluate the vector ∇ × 𝑭𝑭 on the surface 𝑧𝑧 = 𝑓𝑓 (𝑥𝑥, 𝑦𝑦) = 2𝑥𝑥𝑥𝑥, which implies

∇ × 𝑭𝑭 = 〈−4𝑥𝑥𝑥𝑥, −3𝑥𝑥 2 , −1〉

Therefore

�∇ × 𝑭𝑭 ∙ 𝒏𝒏𝑑𝑑𝑑𝑑 = �〈−4𝑥𝑥𝑥𝑥, −3𝑥𝑥 2 , −1〉 ∙ 〈2𝑦𝑦, 2𝑥𝑥, −1〉𝑑𝑑𝑑𝑑𝑑𝑑𝑑𝑑 = � (−8𝑥𝑥𝑦𝑦 2 − 6𝑥𝑥 3 + 1)𝑑𝑑𝑑𝑑𝑑𝑑𝑑𝑑
𝑆𝑆 𝑆𝑆 𝑆𝑆

= − �(8𝑥𝑥𝑦𝑦 2 + 6𝑥𝑥 3 − 1)𝑑𝑑𝑑𝑑𝑑𝑑𝑑𝑑


𝑆𝑆

The bounds for the region of integration are determined by the projection of the contour onto the
𝑥𝑥𝑥𝑥 plane. The bounds can be easily determined by taking the original position vector and setting
the 𝑧𝑧 component of the vector field to 0, which implies,

𝒓𝒓(𝑡𝑡) = 〈𝑠𝑠𝑠𝑠𝑠𝑠𝑠𝑠, 𝑐𝑐𝑐𝑐𝑐𝑐𝑐𝑐, 0〉, 0 ≤ 𝑡𝑡 < 2𝜋𝜋

which is the definition of a circle of radius 1 on the 𝑥𝑥𝑥𝑥 plane. Thus

1 √1−𝑥𝑥 2
�∇ × 𝑭𝑭 ∙ 𝒏𝒏𝑑𝑑𝑑𝑑 = − � � (8𝑥𝑥𝑦𝑦 2 + 6𝑥𝑥 3 − 1)𝑑𝑑𝑑𝑑𝑑𝑑𝑑𝑑
𝑆𝑆 −1 −√1−𝑥𝑥 2

This integral is easier to solve in polar coordinates, so the transformed integral becomes:

96
2𝜋𝜋
1 √1−𝑥𝑥 2 1
−� � (8𝑥𝑥𝑦𝑦 2 + 6𝑥𝑥 3 − 1)𝑑𝑑𝑑𝑑𝑑𝑑𝑑𝑑 = − � � (8𝑟𝑟 3 𝑐𝑐𝑐𝑐𝑐𝑐𝑐𝑐𝑠𝑠𝑠𝑠𝑠𝑠2 𝜃𝜃 + 6𝑟𝑟 3 𝑐𝑐𝑐𝑐𝑐𝑐 3𝜃𝜃 − 1)𝑟𝑟𝑟𝑟𝑟𝑟𝑟𝑟𝑟𝑟
−1 −√1−𝑥𝑥 2 0
0

The solution to the transformed integral equation is:


2𝜋𝜋
8 6 1
−� � 𝑐𝑐𝑜𝑜𝑜𝑜𝑜𝑜𝑠𝑠𝑠𝑠𝑠𝑠2 𝜃𝜃 + 𝑐𝑐𝑐𝑐𝑐𝑐 3 𝜃𝜃 − � 𝑑𝑑𝑑𝑑 = 𝜋𝜋
0 5 5 2

2. Evaluate the integral ∮𝐶𝐶 𝑭𝑭 ∙ 𝒓𝒓̇ (𝑡𝑡)𝑑𝑑𝑑𝑑 where 𝑭𝑭 = ⟨𝑧𝑧, −𝑧𝑧, 𝑥𝑥 2 − 𝑦𝑦 2 ⟩. 𝐶𝐶 consists of the three-line
segments that bound the plane 𝑧𝑧 = 8 − 4𝑥𝑥 − 2𝑦𝑦 in the first octant oriented according to the
figure below. The vertices of the triangle are (0,0,8), (2,0,0) and (0,4,0). (refer to Archived
Lecture Video 24A).

97
Solution

The application of Stokes’ Theorem for a given closed contour makes no constraints on the shape
of surface enclosed by the contour. Hence in this example we choose two different surfaces to
demonstrate that Stokes’ Theorem does not depend on the choice of the surface.

• Surface 1: Blue area represents the surface of the


top of the triangular box (shaded in blue). 𝒏𝒏𝑺𝑺𝑑𝑑𝑆𝑆4 is
the orientation vector for this surface.
• Surface 2: 3 grey triangular areas stitched together
represent the surface of the pyramidal box 𝒏𝒏𝑺𝑺 𝑑𝑑𝑆𝑆𝑖𝑖 is
the orientation vector for a given triangular surface:
𝑖𝑖 = 1,2,3 is the index, representing the three
surfaces in question.
• 𝑻𝑻𝑑𝑑𝑑𝑑 shows the vector tangent to the contour in red.
• Direction of the unit normal vectors is consistent
with the right-hand rule.
• Explore this graph interactively by clicking on the
following link.
https://www.math3d.org/StDVMX7cS

• Slider 𝑇𝑇: move the top face of the triangular shaped


box (shaded in blue) forwards so that the inside
surface of the box indicated in grey (three different
shades corresponding to three discrete triangular
surfaces) can be viewed.

In this problem, you can consider the surface shaded in blue as one surface and the three surfaces
shaded in grey, stitched together, as another possible surface bounded by the same contour 𝐶𝐶.
Stokes’ theorem is

�⃑̇𝑑𝑑𝑑𝑑 = �(∇ × 𝑭𝑭) ∙ 𝒏𝒏𝑑𝑑𝑑𝑑


�𝑭𝑭 ∙ 𝑻𝑻𝑑𝑑𝑑𝑑 = �𝑭𝑭 ∙ 𝒓𝒓
𝐶𝐶 𝐶𝐶 𝑆𝑆
Left − hand side Right − hand side

Question to ask yourself: is the RHS or LHS of Stokes’ theorem easier to compute?

The first thing to consider is the orientation of the curve 𝒓𝒓(𝑡𝑡). The position vector is given and
therefore the direction of the contour can be determined from the problem statement: it is counter-
clockwise, looking from above.

𝐿𝐿𝐿𝐿𝐿𝐿 of Stokes’ Theorem

If you were to write out the 𝐿𝐿𝐿𝐿𝐿𝐿 of Stokes’ theorem, you would start with:

98
𝑑𝑑𝑑𝑑 𝑑𝑑𝑑𝑑 𝑑𝑑𝑑𝑑
𝑑𝑑𝑑𝑑 = 𝑑𝑑𝑑𝑑, 𝑑𝑑𝑑𝑑 = 𝑑𝑑𝑑𝑑 and 𝑑𝑑𝑑𝑑 = 𝑑𝑑𝑑𝑑
𝑑𝑑𝑑𝑑 𝑑𝑑𝑑𝑑 𝑑𝑑𝑑𝑑

𝒓𝒓̇ 𝑑𝑑𝑑𝑑 = 〈𝑑𝑑𝑑𝑑, 𝑑𝑑𝑑𝑑, 𝑑𝑑𝑑𝑑〉.


𝑑𝑑𝒓𝒓 𝑑𝑑𝑑𝑑 𝑑𝑑𝑑𝑑 𝑑𝑑𝑑𝑑
Recall that 𝑻𝑻𝑑𝑑𝑑𝑑 = 𝑑𝑑𝑑𝑑 = 〈 , , 〉 𝑑𝑑𝑑𝑑 = �⃑
𝑑𝑑𝑑𝑑 𝑑𝑑𝑑𝑑 𝑑𝑑𝑑𝑑 𝑑𝑑𝑑𝑑

The closed contour in this problem consists of the sum of three open contour integrals

𝒓𝒓̇ 𝑑𝑑𝑑𝑑 = � 𝑭𝑭 ∙ 𝒓𝒓̇ 𝟏𝟏 𝑑𝑑𝑑𝑑 + � 𝑭𝑭 ∙ 𝒓𝒓̇ 𝟐𝟐 𝑑𝑑𝑑𝑑 + � 𝑭𝑭 ∙ 𝒓𝒓̇ 𝟑𝟑 𝑑𝑑𝑑𝑑


�𝑭𝑭 ∙ 𝑻𝑻𝑑𝑑𝑑𝑑 = �𝑭𝑭 ∙ �⃑
𝐶𝐶 𝐶𝐶 𝐶𝐶1 𝐶𝐶2 𝐶𝐶3
Each segment must be parameterized independently. The results are as follows:

𝐶𝐶1 → 𝒓𝒓𝟏𝟏 = ⟨2 − 𝑡𝑡, 2𝑡𝑡, 0⟩ 0 ≤ 𝑡𝑡 ≤ 2: 𝒓𝒓̇ 𝟏𝟏 = ⟨−1,2,0⟩ 𝑭𝑭 = ⟨0,0, 4 − 4𝑡𝑡 − 3𝑡𝑡 2 ⟩


𝐶𝐶2 → 𝒓𝒓𝟐𝟐 = ⟨0,4 − 𝑡𝑡, 2𝑡𝑡⟩ 0 ≤ 𝑡𝑡 ≤ 4: 𝒓𝒓̇ 𝟐𝟐 = ⟨0, −1,2⟩ 𝑭𝑭 = ⟨2𝑡𝑡, −2𝑡𝑡, −(4 − 𝑡𝑡)2 ⟩
𝐶𝐶3 → 𝒓𝒓𝟑𝟑 = ⟨𝑡𝑡, 0,8 − 4𝑡𝑡⟩ 0 ≤ 𝑡𝑡 ≤ 2: 𝒓𝒓̇ 𝟑𝟑 = ⟨1,0, −4⟩ 𝑭𝑭 = ⟨8 − 4𝑡𝑡, 4𝑡𝑡 − 8, 𝑡𝑡 2 ⟩

Integral for Open Contour 𝐶𝐶1

2 2
� 𝑭𝑭 ∙ 𝒓𝒓̇ 𝟏𝟏 𝑑𝑑𝑑𝑑 = � ⟨0,0, 4 − 4𝑡𝑡 − 3𝑡𝑡 2 ⟩ ∙ ⟨−1,2,0⟩𝑑𝑑𝑑𝑑 = � 0𝑑𝑑𝑑𝑑 = 0
𝐶𝐶1 0 0

Integral for Open Contour 𝐶𝐶2

4 4
80
� 𝑭𝑭 ∙ 𝒓𝒓̇ 𝟐𝟐 𝑑𝑑𝑑𝑑 = � ⟨2𝑡𝑡, −2𝑡𝑡, −(4 − 𝑡𝑡)2 ⟩ ∙ ⟨0, −1,2⟩𝑑𝑑𝑑𝑑 = � (−2𝑡𝑡 2 + 18𝑡𝑡 − 32)𝑑𝑑𝑑𝑑 = −
𝐶𝐶2 0 0 3

Integral for Open Contour 𝐶𝐶3

2 2
2⟩
8
� 𝑭𝑭 ∙ 𝒓𝒓̇ 𝟑𝟑 𝑑𝑑𝑑𝑑 = � ⟨8 − 4𝑡𝑡, 4𝑡𝑡 − 8, 𝑡𝑡 ∙ ⟨1,0, −4⟩𝑑𝑑𝑑𝑑 = � (8 − 4𝑡𝑡 − 4𝑡𝑡 2 )𝑑𝑑𝑑𝑑 = −
𝐶𝐶3 0 0 3

Closed Contour Integral

88
𝒓𝒓̇ 𝑑𝑑𝑑𝑑 = � 𝑭𝑭 ∙ 𝒓𝒓̇ 𝟏𝟏 𝑑𝑑𝑑𝑑 + � 𝑭𝑭 ∙ 𝒓𝒓̇ 𝟐𝟐 𝑑𝑑𝑑𝑑 + � 𝑭𝑭 ∙ 𝒓𝒓̇ 𝟑𝟑 𝑑𝑑𝑑𝑑 = −
�𝑭𝑭 ∙ �⃑
𝐶𝐶 𝐶𝐶1 𝐶𝐶2 𝐶𝐶3 3

The circulation around the boundary is negative indicating a net circulation in the 𝐶𝐶𝐶𝐶 looking
from above.

𝑅𝑅𝑅𝑅𝑅𝑅 of Stokes’ Theorem

Consider two different open surfaces bounded by a common contour 𝐶𝐶. We will show that for
either surface we obtain the same result for ∬𝑆𝑆(∇ × 𝑭𝑭) ∙ 𝒏𝒏𝑑𝑑𝑑𝑑.

99
Triangular planar surface shaded in blue
The first calculation to perform is:
𝑥𝑥� 𝑦𝑦� 𝑧𝑧̂
⎡ ⎤
𝜕𝜕 𝜕𝜕 𝜕𝜕
∇ × 𝑭𝑭 = ⎢ ⎥�� = ⟨1 − 2𝑦𝑦, 1 − 2𝑥𝑥, 0⟩
⎢𝜕𝜕𝜕𝜕 𝜕𝜕𝜕𝜕 𝜕𝜕𝜕𝜕 ⎥
⎣ 𝑧𝑧 −𝑧𝑧 𝑥𝑥 2 − 𝑦𝑦 2 ⎦ 𝑑𝑑𝑑𝑑𝑑𝑑

Use explicit representation to compute 𝒏𝒏𝑺𝑺𝑺𝑺 𝑑𝑑𝑑𝑑

The plane shaded in blue is represented by the following equation: 𝑧𝑧 = 8 − 4𝑥𝑥 − 2𝑦𝑦. Therefore

𝒏𝒏𝑺𝑺𝑺𝑺 𝑑𝑑𝑑𝑑 = �−𝑧𝑧𝑥𝑥 , −𝑧𝑧𝑦𝑦 , 1�𝑑𝑑𝑑𝑑𝑑𝑑𝑑𝑑 = ⟨4,2,1⟩𝑑𝑑𝑑𝑑𝑑𝑑𝑑𝑑

Direction of the normal vector is according to the right-hand convention

Region 𝑅𝑅

The triangular region 𝑅𝑅 in the 𝑥𝑥𝑥𝑥 plane beneath the plane 𝑧𝑧 = 8 − 4𝑥𝑥 − 2𝑦𝑦 is found by setting 𝑧𝑧 =
0 in the equation of the oriented plane. It is the area shaded almost in white on the 𝑥𝑥𝑥𝑥 − plane.

𝑅𝑅 = {(𝑥𝑥, 𝑦𝑦): ; 0 ≤ 𝑥𝑥 ≤ 2, 0 ≤ 𝑦𝑦 ≤ 4 − 2𝑥𝑥 } projected area

Calculation of the 𝑅𝑅𝑅𝑅𝑅𝑅:

2 4−2𝑥𝑥
�(∇ × 𝑭𝑭) ∙ 𝒏𝒏𝑺𝑺𝑺𝑺 𝑑𝑑𝑑𝑑 = � ⟨1 − 2𝑦𝑦, 1 − 2𝑥𝑥, 0⟩ ∙ ⟨4,2,1⟩𝑑𝑑𝑑𝑑 = � � (6 − 4𝑥𝑥 − 8𝑦𝑦)𝑑𝑑𝑑𝑑𝑑𝑑𝑑𝑑
𝑆𝑆 𝑅𝑅 0 0
2 2
88
= � (6𝑦𝑦 − 4𝑥𝑥𝑥𝑥 − 4𝑦𝑦 2 )|4−2𝑥𝑥
0 𝑑𝑑𝑑𝑑 = � (−40 + 36𝑥𝑥 − 8𝑥𝑥 2 ) 𝑑𝑑𝑑𝑑 = −
0 0 3

This result is consistent with the result for the 𝐿𝐿𝐿𝐿𝐿𝐿.

Three discrete triangular grey surfaces stitched together

In this case the integral on the 𝑅𝑅𝑅𝑅𝑅𝑅 is written as follows:

�⃑̇𝑑𝑑𝑑𝑑 = �(∇ × 𝑭𝑭) ∙ 𝒏𝒏𝑑𝑑𝑑𝑑


�𝑭𝑭 ∙ 𝑻𝑻𝑑𝑑𝑑𝑑 = �𝑭𝑭 ∙ 𝒓𝒓
𝐶𝐶 𝐶𝐶 𝑆𝑆

= � (∇ × 𝑭𝑭) ∙ 𝒏𝒏𝑺𝑺𝟏𝟏 𝑑𝑑𝑑𝑑1 + � (∇ × 𝑭𝑭) ∙ 𝒏𝒏𝑺𝑺𝟐𝟐 𝑑𝑑𝑑𝑑2 + � (∇ × 𝑭𝑭) ∙ 𝒏𝒏𝑺𝑺𝟑𝟑 𝑑𝑑𝑑𝑑3


𝑆𝑆1 𝑆𝑆2 𝑆𝑆3

Calculate the Differential Area Vector 𝒏𝒏𝑺𝑺𝐢𝐢 𝑑𝑑𝑑𝑑i 𝑖𝑖 = 1,2,3 and ∇ × 𝑭𝑭, for each triangular plane:

100
Plane ∇ × 𝑭𝑭 Differential Area Vector
𝑧𝑧 = 0 ∇ × 𝑭𝑭|𝑧𝑧=0 = ⟨1 − 2𝑦𝑦, 1 − 2𝑥𝑥, 0⟩ 𝒏𝒏𝑺𝑺𝟏𝟏 𝑑𝑑𝑑𝑑1 = ⟨0,0,1⟩𝑑𝑑𝑑𝑑𝑑𝑑𝑑𝑑
𝑥𝑥 = 0 ∇ × 𝑭𝑭|𝑥𝑥=0 = ⟨1 − 2𝑦𝑦, 1,0⟩ 𝒏𝒏𝑺𝑺𝟐𝟐 𝑑𝑑𝑑𝑑2 = ⟨1,0,0⟩𝑑𝑑𝑑𝑑𝑑𝑑𝑑𝑑
𝑦𝑦 = 0 ∇ × 𝑭𝑭|𝑦𝑦=0 = ⟨1,1 − 2𝑥𝑥, 0⟩ 𝒏𝒏𝑺𝑺𝟑𝟑 𝑑𝑑𝑑𝑑3 = ⟨0,1,0⟩𝑑𝑑𝑑𝑑𝑑𝑑𝑑𝑑

Region 𝑆𝑆:
𝑆𝑆1 → {(𝑥𝑥, 𝑦𝑦): 0 ≤ 𝑦𝑦 ≤ 4 − 2𝑥𝑥, 0 ≤ 𝑥𝑥 ≤ 2}
𝑆𝑆2 → {(𝑦𝑦, 𝑧𝑧): 0 ≤ 𝑧𝑧 ≤ 8 − 2𝑦𝑦, 0 ≤ 𝑦𝑦 ≤ 4}
𝑧𝑧
𝑆𝑆3 → �(𝑥𝑥, 𝑧𝑧): 0 ≤ 𝑥𝑥 ≤ 2 − , 0 ≤ 𝑧𝑧 ≤ 8�
4

Calculating the Integrand on the 𝐿𝐿𝐿𝐿𝐿𝐿:


∇ × 𝑭𝑭|𝑧𝑧=0 ∙ 𝒏𝒏𝑺𝑺𝟏𝟏 𝑑𝑑𝑑𝑑1 = 0
∇ × 𝑭𝑭|𝑥𝑥=0 ∙ 𝒏𝒏𝑺𝑺𝟐𝟐 𝑑𝑑𝑑𝑑2 = (1 − 2𝑦𝑦)𝑑𝑑𝑑𝑑𝑑𝑑𝑑𝑑
∇ × 𝑭𝑭|𝑦𝑦=0 ∙ 𝒏𝒏𝑺𝑺𝟑𝟑 𝑑𝑑𝑑𝑑3 = (1 − 2𝑥𝑥 )𝑑𝑑𝑑𝑑𝑑𝑑𝑑𝑑

Final Result:

� (∇ × 𝑭𝑭) ∙ 𝒏𝒏𝑺𝑺𝟏𝟏 𝑑𝑑𝑑𝑑1 + � (∇ × 𝑭𝑭) ∙ 𝒏𝒏𝑺𝑺𝟐𝟐 𝑑𝑑𝑑𝑑2 + � (∇ × 𝑭𝑭) ∙ 𝒏𝒏𝑺𝑺𝟑𝟑 𝑑𝑑𝑑𝑑3


𝑆𝑆1 𝑆𝑆2 𝑆𝑆3
𝑧𝑧
4 8−2𝑦𝑦 8 2−
4 88
=0+� � (1 − 2𝑦𝑦)𝑑𝑑𝑑𝑑𝑑𝑑𝑑𝑑 + � � (1 − 2𝑥𝑥 )𝑑𝑑𝑑𝑑𝑑𝑑𝑑𝑑 = −
0 0 0 0 3

This result is consistent with the result for the 𝐿𝐿𝐿𝐿𝐿𝐿.

At this point you can work on the following problems which have been extracted from section 17.7
of the textbook:

4.2.3 Problems from Section 17.7: 1, 3, 5, 9, 13, 15, 19, 29(a-c), 33, 41, 42, 43, 45,
47, 50

1. (question 1) Explain the meaning of the integral ∮𝐶𝐶 𝑭𝑭 ∙ 𝒅𝒅𝒅𝒅 in Stokes’ Theorem
2. (question 3) Explain the meaning of Stokes’ Theorem
3. (question 5) Verifying Stokes’ Theorem Verify that the line integral and the surface
integral of Stokes’ Theorem are equal for the following vector field, surface 𝑆𝑆 and closed
contour 𝐶𝐶. Assume 𝐶𝐶 has a counterclockwise orientation and 𝑆𝑆 has a consistent orientation:
𝑭𝑭 = 〈𝑦𝑦, −𝑥𝑥, 10〉: 𝑆𝑆 is the upper half of the sphere 𝑥𝑥 2 + 𝑦𝑦 2 + 𝑧𝑧 2 = 1 and 𝐶𝐶 is the circle
𝑥𝑥 2 + 𝑦𝑦 2 = 1 in the 𝑥𝑥𝑥𝑥 − plane.

101
4. (question 9) Verifying Stokes’ Theorem Verify that the line integral and the surface
integral of Stokes’ Theorem are equal for the following vector field, surface 𝑆𝑆 and closed
contour 𝐶𝐶. Assume 𝐶𝐶 has a counterclockwise orientation and 𝑆𝑆 has a consistent orientation:
𝑭𝑭 = 〈𝑦𝑦 − 𝑧𝑧, 𝑧𝑧 − 𝑥𝑥, 𝑥𝑥 − 𝑦𝑦〉: 𝑆𝑆 is the cap of the sphere 𝑥𝑥 2 + 𝑦𝑦 2 + 𝑧𝑧 2 = 16 above the plane
𝑧𝑧 = √7 and 𝐶𝐶 is the boundary of 𝑆𝑆.
5. (question 13) Stokes’ Theorem for evaluating line integrals Evaluate the line integral
∮𝐶𝐶 𝑭𝑭 ∙ 𝒅𝒅𝒅𝒅 by evaluating the surface integral in Stokes’ Theorem with an appropriate choice
of 𝑆𝑆. Assume 𝐶𝐶 has a counterclockwise orientation: 𝑭𝑭 = 〈𝑥𝑥 2 − 𝑧𝑧 2 , 𝑦𝑦, 2𝑥𝑥𝑥𝑥〉: 𝐶𝐶 is the
boundary of the plane 𝑧𝑧 = 4 − 𝑥𝑥 − 𝑦𝑦 in the first octant.
6. (question 15) Stokes’ Theorem for evaluating line integrals Evaluate the line integral
∮𝐶𝐶 𝑭𝑭 ∙ 𝒅𝒅𝒅𝒅 by evaluating the surface integral in Stokes’ Theorem with an appropriate choice
of 𝑆𝑆. Assume 𝐶𝐶 has a counterclockwise orientation: 𝑭𝑭 = 〈𝑦𝑦 2 , −𝑧𝑧 2 , 𝑥𝑥 〉: 𝐶𝐶 is the circle 𝒓𝒓 =
〈3𝑐𝑐𝑐𝑐𝑐𝑐𝑐𝑐, 4𝑐𝑐𝑐𝑐𝑐𝑐𝑐𝑐, 5𝑠𝑠𝑠𝑠𝑠𝑠𝑠𝑠〉 for 0 ≤ 𝑡𝑡 ≤ 2𝜋𝜋.
7. (question 19) Stokes’ Theorem for evaluating surface integrals Evaluate the line integral
in Stokes’ Theorem to determine the value of the surface integral ∬𝑆𝑆 ∇ × 𝑭𝑭 ∙ 𝒏𝒏𝑑𝑑𝑑𝑑. Assume
𝒏𝒏 points in an upward direction: 𝐹𝐹 = 〈2𝑦𝑦, −𝑧𝑧, 𝑥𝑥 − 𝑦𝑦 − 𝑧𝑧〉: 𝑆𝑆 is the cap of the sphere 𝑥𝑥 2 +
𝑦𝑦 2 + 𝑧𝑧 2 = 25, for 3 ≤ 𝑥𝑥 ≤ 5 (excluding its base).
8. (question 29 a-c) Explain why or why not Determine whether the following statements
are true and give an explanation or counterexample.
a. A paddle wheel with its axis in the direction 〈0,1, −1〉 would not spin when put in
the vector field 𝑭𝑭 = 〈1,1,2〉 × 〈𝑥𝑥, 𝑦𝑦, 𝑧𝑧〉.
b. Stokes’ Theorem relates the flux of a vector field 𝑭𝑭 across a surface to values of 𝑭𝑭
on the boundary of the surface.
c. A vector field of the form 𝐹𝐹 = 〈𝑎𝑎 + 𝑓𝑓 (𝑥𝑥 ), 𝑏𝑏 + 𝑔𝑔(𝑦𝑦), 𝑐𝑐 + ℎ(𝑧𝑧)〉, where 𝑎𝑎, 𝑏𝑏 and 𝑐𝑐
are constants, has zero circulation on a closed curve.
9. (question 33) Conservative field (∮𝑪𝑪 𝑭𝑭 ∙ 𝒅𝒅𝒅𝒅 = 𝟎𝟎 or 𝛁𝛁 × 𝑭𝑭 = 𝟎𝟎) Use Stokes’ Theorem to
find the circulation of the following vector field around any smooth simple closed curve:
𝐹𝐹 = 〈𝑦𝑦 2 𝑧𝑧 3 , 2𝑥𝑥𝑥𝑥𝑧𝑧 3 , 3𝑥𝑥𝑦𝑦 2 𝑧𝑧 2 〉.
10. (question 41) Compound surface and boundary Begin with the paraboloid 𝑧𝑧 = 𝑥𝑥 2 + 𝑦𝑦 2 ,
for 0 ≤ 𝑧𝑧 ≤ 4 and slice it with the plane 𝑦𝑦 = 0. Let 𝑆𝑆 be the surface that remains for 𝑦𝑦 ≥ 0
(including the plane surface in the 𝑥𝑥𝑥𝑥 − plane) (refer to figure below). Let 𝐶𝐶 be the
semicircle and line segment that bound the cap of 𝑆𝑆 in the plane 𝑧𝑧 = 4 with
counterclockwise orientation. Let 𝑭𝑭 = 〈2𝑧𝑧 + 𝑦𝑦, 2𝑥𝑥 + 𝑧𝑧, 2𝑦𝑦 + 𝑥𝑥 〉.
a. Describe the direction of the vectors normal to the surface that are consistent with
the orientation of 𝐶𝐶.
b. Evaluate ∬𝑆𝑆 ∇ × 𝑭𝑭 ∙ 𝒏𝒏𝑑𝑑𝑑𝑑 by computing ∇ × 𝑭𝑭 and appealing to symmetry.
c. Evaluate ∮𝐶𝐶 𝑭𝑭 ∙ 𝒅𝒅𝒅𝒅 and check for agreement with part(b).

102
11. (question 42) Ampere’s Law The French physicist André-Marie Ampere (1775-1836)
discovered that an electrical current 𝐼𝐼 in a wire produces a magnetic field 𝑯𝑯. A special case
of Ampere’s Law relates the current to the magnetic field though the equation ∮𝐶𝐶 𝑯𝑯 ∙ 𝒅𝒅𝒅𝒅 =
𝐼𝐼, where 𝐶𝐶 is any closed curve through which the wire passes. Assume the current is given
in terms of the current density 𝑱𝑱 as 𝐼𝐼 = ∬𝑆𝑆 𝑱𝑱 ∙ 𝒏𝒏𝑑𝑑𝑑𝑑, where 𝑆𝑆 is an oriented surface with 𝐶𝐶
as a boundary. Use Stokes’ Theorem to show that an equivalent form of Ampere’s Law is
∇ × 𝑯𝑯 = 𝑱𝑱
12. (question 43) Maximum surface integral Let S be the paraboloid 𝑧𝑧 = 𝑎𝑎(1 − 𝑥𝑥 2 − 𝑦𝑦 2 ),
for 𝑧𝑧 ≥ 0 is a real number. Let 𝐹𝐹 = 〈𝑥𝑥 − 𝑦𝑦, 𝑦𝑦 + 𝑧𝑧, 𝑧𝑧 − 𝑥𝑥 〉. For what value(s) of a (if any)
does ∬𝑆𝑆 ∇ × 𝑭𝑭 ∙ 𝒏𝒏𝑑𝑑𝑑𝑑 have its maximum value.
13. (question 45) Choosing a more convenient surface The goal is to evaluate 𝐴𝐴 =
∬𝑆𝑆 ∇ × 𝑭𝑭 ∙ 𝒏𝒏𝑑𝑑𝑑𝑑, where 𝑭𝑭 = 〈𝑦𝑦𝑦𝑦, −𝑥𝑥𝑥𝑥, 𝑥𝑥𝑥𝑥〉 and 𝑆𝑆 is the upper half of the ellipsoid 𝑥𝑥 2 + 𝑦𝑦 2 +
8𝑧𝑧 2 = 1 (𝑧𝑧 ≥ 0).
a. Evaluate a surface integral over a more convenient surface to find the value of 𝐴𝐴.
b. Evaluate 𝐴𝐴 using a line integral.
𝑦𝑦 𝑥𝑥
14. (question 47) Zero curl Consider the vector field 𝑭𝑭 = 〈− 2 2 𝒊𝒊 + 2 2 𝒋𝒋 + 𝑧𝑧𝒌𝒌〉
𝑥𝑥 +𝑦𝑦 𝑥𝑥 +𝑦𝑦
a. Show that ∇ × 𝑭𝑭 = 𝟎𝟎
b. Show that ∮𝐶𝐶 𝑭𝑭 ∙ 𝒅𝒅𝒅𝒅 is not zero on a circle 𝐶𝐶 in the 𝑥𝑥𝑥𝑥 − plane enclosing the origin.
c. Explain why Stokes’ Theorem does not apply in this case.
15. (question 50) Stokes’ Theorem on closed surfaces Prove that if 𝑭𝑭 satisfies the conditions
of Stokes’ Theorem, then ∯𝑆𝑆 ∇ × 𝑭𝑭 ∙ 𝒏𝒏𝑑𝑑𝑑𝑑 = 0, where 𝑆𝑆 is a smooth surface that encloses
the region.
16. Suppose that 𝑭𝑭 = 〈𝐹𝐹𝑥𝑥 , 𝐹𝐹𝑦𝑦 , 𝐹𝐹𝑧𝑧 〉 (subscripts here indicating vector components, not partial
derivatives) is a continuously differentiable vector field. Let 𝐶𝐶𝑅𝑅 be the boundary of the disc
of radius 𝑅𝑅 centred at (0,0,0) with normal 𝒏𝒏 = 〈0,1,0〉.

103
17. Suppose that 𝑭𝑭 is a continuously differentiable vector field. Let 𝐶𝐶 be the circle of radius 2,
𝜖𝜖
traversed in a counter-clockwise direction. Suppose as well that ‖∇ × 𝐹𝐹 ‖ ≤ ( 2 2 2). Use
+𝑦𝑦 +𝑧𝑧𝑥𝑥
Stokes’ theorem to show that:

��𝐹𝐹 ∙ 𝑇𝑇𝑇𝑇𝑇𝑇� ≤ 2𝜋𝜋𝜋𝜋


𝐶𝐶

You can find the solutions to the problems at the following link: Solutions to Reading Assignment
4 Problems.

4.2.4 2d formulation

𝜕𝜕𝜕𝜕 𝜕𝜕𝜕𝜕
Stokes’ Theorem in 2d is given by ∮𝐶𝐶 𝑭𝑭𝟐𝟐𝟐𝟐 ∙ 𝑻𝑻𝟐𝟐𝟐𝟐 𝑑𝑑𝑑𝑑 = ∬𝐴𝐴 �𝜕𝜕𝜕𝜕 − 𝜕𝜕𝜕𝜕� 𝑑𝑑𝑑𝑑 where 𝑭𝑭𝟐𝟐𝟐𝟐 =
〈𝑓𝑓(𝑥𝑥, 𝑦𝑦), 𝑔𝑔(𝑥𝑥, 𝑦𝑦)〉 and the differential arc length 𝑻𝑻𝟐𝟐𝟐𝟐 𝑑𝑑𝑑𝑑 is in the 𝑥𝑥𝑥𝑥 plane. The equivalence
expression above is referred to as Green’s Circulation theorem. Chapter 17.4 of the book devotes
a special section to Green’s theorem but you do not need to look at this section unless you want a
different perspective. We will now prove Green’s theorem from Stokes’ theorem.

Proof

We begin with Stokes’ theorem given by

�𝑭𝑭 ∙ 𝑑𝑑𝒔𝒔 = �∇ × 𝑭𝑭 ∙ 𝒅𝒅𝒅𝒅


𝐶𝐶 𝑆𝑆

With reference to Figure 15, define 𝑭𝑭 = 〈𝑓𝑓(𝑥𝑥, 𝑦𝑦), 𝑔𝑔(𝑥𝑥, 𝑦𝑦), 0〉, 𝑭𝑭𝟐𝟐𝟐𝟐 = 〈𝑓𝑓(𝑥𝑥, 𝑦𝑦), 𝑔𝑔(𝑥𝑥, 𝑦𝑦)〉, and

𝒅𝒅𝒅𝒅 = 〈0,0,1〉𝑑𝑑𝑑𝑑𝑑𝑑𝑑𝑑 = 〈0,0,1〉𝑑𝑑𝑑𝑑. Let 𝒓𝒓 = 〈𝑥𝑥(𝜃𝜃), 𝑦𝑦(𝜃𝜃),0〉 with 0 < 𝜃𝜃 ≤ 2𝜋𝜋.

Fig. 15 Interpretation of Stokes’ Theorem in 2d (Green’s Circulation Theorem)

104
and 𝒅𝒅𝒅𝒅 = 𝑻𝑻𝑑𝑑𝑑𝑑 = 〈𝑥𝑥 ′(𝜃𝜃 ), 𝑦𝑦 ′(𝜃𝜃 ), 0〉𝑑𝑑𝑑𝑑 or 𝒅𝒅𝒅𝒅 = 𝑻𝑻𝟐𝟐𝟐𝟐 𝑑𝑑𝑑𝑑 = 〈𝑥𝑥 ′ (𝜃𝜃 ), 𝑦𝑦 ′(𝜃𝜃 )〉𝑑𝑑𝑑𝑑

Then

𝚤𝚤̂ 𝚥𝚥̂ 𝑘𝑘�


𝜕𝜕 𝜕𝜕 𝜕𝜕 � 𝜕𝜕𝜕𝜕 𝜕𝜕𝜕𝜕
∇ × 𝑭𝑭 = �� � = � − � 〈0,0,1〉
𝜕𝜕𝜕𝜕 𝜕𝜕𝜕𝜕 𝜕𝜕𝜕𝜕 𝜕𝜕𝜕𝜕 𝜕𝜕𝜕𝜕
𝑓𝑓 𝑔𝑔 0 𝑑𝑑𝑑𝑑𝑑𝑑

Stokes’ Theorem when applied to this problem becomes


𝜕𝜕𝜕𝜕 𝜕𝜕𝜕𝜕 𝜕𝜕𝜕𝜕 𝜕𝜕𝜕𝜕
∬𝑆𝑆 ∇ × 𝑭𝑭 ∙ 𝒅𝒅𝒅𝒅 = ∬𝑆𝑆 �𝜕𝜕𝜕𝜕 − 𝜕𝜕𝜕𝜕� 〈0,0,1〉 ∙ 〈0,0,1〉𝑑𝑑𝑑𝑑𝑑𝑑𝑑𝑑 = ∬𝐴𝐴 �𝜕𝜕𝜕𝜕 − 𝜕𝜕𝜕𝜕� 𝑑𝑑𝑑𝑑 for the right side and

2𝜋𝜋 2𝜋𝜋
∮𝐶𝐶 𝑭𝑭 ∙ 𝒅𝒅𝒅𝒅 = ∫0 ⟨𝑓𝑓, 𝑔𝑔, 0⟩ ∙ 〈𝑥𝑥 ′(𝜃𝜃 ), 𝑦𝑦 ′(𝜃𝜃 ), 0〉𝑑𝑑𝑑𝑑 = ∫0 ⟨𝑓𝑓, 𝑔𝑔⟩ ∙ 〈𝑥𝑥 ′ (𝜃𝜃 ), 𝑦𝑦 ′(𝜃𝜃 )〉𝑑𝑑𝑑𝑑 = ∮𝐶𝐶 𝑭𝑭𝟐𝟐𝒅𝒅 ∙ 𝑻𝑻𝟐𝟐𝒅𝒅 𝑑𝑑𝑑𝑑
for the left side.

Hence Stokes’ theorem in 2d becomes

𝜕𝜕𝜕𝜕 𝜕𝜕𝜕𝜕
�𝑭𝑭𝟐𝟐𝟐𝟐 ∙ 𝑻𝑻𝟐𝟐𝟐𝟐 𝑑𝑑𝑑𝑑 = � � − � 𝑑𝑑𝑑𝑑
𝐶𝐶 𝐴𝐴 𝜕𝜕𝜕𝜕 𝜕𝜕𝜕𝜕

and is referred to as Green’s Circulation Theorem ∎.

We will discuss the applications of Green’s Circulation Theorem and some examples during the
second last week of term.

4.2.5 Green’s Circulation Theorem

Greens Theorem is used in several applications such as

• Computing the area of a region enclosed by a contour


• Proof of a theorem in complex number analysis, as discussed in MAT290F,
• Proof of Huygens’ principle which is a simple method of determining a diffraction pattern
when waves propagate through an infinitely long slit in a plate.

In these set of notes, we will formulate the process for computing an area using Green’s Circulation
Theorem.

Computing an area

A useful consequence of Green’s Circulation Theorem arises with the vector fields 𝑭𝑭 = 〈0, 𝑥𝑥 〉 and
𝜕𝜕𝜕𝜕 𝜕𝜕𝜕𝜕
𝑭𝑭 = 〈𝑦𝑦, 0〉. In the first case, we have = 1 and = 0, therefore by Green’s Circulation
𝜕𝜕𝜕𝜕 𝜕𝜕𝜕𝜕
Theorem,

105
�𝑭𝑭 ∙ 𝒅𝒅𝒅𝒅 = �𝑥𝑥𝑥𝑥𝑥𝑥 = � 𝑑𝑑𝑑𝑑
𝐶𝐶 𝐶𝐶 𝑅𝑅
𝜕𝜕𝜕𝜕 𝜕𝜕𝜕𝜕
In the second case, we have = 0 and = 1, therefore by Green’s Circulation Theorem,
𝜕𝜕𝜕𝜕 𝜕𝜕𝜕𝜕

�𝑭𝑭 ∙ 𝒅𝒅𝒅𝒅 = �𝑦𝑦𝑦𝑦𝑦𝑦 = − � 𝑑𝑑𝑑𝑑


𝐶𝐶 𝐶𝐶 𝑅𝑅
These two results can be combined in one statement

1
�𝑥𝑥𝑥𝑥𝑥𝑥 = − �𝑦𝑦𝑦𝑦𝑦𝑦 = �𝑥𝑥𝑥𝑥𝑥𝑥 − 𝑦𝑦𝑦𝑦𝑦𝑦 = � 𝑑𝑑𝑑𝑑
𝐶𝐶 𝐶𝐶 2 𝐶𝐶 𝑅𝑅

4.2.6 Example Green’s Circulation Problems

𝑥𝑥 2 𝑦𝑦 2
1. Compute the area of the ellipse 𝑎𝑎2 + 𝑏𝑏2 = 1 (refer to Archived Lecture Video 33)

Solution

An ellipse with a counter-clockwise orientation is described parametrically by

𝑟𝑟⃑(𝑡𝑡) = 〈𝑥𝑥, 𝑦𝑦〉 = 〈𝑎𝑎𝑎𝑎𝑎𝑎𝑎𝑎𝑎𝑎, 𝑏𝑏𝑏𝑏𝑏𝑏𝑏𝑏𝑏𝑏〉, for 0 ≤ 𝑡𝑡 ≤ 2𝜋𝜋. Noting that 𝑑𝑑𝑑𝑑 = −𝑎𝑎𝑎𝑎𝑎𝑎𝑎𝑎𝑎𝑎𝑎𝑎𝑎𝑎 and 𝑑𝑑𝑑𝑑 =
𝑏𝑏𝑏𝑏𝑏𝑏𝑏𝑏𝑏𝑏𝑏𝑏𝑏𝑏, we have:

𝑥𝑥𝑥𝑥𝑥𝑥 − 𝑦𝑦𝑦𝑦𝑦𝑦 = (𝑎𝑎𝑎𝑎𝑎𝑎𝑎𝑎𝑎𝑎)(𝑏𝑏𝑏𝑏𝑏𝑏𝑏𝑏𝑏𝑏𝑏𝑏𝑏𝑏) − (𝑏𝑏𝑏𝑏𝑏𝑏𝑏𝑏𝑏𝑏)(−𝑏𝑏𝑏𝑏𝑏𝑏𝑏𝑏𝑏𝑏)𝑑𝑑𝑑𝑑 = 𝑎𝑎𝑎𝑎(𝑐𝑐𝑐𝑐𝑐𝑐 2𝑡𝑡 + 𝑠𝑠𝑠𝑠𝑠𝑠2 𝑡𝑡)𝑑𝑑𝑑𝑑 = 𝑎𝑎𝑏𝑏𝑑𝑑𝑑𝑑

Expressing the line integral as an ordinary integral with respect to 𝑡𝑡, the area of the ellipse
is:

1 𝑎𝑎𝑎𝑎 2𝜋𝜋
�𝑥𝑥𝑥𝑥𝑥𝑥 − 𝑦𝑦𝑦𝑦𝑦𝑦 = � 𝑑𝑑𝑑𝑑 = 𝜋𝜋𝜋𝜋𝜋𝜋
2 𝐶𝐶 2 0

2. Consider the example from section 1.8 which included a pure rotation field 𝑭𝑭 = 〈−𝑦𝑦, 𝑥𝑥 〉.
Assume a disk 𝑅𝑅 = {(𝑥𝑥, 𝑦𝑦): 𝑥𝑥 2 + 𝑦𝑦 2 ≤ 𝑟𝑟 2 }. Integrating around a closed circular contour
with radius 2 gives ∮𝐶𝐶 𝑭𝑭 ∙ 𝑻𝑻𝑑𝑑𝑑𝑑 = 2𝜋𝜋𝑟𝑟 2, as discussed in Lecture 24. 𝐶𝐶 is oriented counter-
clockwise and is the boundary of 𝑅𝑅. Now confirm this result using Green’s Circulation
Theorem.

Solution
𝜕𝜕𝜕𝜕 𝜕𝜕𝜕𝜕
Note that 𝑓𝑓 (𝑥𝑥, 𝑦𝑦) = −𝑦𝑦 and 𝑔𝑔(𝑥𝑥, 𝑦𝑦) = 𝑥𝑥; therefore, the curl of 𝑭𝑭 is − = 2. By
𝜕𝜕𝜕𝜕 𝜕𝜕𝜕𝜕
Green’s Circulation Theorem,
2𝜋𝜋 𝑟𝑟
𝜕𝜕𝜕𝜕 𝜕𝜕𝜕𝜕
�𝑭𝑭 ∙ 𝒅𝒅𝒅𝒅 = � � − � 𝑑𝑑𝑑𝑑 = � 2𝑑𝑑𝐴𝐴 = 2 � � 𝑟𝑟 ′𝑑𝑑′𝑟𝑟𝑟𝑟𝑟𝑟 = 2𝜋𝜋𝑟𝑟 2
𝐶𝐶 𝐴𝐴 𝜕𝜕𝜕𝜕 𝜕𝜕𝜕𝜕 𝑅𝑅 0 0

106
At this point you can work on the following problems which have been extracted from section 17.4
of the textbook:

4.2.7 Problems from Section 17.4: 1, 6, 17, 23, 35, 43 (circulation), 47


(circulation), 53

1. (question 1) Explain why the two forms of Green’s theorem are analogs of the Fundamental
Theorem of Calculus
2. (question 6) Sketch a two-dimensional vector field that has zero curl everywhere in the
plane.
3. (question 17) Green’s Theorem Circulation form Consider the following region 𝑅𝑅 and
vector field 𝑭𝑭: 𝑭𝑭 = 〈2𝑦𝑦, −2𝑥𝑥 〉: 𝑅𝑅 is the region bounded by 𝑦𝑦 = 𝑠𝑠𝑠𝑠𝑠𝑠𝑠𝑠 and 𝑦𝑦 = 0, for 0 ≤
𝑥𝑥 ≤ 𝜋𝜋.
a. Compute the two-dimensional curl of the vector field.
b. Evaluate both integrals in Green’s Theorem and check for consistency.
4. (question 23) Area of regions Use a line integral on the boundary to find the area of the
following region: {(𝑥𝑥, 𝑦𝑦): 𝑥𝑥 2 + 𝑦𝑦 2 ≤ 16}.
5. (question 35) Line integrals Use Green’s Circulation Theorem to evaluate the following
line integral. Assume all curves are oriented clockwise. A sketch is helpful:
2 2
∮𝐶𝐶 �2𝑥𝑥 + 𝑒𝑒 𝑦𝑦 � 𝑑𝑑𝑑𝑑 − �4𝑦𝑦 2 + 𝑒𝑒 𝑥𝑥 �𝑑𝑑𝑑𝑑 where 𝐶𝐶 is the boundary of the square with vertices
(0,0), (1,0), (1,1) and (0,1).
6. (question 43) Circulation and flux For the following vector field, compute the circulation
on the boundary of the region. Assume boundary curves are oriented counterclockwise:
𝜋𝜋
𝑭𝑭 = 〈2𝑥𝑥 + 𝑦𝑦, 𝑥𝑥 − 4𝑦𝑦〉: 𝑅𝑅 is the quarter annulus �(𝑟𝑟, 𝜃𝜃 ): 1 ≤ 𝑟𝑟 ≤ 4, 0 ≤ 𝜃𝜃 ≤ �
2
7. (question 47) Circulation and flux For the following vector field, compute the circulation
on the boundary of the region. Assume boundary curves are oriented counterclockwise:
𝑭𝑭 = 〈𝑥𝑥 + 𝑦𝑦 2 , 𝑥𝑥 2 − 𝑦𝑦〉: 𝑅𝑅 = {(𝑥𝑥, 𝑦𝑦): 𝑦𝑦 2 ≤ 𝑥𝑥 ≤ 2 − 𝑦𝑦 2 }
8. (question 53) Area line integral Show that the value of ∮𝐶𝐶 𝑥𝑥𝑦𝑦 2 𝑑𝑑𝑑𝑑 + (𝑥𝑥 2 𝑦𝑦 + 2𝑥𝑥 )𝑑𝑑𝑑𝑑
depends only on the area of the region enclosed by 𝐶𝐶.

You can find the solutions to the problems at the following link: Solutions to Reading Assignment
4 Problems.

4.3 Conservative Vector Fields (Irrotational Vector Fields)

This section discusses the approach taken to model an irrotational field. Details are given in the
expanded set of notes. 2d and 3d irrotational fields and proofs are discussed.

4.4 Source Free Vector Field

107
This section discusses the approach taken to model a source free field. Details are given in the
expanded set of notes. 2d and 3d source free fields and proofs are discussed.

4.5 Computing the Field 𝑭𝑭 Given the Curl and Divergence of the Vector
Field 𝑭𝑭

In this section we show how Helmholtz’s Decomposition theorem is exploited to compute the field
𝑭𝑭. Details are provided in the expanded set of the notes.

4.6 A Sampling of Applications that Employ Vector Calculus Expressions

This section gives a brief survey of the issues that arise in modeling physical or man-made systems
described in terms of vector equations. The details for this section are in the expanded set of notes.

4.7 Applying Divergence and Stokes’ Theorem if Region is Not Simply


Connected or Simply Enclosed or the Region Contains Problematic
Points

Up to this point we have assumed that when applying the Divergence Theorem and Stokes’
Theorem, the following conditions are satisfied:

a. The partial derivatives of the vector components at each point on the open surface 𝑆𝑆𝑜𝑜 ,
bounded by a closed contour 𝐶𝐶 (2d and 3d Stokes’ theorem problems and 2d Divergence
theorem problems) and the partial derivatives of the vector components at each point within
a volume 𝑉𝑉, bounded by a closed surface 𝑆𝑆𝑐𝑐 (3d Divergence theorem problems) are well
behaved.
b. The vector components at each point on the closed contour 𝐶𝐶, (2d and 3d Stokes’ theorem
problems and Divergence theorem problems in 2d) and at each point on a closed surface
(𝑆𝑆𝑐𝑐 ) (3d Divergence theorem problems) are continuous functions.
c. A contour must completely enclose an open surface without having crossovers (2d and 3d
Stokes’ theorem problems and Divergence theorem problems in 2d) and a closed surface
must be a single surface which completely encloses a volume (3d Divergence theorem
problems).

Contours and regions

Contours: The categories used to characterize curves are (A) simple versus not simple and (B)
closed versus not closed (open). Hence there are four possible scenarios to consider, as shown
below and depicted in Fig. 18. A curve which is simple does not have any crossover points. Closed
curves enclose regions.

108
If a problem is 2d then the contour and region lie in the plane. If a problem is 3d, then the contour
can be in 3d and the region lies on a curved surface.

Category A Category B
simple closed
not simple closed
simple not closed
not simple not closed

Fig. 18 4 possible cases to consider for the contour

Regions on a Plane or Curved Surface: The categories used to characterize regions is (A)
connected versus not connected and (B) simply connected versus not simply connected. Hence
there are four possible scenarios to consider, as shown below and depicted in Fig. 19. A region
which is connected consists of a single region and not islanded regions. A region is simply
connected if every closed simple curve in the region can be deformed and contracted to a point
without exiting the region.

Category A Category B
connected simply connected
not connected simply connected
connected not simply connected
not connected not simply connected

Fig. 19 4 possible cases to consider for the region

109
We have assumed up to this point that contours are simple and closed curves and that regions are
connected and simply connected when applying Stokes’ theorem, Green’s Circulation Theorem,
and Green’s Flux Theorem.

There are cases in which we must consider a region which is connected but not simply connected.
An example where a region is not simply connected arises in the case of an annulus. For reference
purposes, the annulus case is discussed in Chapter 17.3 of the textbook. In this section we will
develop the tools to handle this case.

Surfaces and regions in 3d

Surfaces: We can use diagrams similar to that of Fig. 18 but coinsider the diagrams as traces rather
than as contours. So for example Fig. 18 (Closed simple) would be interpreted in 3d as a closed
surface.

Regions in 3d: We can use diagrams similar to that of Fig. 19 but consider the diagarams as traces.
The wording for Fig. 19 (Connected and not simply connected) would be replaced by (Enclosed
and not simply enclosed. The region would now be the volume between the inner closed surface
and the outer closed surface.

We have assumed up to this point a single surface that encloses a volume, which implies the region
is enclosed and simply enclosed, when applying the Divergence theorem.

An example where a region is not simply enclosed arises in the case of a hollowed-out sphere. The
hollowed-out sphere case is discussed in Chapter 17.8 of the textbook under the heading
“Divergence Theorem for Hollow Regions”. In this section we will develop the tools to handle
this case.

The purpose of this section is to show three things:

• It is possible to convert a not simply connected problem into a simply connected problem
so that Stokes’ theorem can be applied
• It is possible to convert a not simply enclosed problem into a simply enclosed problem so
that the Divergence theorem can be employed.
• It is possible to create a contour, surface, or region so that problematic points can be
avoided and thus Stokes’ theorem and the Divergence theorem can be employed.

In this section we will frame our discussion based on Fig. 20.

Fig. 20a shows an example of a connected but not simply connected 2d region. The region can be
converted into a simply connected region by introducing a cut, as shown in Fig. 20b. Consequently,
the region enclosed by the closed contour is now a simply connected region. The two sections of
the contour representing the cut are over top of each other, in a limiting process, and so the

110
contribution of the line integrals for these two straight sections (for 2d and 3d Stokes or 2d
Divergence) cancel out.
We now show how Stokes’ theorem for the 2d and 3d case and the Divergence theorem for the 2d
case are applied. Define contours as being in a counter-clockwise direction with 𝐶𝐶1 being the outer
contour and 𝐶𝐶2 being the inner contour. Applying Stokes’ theorem for the 2d case gives:
𝜕𝜕𝜕𝜕 𝜕𝜕𝜕𝜕
� 𝑭𝑭 ∙ 𝑻𝑻𝑑𝑑𝑑𝑑 − � 𝑭𝑭 ∙ 𝑻𝑻𝑑𝑑𝑑𝑑 = � � − � 𝑑𝑑𝑑𝑑
𝐶𝐶1 𝐶𝐶2 𝐴𝐴 𝜕𝜕𝜕𝜕 𝜕𝜕𝜕𝜕

where 𝑭𝑭 = 〈𝑓𝑓(𝑥𝑥, 𝑦𝑦), 𝑔𝑔(𝑥𝑥, 𝑦𝑦)〉. A similar expression exists for the 3d case. In ths case, the right-
hand side of Stokes’ theorem would become ∬𝑆𝑆 𝛁𝛁 × 𝑭𝑭 ∙ 𝒏𝒏𝑑𝑑𝑑𝑑 and 𝑭𝑭 would be a 3d vector field.

Applying the Divergence theorem for the 2d case gives:

� 𝑭𝑭 ∙ 𝒏𝒏𝒔𝒔 𝑑𝑑𝑑𝑑 − � 𝑭𝑭 ∙ 𝒏𝒏𝒔𝒔 𝑑𝑑𝑑𝑑 = � 𝛁𝛁 ∙ 𝑭𝑭𝑑𝑑𝑑𝑑


𝐶𝐶1 𝐶𝐶2 𝐴𝐴

where 𝐴𝐴 represents a planar area in the 𝑥𝑥𝑥𝑥 plane.

Fig. 20c shows a connected and simply connected region with two problematic points within the
enclosed region. The vector valued function 𝛁𝛁 × 𝑭𝑭 is undefined at these points. We can get around
this problem and still use Stokes’ theorem in 2d or 3d and the Divergence theorem in 2d by going
around these points, as shown in Fig. 20d. There are now two enclosed regions.

We will assume that the contours on the inner circles and the outer circles are counter-clockwise.
The upper half of the outer circle and the lower half of the outer circle when summed correspond
to a closed contour which we will refer to as 𝐶𝐶1 . Similarly, the two inner circles are labeled as 𝐶𝐶2
and 𝐶𝐶3 . We write out Stokes’ theorem in 2d or 3d or the Divergence theorem in 2d for both regions
and sum the results. After summation, the contributions of the line integrals for the straight sections
cancel. Now integrating around a closed contour, we notice that 𝐶𝐶2 and 𝐶𝐶3 , when following the
closed contour, are opposite in direction to the orientation of the contour for 𝐶𝐶1 . Applying Stokes’
theorem for the 2d case gives:
𝜕𝜕𝜕𝜕 𝜕𝜕𝜕𝜕
� 𝑭𝑭 ∙ 𝑻𝑻𝑑𝑑𝑑𝑑 − � 𝑭𝑭 ∙ 𝑻𝑻𝑑𝑑𝑑𝑑 − � 𝑭𝑭 ∙ 𝑻𝑻𝑑𝑑𝑑𝑑 = � � − � 𝑑𝑑𝑑𝑑
𝐶𝐶1 𝐶𝐶2 𝐶𝐶3 𝑆𝑆 𝜕𝜕𝜕𝜕 𝜕𝜕𝜕𝜕

A similar expression exists for the 3d case. However, the right-hand side of Stokes’ theorem would
become ∬𝑆𝑆 𝛁𝛁 × 𝑭𝑭 ∙ 𝒏𝒏𝑑𝑑𝑑𝑑 and 𝑭𝑭 would be a 3d vector field.

Applying the Divergence theorem for the 2d case gives:

� 𝑭𝑭 ∙ 𝒏𝒏𝒔𝒔 𝑑𝑑𝑑𝑑 − � 𝑭𝑭 ∙ 𝒏𝒏𝒔𝒔 𝑑𝑑𝑑𝑑 − � 𝑭𝑭 ∙ 𝒏𝒏𝒔𝒔 𝑑𝑑𝑑𝑑 = � 𝛁𝛁 ∙ 𝑭𝑭𝑑𝑑𝑑𝑑


𝐶𝐶1 𝐶𝐶2 𝐶𝐶3 𝐴𝐴

where 𝐴𝐴 represents a planar area in the 𝑥𝑥𝑥𝑥 plane.


111
(a) Hollowed out region (annulus) (b) Method of generating a closed (c) Two points at which vector field is
planar surface in 2d & curved surface in 3d contour that encloses the region undefined on a planar surface in 2d
(connected and not simply connected) (connected and simply connected)

(d) Method of avoiding regions with (e) Hollowed out sphere (f) Method of generating a closed surface
problematic points (enclosed and not simply enclosed) that encloses a volume by slicing and then
gluing the two volumes back
(each hemisphere is enclosed and simply
enclosed)

112
(g) Two points at which vector field is not (h) Method of avoiding problematic (i) Method of generating a closed surface that
defined points encloses a volume by slicing through the
volume and creating two volumes and then
gluing the two volumes back together

Fig. 20 Method for Applying the Stokes’ and Divergence Theorems for Problematic Cases

113
Fig. 20e gives a pictorial description of an enclosed and not simply enclosed region. Fig. 20f shows
the method for generating an enclosed and simply enclosed region so that the Divergence Theorem
can be applied. To do so, define the exterior closed surface of the sphere as 𝑆𝑆1 and the interior
closed surface of the sphere as 𝑆𝑆2 . The sphere is cut and then opened. The upper and lower
hemispheres each have three distinct surfaces forming a closed surface bounding the volume in
each hemisphere. By convention the surface area points outwards from a surface area element. We
apply the Divergence Theorem to both objects and sum the results. The sum of the flux on the top
and bottom flat surfaces cancels. Hence the Divergence theorem can be expressed as follows with
reference to Fig. 20f:

� 𝑭𝑭 ∙ 𝒅𝒅𝒅𝒅 − � 𝑭𝑭 ∙ 𝒅𝒅𝒅𝒅 = � 𝛁𝛁 ∙ 𝑭𝑭𝑑𝑑𝑑𝑑


𝑆𝑆1 𝑆𝑆2 𝑉𝑉

Fig. 20g shows the case where two problematic points within a region invalidate the applicability
of the Divergence theorem. The vector valued function 𝑭𝑭 is undefined at these points. Fig. 20h
shows how the problem can be avoided by going around the problematic points. A cut is
introduced, as shown in Fig. 20i, thus creating two separate regions that are each enclosed by a
surface. The outer upper hemisphere and outer lower hemisphere when summed correspond to a
closed surface which we will refer to as 𝑆𝑆1 . Similarly, the two closed inner surfaces are labeled as
𝑆𝑆2 and 𝑆𝑆3 . We will assume that the surface area vectors on the surfaces of the inner spheres and
the outer sphere point outwards.

We write out the Divergence Theorem in 3d for both regions and sum the results. After summation
the contributions of the surface integrals for the planar surfaces cancel. For the closed surfaces 𝑆𝑆2
and 𝑆𝑆3 , we notice that the direction vectors for 𝑆𝑆2 and 𝑆𝑆3 are inwards. Hence, we can write the
following final expression for the Divergence Theorem in 3d as follows:

� 𝑭𝑭 ∙ 𝒅𝒅𝒅𝒅 − � 𝑭𝑭 ∙ 𝒅𝒅𝒅𝒅 − � 𝑭𝑭 ∙ 𝒅𝒅𝒅𝒅 = � 𝛁𝛁 ∙ 𝑭𝑭𝑑𝑑𝑑𝑑


𝑆𝑆1 𝑆𝑆2 𝑆𝑆3 𝑉𝑉

4.7.1 Example Problems

1. Find the outward flux of the vector field 𝑭𝑭 = 〈𝑥𝑥𝑦𝑦 2 , 𝑥𝑥 2 𝑦𝑦〉 across the boundary of the annulus
𝑅𝑅 = {(𝑥𝑥 , 𝑦𝑦): 1 ≤ 𝑥𝑥 2 + 𝑦𝑦 2 ≤ 4} = {(𝑟𝑟, 𝜃𝜃 ): 1 ≤ 𝑟𝑟 ≤ 2, 0 ≤ 𝜃𝜃 ≤ 2𝜋𝜋} (refer to Archived
Lecture Video 32)

Solution

Because the annulus is not simply connected, Green’s Theorem does not apply. This
difficulty is overcome by defining a curve 𝐶𝐶 as shown in Fig. 20b, which is simple, closed,
and piecewise smooth. The connecting links along the 𝑥𝑥 axis is parallel and are traversed
in an opposite direction. Therefore, the contributions to the line integral cancel on both
lines. Because of this cancellation, we take 𝐶𝐶 to be the curve that runs counterclockwise on

108
the outer boundary and clockwise on the inner boundary. Using the flux form of Green’s
Theorem and converting to polar coordinates, we have:

�𝑭𝑭 ∙ 𝒏𝒏𝒔𝒔 𝑑𝑑𝑑𝑑 = �𝑓𝑓𝑓𝑓𝑓𝑓 − 𝑔𝑔𝑔𝑔𝑔𝑔 = �𝑥𝑥𝑦𝑦 2 𝑑𝑑𝑑𝑑 − 𝑥𝑥 2 𝑦𝑦𝑦𝑦𝑦𝑦 = � (𝑦𝑦 2 + 𝑥𝑥 2 )𝑑𝑑𝑑𝑑
𝐶𝐶 𝐶𝐶 𝐶𝐶 𝑅𝑅
2𝜋𝜋 2 2𝜋𝜋 4 2
𝑟𝑟 15 2𝜋𝜋 15𝜋𝜋
=� � (𝑟𝑟 2 ) 𝑟𝑟𝑟𝑟𝑟𝑟𝑟𝑟𝑟𝑟 = � � 𝑑𝑑𝑑𝑑 = � 𝑑𝑑𝑑𝑑 =
0 1 0 4 1 4 0 2

2. Find the outward flux of the vector field 𝑭𝑭 = 〈𝑥𝑥 3 , 𝑦𝑦 3 , 𝑧𝑧 3 〉 across the boundary of the
hollowed-out sphere 𝑅𝑅 = {(𝑥𝑥 , 𝑦𝑦, 𝑧𝑧): 1 ≤ 𝑥𝑥 2 + 𝑦𝑦 2 + 𝑧𝑧 2 ≤ 4} = {(𝑟𝑟, 𝜑𝜑, 𝜃𝜃 ): 1 ≤ 𝑟𝑟 ≤ 2, 0 ≤
𝜑𝜑 ≤ 𝜋𝜋, 0 ≤ 𝜃𝜃 ≤ 2𝜋𝜋}.

Solution

Because the hollowed-out sphere is not simply enclosed, the Divergence Theorem does not
apply. This difficulty is overcome by defining a surface 𝑆𝑆𝑎𝑎 and 𝑆𝑆𝑏𝑏 , shown in Fig. 20f, which
are enclosed and piecewise smooth. The connecting surfaces along the 𝑥𝑥𝑥𝑥 plane is parallel
and are oriented in opposite directions. Therefore, when adding the surface integrals for
the top and bottom enclosed regions, the contributions of the surfaces on the 𝑥𝑥𝑥𝑥 plane to
the total surface integral cancel on both surfaces. Because of this cancellation, we take 𝑆𝑆1 =
𝑆𝑆1𝑎𝑎 ∪ 𝑆𝑆1𝑏𝑏 to be the surface that points outwards and 𝑆𝑆2 = 𝑆𝑆2𝑎𝑎 ∪ 𝑆𝑆2𝑏𝑏 to be the surface that
points inwards. Using the Divergence Theorem and converting to spherical coordinates,
we have the net flux: ∯𝑆𝑆 𝑭𝑭 ∙ 𝒅𝒅𝒅𝒅 − ∯𝑆𝑆 𝑭𝑭 ∙ 𝒅𝒅𝒅𝒅 = ∭𝑉𝑉 𝛁𝛁 ∙ 𝑭𝑭𝑑𝑑𝑑𝑑.
1 2

2𝜋𝜋 𝜋𝜋 2 2𝜋𝜋 𝜋𝜋
3 52
� 𝛁𝛁 ∙ 𝑭𝑭𝑑𝑑𝑑𝑑 = � � � (3𝑟𝑟 2 )𝑟𝑟 2 𝑠𝑠𝑠𝑠𝑠𝑠𝑠𝑠𝑠𝑠𝑠𝑠𝑠𝑠𝑠𝑠𝑠𝑠𝑠𝑠 = � � 𝑟𝑟 � 𝑠𝑠𝑠𝑠𝑠𝑠𝑠𝑠𝑠𝑠𝑠𝑠𝑠𝑠𝑠𝑠𝑠𝑠𝑠𝑠
𝑉𝑉 0 0 1 0 0 5 1
2𝜋𝜋 𝜋𝜋
93 372𝜋𝜋
= � � 𝑠𝑠𝑠𝑠𝑠𝑠𝑠𝑠𝑠𝑠𝑠𝑠𝑠𝑠𝑠𝑠 =
5 0 0 5

At this point you can work on the following problems which have been extracted from section 17.4
and 17.8 of the textbook:

4.7.2 Problems from Section 17.8: 27, Section 17.4: 66, 67, 17.8: 39b

1. (question 27) Divergence Theorem for more general regions Use the Divergence
Theorem to compute the net outward flux of the following vector field across the boundary
𝒓𝒓 〈𝑥𝑥.𝑦𝑦,𝑧𝑧〉
of the given region 𝐷𝐷: 𝑭𝑭 = |𝒓𝒓| = : 𝐷𝐷 is the region between the spheres of radius
�𝑥𝑥 2 +𝑦𝑦 2+𝑧𝑧 2
1 and 2 centered at the origin.
〈−𝑦𝑦,𝑥𝑥〉
2. (question 66) What’s wrong Consider the rotation field 𝑭𝑭 = 𝑥𝑥2 +𝑦𝑦2.

109
a. Verify that the two-dimensional curl of 𝑭𝑭 is zero, which suggests that the double
integral in the circulation form is of Green’s Theorem is zero.
b. Use a line integral to verify that the circulation on the unit circle of the vector field
is 2𝜋𝜋.
c. Explain why the results from part(a) and part(b) do not agree.
〈𝑥𝑥,𝑦𝑦〉
3. (question 67) What’s wrong Consider the radial field 𝑭𝑭 = 𝑥𝑥2 +𝑦𝑦2.
a. Verify that the divergence of 𝑭𝑭 is zero, which suggests that the double integral in
the flux form is of Green’s Theorem is zero.
b. Use a line integral to verify that the outward flux on the unit circle of the vector
field is 2𝜋𝜋.
c. Explain why the results from part(a) and part(b) do not agree.
4. (question 39b) Gauss’ Law (Divergence Theorem) for electric fields The electric field
𝑄𝑄 𝒓𝒓
due to a point charge 𝑄𝑄 is 𝑬𝑬 = |𝒓𝒓|3
, where 𝒓𝒓 = 〈𝑥𝑥, 𝑦𝑦, 𝑧𝑧〉 and 𝜀𝜀𝑜𝑜 is a constant.
4𝜋𝜋𝜀𝜀𝑜𝑜
a. Let 𝑆𝑆 be the boundary of the region between two spheres centered at the origin of
radius 𝑎𝑎 and 𝑏𝑏, respectively with 𝑎𝑎 < 𝑏𝑏. Use the Divergence Theorem to show that
the net outward flux across 𝑆𝑆 is zero.
5. Suppose that 𝑭𝑭 is a continuously differentiable vector field. Let 𝑆𝑆𝑅𝑅 denote the sphere of
radius 𝑅𝑅 that is centred at (0,0,0) and oriented outwards. Let 0 < 𝑅𝑅1 < 𝑅𝑅2 , and suppose as
well that |∇ ∙ 𝑭𝑭| ≤ 𝜖𝜖. Show that:
4𝜋𝜋𝜋𝜋 3
�� 𝑭𝑭 ∙ 𝒏𝒏𝑑𝑑𝑑𝑑 − � 𝑭𝑭 ∙ 𝒏𝒏𝑑𝑑𝑑𝑑� ≤ (𝑅𝑅2 − 𝑅𝑅13 )
𝑆𝑆𝑅𝑅2 𝑆𝑆𝑅𝑅1 3

You can find the solutions to the problems at the following link: Solutions to Reading Assignment
4 Problems.

4.8 Identities Exploiting the Divergence Theorem and Stokes’ Theorem

This section describes several identities that can be derived using the Divergence Theorem and
Stokes’ Theorem and are of practical use in the derivation of other expressions. These identities
are only for your information. The details can be found in the expanded set of notes.

4.9 Time Derivatives of Integral Quantities

This last section deals with the time derivative of integral quantities which we have encountered
so far. In this case the object in question is moving or the object is stationary, and the medium is
moving. We are interested in the direct time derivative of an integrated quantity. This material is
covered in the expanded set of notes.

110

You might also like